Você está na página 1de 234

Anlisis Funcional y Variable

Compleja
Ignacio Villanueva Dez
ndice general
Introduccin 3
Captulo 1. Espacios normados. Espacios de Banach 7
Captulo 2. Aplicaciones lineales continuas entre espacios
normados 37
Captulo 3. Teoremas de Hahn-Banach 49
Captulo 4. El Teorema de Baire y sus consecuencias: El Principio
de Acotacin Uniforme y el Teorema de la Grca
Cerrada. 75
Captulo 5. Espacios duales y operadores traspuestos 103
Captulo 6. Topologas dbil y dbil

119
Captulo 7. Operadores compactos 135
Captulo 8. Teora espectral de operadores compactos 149
Captulo 9. Espacios de Hilbert 171
Captulo 10. Teora espectral en espacios de Hilbert: Operadores
compactos normales 201
Captulo 11. Requisitos 219
Captulo 12. Funciones holomorfas 221
Captulo 13. Funciones meromorfas 223
Captulo 14. Teorema de la Aplicacin abierta 225
Captulo 15. Topologa compacto-abierta 227
Captulo 16. Funciones Armnicas 229
Bibliografa 231
1
Introduccin
0.1. El Anlisis Funcional. El Anlisis Funcional como tal fue
surgiendo a principios del siglo XX como el marco abstracto adecuado
para solucionar una serie de problemas del Anlisis muy importantes
en esos momentos.
Desde entonces ha experimentado un gran desarrollo y en este mo-
mento es una herramienta muy sosticada til para abordar una amplia
variedad de problemas. En [11] se puede ver un descripcin del pro-
ceso histrico que desemboc en la aparicin del Anlisis Funcional.
Incluimos aqu tan slo una breve descripcin de este proceso que nos
permita ubicar nuestra materia en el seno de la matemtica.
Desde el desarrollo del Clculo Diferencial, al considerar las solu-
ciones de una ecuacin diferencial, se vio que en ocasiones era nece-
sario considerar propiedades del espacio (o conjunto) de soluciones de
la ecuacin, pero no estaba claro cul era la estructura que posea dicho
espacio de soluciones. Los trabajos de D. Bernouilli, Lagrange y sobre
todo Fourier acerca de la resolucin de ecuaciones diferenciales se em-
piezan a enfrentar a problemas que anticipan lo que ser el desarrollo
posterior del Anlisis Funcional. Una de las caractersticas comunes a
varios de estos procesos era el paso de un problema nito, por ejemp-
lo la solucin de un sistema nito de ecuaciones lineales, a la versin
innita del problema, lo que les fuerza a enfrentarse a problemas de
convergencia que en esa poca no eran entendidos.
Merece tambin la pena mencionar los trabajos de Sturm (1836)
y Liouville (1837) sobre la solucin de ecuaciones diferenciales de la
forma
y
tt
q(x)y + y = 0
con ciertas condiciones de contorno, que inuyeron notablemente en el
desarrollo de la Teora Espectral.
Con el desarrollo del Clculo de Variaciones aparece ya explcita-
mente la nocin de campo funcional y de distancia entre funciones y
aparecen de forma implcita algunos de los problemas provocados por la
3
4 INTRODUCCIN
no compacidad de los conjuntos cerrados y acotados en espacios innito
dimensionales.
Uno de los problemas que ms inuy en la aparicin del Anlisis
Funcional fue el estudio de las Ecuaciones Integrales. Los trabajos de
varios matemticos en este rea llevaron al desarrollo por Fredholm de
una tcnica para solucionar algunos de estos problemas que dio lugar
a lo que hoy conocemos como Alternativa de Fredholm.
A partir de ese momento la situacin estaba sucientemente madura
para el comienzo de la cristalizacin del Anlisis Funcional. A principios
del siglo XX, Hilbert se interes por los resultados de Fredholm y se
dedic al estudio de las ecuaciones integrales, publicando una serie de
artculos en los que van apareciendo de forma ms o menos explcita las
nociones de autovalor, ortogonalidad de los distintos autoespacios, y la
misma nocin de bola unidad de un espacio de Hilbert (obviamente no
en esos trminos).
En el ltimo de dichos artculos, Hilbert abandona el marco de las
ecuaciones integrales y da el salto de abstraccin que caracteriza al
Anlisis Funcional: intenta desarrollar una teora abstracta de formas
bilineales y cuadrticas de innitas variables para luego aplicar dicha
teora el estudio de las ecuaciones integrales. En el desarrollo de esta
teora aparece ya de forma bastante explcita el espacio
2
, la convergen-
cia dbil de sucesiones en
2
y su principio de eleccin, que en trminos
modernos podemos pensar como el Teorema de Eberlein para el caso
particular de
2
.
A continuacin, en los trabajos de una serie de autores (Frechet,
Schmidt, F. Riesz, Fischer entre otros) se fueron desarrollando muchas
nociones que hoy son centrales en el Anlisis Funcional como la distan-
cia en un conjunto abstracto y desde ah diversas nociones topolgicas
como completitud, compacidad, separabilidad, los espacios L
p
y
p
, el
estudio de sus duales y el dual de C[a, b]. En un artculo seminal de F.
Riesz en 1918 se desarrolla la Teora de Operadores Compactos sobre
espacios C[a, b], pero el propio Riesz dice que la teora se puede exten-
der a otros espacios funcionales. En ese artculo aparece la denicin
de la norma de C[a, b].
Hemos citado algunos autores y artculos, pero hay muchos ms de
esa poca que hicieron que la situacin estuviese madura para la apari-
cin en 1922 de la Tesis de Banach, en la que se desarrolla una teora
general de espacios normados y operadores lineales entre ellos. Desde
ese momento y hasta 1932 el Anlisis Funcional vivi una dcada de
vertiginoso desarrollo, en la que se sistematiz la aplicacin de mtodos
topolgicos al estudio de los espacios de Banach (B-espacios en la no-
tacin del propio Banach) y se probaron entre otros muchos resultados
INTRODUCCIN 5
el Principio de Acotacin Uniforme (que ya haba aparecido en una
forma ms dbil en la tesis de Banach), el Teorema de Hahn-Banach y
el Teorema de la Aplicacin Abierta, tres de los principales resultados
de la asignatura que proponemos.
En esa misma dcada, von Neumann utiliza la nocin de espacio
de Hilbert abstracto (ya no exclusivamente
2
o L
2
) para satisfacer una
necesidad de los fsicos de la poca: la construccin del lenguaje formal
adecuado para la Mecnica Cuntica, que sigue siendo esencialmente
el utilizado hoy en da.
En 1932 el Anlisis Funcional alcanza la mayora de edad con la
aparicin de los libros Theorie des Operations Lineaires, de Banach,
Grundlagen der Quantenmechanik, de von Neumann, y Linear Trans-
formations in Hilbert Space, de Stone. Prcticamente todo el contenido
de nuestra asignatura (y mucho ms) se halla (con otro lenguaje, por
supuesto) en esos libros.
Desde entonces el desarrollo del Anlisis Funcional ha sido muy
amplio, y no nos atrevemos ni siquiera a incluir aqu un pequeo re-
sumen de este, aunque s nos gustara mencionar brevemente que la
necesidad de estudiar espacios funcionales no normados dio lugar a la
Teora de Espacios Vectoriales Topolgicos y en particular al estudio
de los Espacios Localmente Convexos de donde surgi la Teora de las
Distribuciones que ha tenido tantas aplicaciones en el estudio de las
Ecuaciones Diferenciales.
Tambin a lo largo del siglo pasado numerosas reas de la matemti-
ca han ido adoptando parte del lenguaje y las herramientas del Anlisis
Funcional, debido a su gran potencia.
CAPTULO 1
Espacios normados. Espacios de Banach
La base abstracta del Anlisis Funcional consiste en el estudio de
Espacios Funcionales. Estos espacios tienen una doble estructura que
nos permitir su estudio. De un lado una estructura algebraica de espa-
cio vectorial (habitualmente de dimensin innita) y de otro lado una
estructura topolgica bien relacionada con la estructura algebraica.
La forma ms natural de denir una topologa en un espacio vectori-
al, siempre que sea posible, es mediante una distancia asociada a una
norma, y de hecho los primeros espacios funcionales y probablemente
los ms utilizados son espacios normados, cuya denicin y estudio
introducimos en este captulo.
A continuacin introducimos la nocin de espacio de Banach, puesto
que la completitud ser una de las herramientas ms tiles a nuestra
disposicin.
En muchas ocasiones necesitaremos estudiar subespacios y cocientes
de espacios de Banach y normados, y por ellos vemos aqu que ambos
respetan esas estructuras.
Mediante el Lema de Riesz probamos la no compacidad de la bola
unidad de los espacios de dimensin innita. Este es uno de los leit-
motiv del Anlisis Funcional, puesto que buena parte de las tcnicas
que estudiaremos van precisamente destinadas a remedar dicha falta
de compacidad. Estudiamos tambin la compacidad en el caso nito-
dimensional, lo que nos da pie a hablar de normas equivalentes, que
volvern a aparecer ms adelante cuando hablemos de isomorsmos
entre espacios de Banach.
Los espacios de Banach habitualmente llamados clsicos son C(K),
c
0
, los
p
y los L
p
. C(K) es fcil de denir. La construccin de los
p
es
sencilla una vez probada la desigualdad de Hlder, til por s misma
en muchas ocasiones.
Puesto que nuestros alumnos previsiblemente no conocern Teora
de la Medida, no podemos abordar la construccin de los espacios L
p
()
en general. Sin embargo, no creemos adecuado ensear Anlisis Fun-
cional sin hablar al menos de los espacios L
p
[0, 1]. Dado que no pode-
mos estar seguros de que nuestros alumnos conozcan la Teora de la
7
8 1. ESPACIOS NORMADOS. ESPACIOS DE BANACH
Integracin de Lebesgue, y dado que una presentacin rigurosa de esta
nos llevara demasiado tiempo, hemos decidido incluir en este captu-
lo una presentacin de dicha teora escueta y sin demostraciones, pero
suciente para poder construir los espacios L
p
[0, 1] y demostrar su com-
pletitud.
Para la preparacin de este captulo hemos seguido sobre todo [24],
[31] y [13]. La exposicin de la Teora de la Integracin de Lebesgue
se basa principalmente en [40] y [42].
Espacios normados y espacios de Banach
Empezamos recordando la denicin de norma de un espacio vec-
torial.
Definicin 1.1. Sea X un espacio vectorial sobre R o C. Una
funcin [[ [[ : X [0, +) es una norma en X si:
1. [[x[[ = 0 si y slo si x = 0.
2. [[x[[ = [[ [[x[[, para todo x X y R ( C).
3. [[x + y[[ [[x[[ +[[y[[, para todos x, y X.
Un espacio vectorial con una norma (X, [[[[) es un espacio vectorial
normado.
Veremos a lo largo de la asignatura que la completitud juega un
importante papel en muchos de los razonamientos caractersticos del
Anlisis Funcional, por lo que habitualmente trabajaremos con espacios
normados y completos. Esta es precisamente la denicin de un Espacio
de Banach llamados as porque fue Stefan Banach quien comenz su
estudio sistemtico en [6] y sobre todo en [7].
Dado un espacio vectorial normado (X, | |), automticamente
posee una estructura natural de espacio mtrico, donde la mtrica
d : X X [0, +) viene dada por
d(x, y) = |x y|
para todos x, y X. Es fcil ver que, as denida, d es efectivamente
una distancia. Para comprobar la propiedad triangular, observamos que
d(x, z) = |xz| = |xy+yz| |xy|+|yz| = d(x, y)+d(y, z)
1. ESPACIOS NORMADOS. ESPACIOS DE BANACH 9
Dos observaciones son importantes: en primer lugar, observar que
la norma es uniformemente continua, dado que
[|x| |y|[ |x y|
(Para demostrar esto, ntese que
|x| = |x y + y| |x y| +|y|
y por tanto
|x| |y| |x y|.
Anlogamente se tiene que
|y| |x| |x y|.)
En segundo lugar es muy importante notar que la norma dota a
X de una topologa bien relacionada con su estructura vectorial, en
el sentido de que tanto la suma como el producto por escalares son
continuos. Vemoslo.
Sea > 0. Entonces dados x, y X, para todos z
1
B(x,

2
),
z
2
B(y,

2
),
|z
1
+z
2
(x+y)| = |x+z
1
x+y+z
2
yxy| |z
1
x|+|z
2
y| = .
Anlogamente, dados x X, k K (R C), entonces tomando
por ejemplo

1
= mn

3|x|
,

3
,
2
= mn

3[k[
,

se tiene que, para todos B(k,


1
) z B(x,
2
),
|z kx| = |(k + k)(x + z x) kx| .
De aqu se sigue que la topologa denida por la norma viene car-
acterizada por los entornos del origen.
Ejemplo 1.2. 1. Ya se han estudiado en cursos anteriores
normas en espacios nito dimensionales. En particular se sabe
que

n
i
:= (K
n
, | |
i
)
es un espacio de Banach para i = 1, 2, con las deniciones
habituales. Mas adelante veremos que se sigue de la desigual-
dad de Minkowsky que
n
p
es un espacio de Banach para todo
1 p .
2. Dado un conjunto T, podemos denir B(T) como el espacio
de las funciones escalares acotadas denidas sobre T. Se ve
fcilmente que la norma del supremo | |

denida como
|f|

= sup
xT
[f(x)[
10 1. ESPACIOS NORMADOS. ESPACIOS DE BANACH
es efectivamente una norma en B(T). Veamos que con esta
norma B(T) es un espacio de Banach.
Sea (f
n
)
nN
B(T) una sucesin de Cauchy. Entonces
para cada x T la sucesin (f
n
(x))
nN
tambin es de Cauchy.
Por tanto existe el lm
n
f
n
(x). Denimos entonces la funcin
f : T K como
f(x) = lm
n
f
n
(x)
Por ser la sucesin (f
n
) de Cauchy en la norma del supre-
mo, se tiene que la sucesin est uniformemente acotada, y por
tanto f est acotada. Slo nos falta ver que f
n
tiende a f en la
norma del supremo. Sea > 0. Entonces existe n
0
N tal que
para todos n, m n
0
|f
n
f
m
| < . Sea ahora x T. Sabemos
que existe m n
0
tal que [f(x) f
m
(x)[ < . Entonces
[f(x) f
n
0
(x)[ [f(x) f
m
(x)[ +[f
m
(x) f
n
0
(x)[ 2.
Ntese que para cada x T m puede variar, pero siempre
existe algn m n
0
para el que se tiene el resultado, y por
tanto
lm
n
|f f
n
|

= 0.
La demostracin es interesante no tanto por el resultado
en s mismo, sino porque esta es a menudo la tcnica seguida
para vericar que un espacio normado es completo: se parte de
una sucesin de Cauchy, se encuentra un candidato a lmite (a
menudo, como aqu, utilizando la completitud de los escalares),
y luego se verica que (1) el candidato a lmite est en el es-
pacio, y (2) la sucesin converge efectivamente a ese lmite en
la norma del espacio.
3. Sea K un espacio compacto de Hausdor. Denimos C(K) co-
mo el espacio de las funciones f : K K continuas, dotado
de la norma del supremo, | |

. Se ve igual que en el ejemplo


anterior que | |

es una norma. Veamos que con esta norma


C(K) es un espacio de Banach: dado que C(K) es un subespa-
cio vectorial de B(K), las funciones acotadas sobre K, puesto
que ya hemos visto que B(K) es completo slo es necesario
demostrar que C(K) es un subespacio cerrado de B(K). Sea
entonces (f
n
) C(K) una sucesin tal que f
n
f B(K).
Slo hay que ver que f es continua. Esto es un ejercicio de
topologa, demostrar que el lmite uniforme de funciones con-
tinuas es una funcin continua. Lo detallamos: Sea > 0.
1. ESPACIOS NORMADOS. ESPACIOS DE BANACH 11
Existe n
0
N tal que para todo n n
0
|f
n
f| < .
Sea ahora t
0
K. Existe un entorno V de t
0
tal que para todo
t V
[f
n
0
(t
0
) f
n
0
(t)[ < .
Entonces, para todo t V
[f(t) f(t
0
)[ [f(t) f
n
0
(t)[ +[f
n
0
(t) f
n
0
(t
0
)[+
+[f
n
0
(t
0
) f(t
0
)[ 3
y por tanto f es continua.
4. Veremos ms adelante que siempre que podamos denir un
producto escalar , ) en X, ste dene una norma en X me-
diante
|x|
2
= x, x).
Los espacios de Banach cuya norma procede de un producto
escalar se llaman espacios de Hilbert. Su importancia justica
que se estudien en un captulo aparte.
Sabemos del estudio de los espacios vectoriales la importancia que
tiene, dado un espacio vectorial, el estudio de sus subespacios y sus
espacios cocientes. Por tanto, resulta tambin importante ver cmo
ambos heredan la estructura de espacio de Banach del espacio original.
Proposicin 1.3. Sea (X, | |) un espacio vectorial normado y sea
Y un subespacio vectorial de X. Entonces tanto Y como su clausura
Y son espacios normados con la norma heredada. Adems, si X es un
espacio de Banach Y tambin lo es.
Demostracin. Ntese primero que dado que la suma y el pro-
ducto por escalares son continuos para la norma, Y es efectivamente
un subespacio vectorial de X: Veamos por ejemplo que Y est cerrado
para la suma: Sean x, y Y , y sea > 0. Existen x
t
, y
t
Y tales que
|x
t
x| , |y
t
y| . Entonces x
t
+ y
t
Y y
|x
t
+ y
t
(x + y)| 2.
Es trivial que | | induce una norma tanto en Y como en Y . Adems,
si X es un espacio de Banach entonces Y es un subespacio cerrado de
un espacio completo, y por tanto l mismo es completo.
Proposicin 1.4. Sea (X, | |) un espacio vectorial normado y sea
Y X un subespacio vectorial cerrado. Dado [x] X/Y , denimos
|[[x]|[ = nf
yY
|x + y|.
12 1. ESPACIOS NORMADOS. ESPACIOS DE BANACH
Entonces |[ |[ es una norma en X/Y llamada la norma cociente.
Adems si X es un espacio de Banach entonces X/Y tambin es un
espacio de Banach.
Demostracin. Sea x X. Claramente |[[x]|[ 0. Si |[[x]|[ = 0
entonces existe una sucesin (y
n
)
n
Y tal que
|x y
n
| <
1
n
.
Entonces y
n
x y como Y es cerrado se tiene que x Y , luego [x] = 0.
Para ver la desigualdad triangular, sean x
1
, x
2
X y sean y
1
, y
2
Y
tales que
|x
1
+ y
1
| < |[[x
1
]|[ +
y
|x
2
+ y
2
| < |[[x
2
]|[ + .
Entonces
|[[x
1
] +[x
2
]|[ = |[[x
1
+x
2
]|[ = nf
yY
|x
1
+x
2
+y| |x
1
+x
2
+y
1
+y
2
|
|x
1
+ y
1
| +|x
2
+ y
2
| < |[[x
1
]|[ +|[[x
2
]|[ + 2
y como esto es cierto para todo > 0 se sigue la propiedad triangular.
Finalmente, si x X y 0 ,= k K, se tiene
|[k[x]|[ = |[[kx]|[ = nf
yY
|kx + y| = nf
yY
[k[
_
_
_x +
y
k
_
_
_ =
= [k[ nf
yY
|x + y| = [k[|[[x][|,
y por tanto |[ [| es una norma en X/Y .
Veamos nalmente que si X es un espacio de Banach entonces X/Y
tambin lo es. Sea ([x
n
])
nN
X/Y una sucesin de Cauchy. Elijamos
una subsucesin (n
k
)
k
tal que
|[[x
n
k
] [x
n
k+1
][| < 2
k
.
Como
|[x
n
1
] [x
n
2
]| = |[x
n
1
x
n
2
]| <
1
2
,
se tiene que existe y
n
2
Y tal que
|x
n
1
(x
n
2
+ y
n
2
)| <
1
2
.
A continuacin, puesto que
|[x
n
2
] [x
n
3
]| = |[x
n
2
x
n
3
]| <
1
4
,
1. ESPACIOS NORMADOS. ESPACIOS DE BANACH 13
existe y
n
3
Y tal que
|(x
n
2
+ y
n
2
) (x
n
3
+ y
n
3
)| <
1
4
.
Continuando el proceso se tiene la existencia de una sucesin (y
n
k
)
k
Y
tal que
|(x
n
k
+ y
n
k
) (x
n
k+1
+ y
n
k+1
)| < 2
k
y por tanto podemos elegir los representantes x
n
k
de manera que
|x
n
k
x
n
k+1
| < 2
k
.
De aqu se sigue que (x
n
k
)
k
es de Cauchy en X y por la completitud
de X existe x X tal que x
n
k
tiende a x. Como
|[x
n
k
] [x]| = |[x
n
k
x]| |x
n
k
x|
se sigue que [x
n
k
] [x] y por ser ([x
n
])
n
de Cauchy se tiene nalmente
que [x
n
] [x]
Observacin 1.5. A la hora de pensar en la norma cociente, es a
menudo til darse cuenta de que
|[x]| = nf
yY
|x + y| = nf
yY
|x y| = dist(x, Y ).
Queremos estudiar la compacidad de los subconjuntos de espacios
normados. Empecemos recordando la situacin en dimensin nita.
Previamente necesitamos la nocin de normas equivalentes, que volver
a aparecer ms adelante.
Definicin 1.6. Sea X un espacio vectorial. Dadas dos normas
| |
1
y | |
2
sobre X, decimos que son equivalentes si inducen la misma
topologa.
Proposicin 1.7. Dos normas ||
1
y ||
2
sobre X son equivalentes
si y slo si existen dos constantes c, C (0, +) tales que, para todo
x X
c|x|
1
|x|
2
C|x|
1
Demostracin. Supongamos que existen c, C como en el enunci-
ado. Sea x
0
X y > 0. Entonces
B
1
(x
0
,

C
) B
2
(x
0
, )
y
B
2
(x
0
, c) B
2
(x
0
, )
14 1. ESPACIOS NORMADOS. ESPACIOS DE BANACH
lo que muestra que las dos topologas son equivalentes.
Recprocamente, supongamos que las dos topologas son equiva-
lentes. Por tanto el conjunto B
1
(0, 1) es un entorno abierto del 0 en la
topologa dada por la norma 2, y por ello existe r > 0 tal que
B
2
(0, r) B
1
(0, 1).
De aqu se sigue que, para todo x X,
|x|
1
r
1
|x|
2
.
La otra desigualdad se obtiene anlogamente.
Sea X un espacio vectorial en el que hay denidas dos normas
equivalentes | |
1
y | |
2
. Entonces la aplicacin identidad
Id : (X, | |
1
) (X, | |
2
)
es lineal, continua, biyectiva, y su inversa tambin es lineal y continua,
como se ve al principio del Captulo 3. Dos espacios normados entre
los que podemos establecer una aplicacin con esas caractersticas se
dicen isomorfos.
Proposicin 1.8. Sea X un espacio vectorial de dimensin nita.
Entonces dos normas cualesquiera en X son equivalentes.
Demostracin. Suponemos X = K
n
y consideramos en el la nor-
ma 1 | |
1
y otra norma cualquiera | |. Si (e
i
)
n
i=1
es la base cannica
de K
n
, se tiene
|x| =
_
_
_

x
i
e
i
_
_
_

[x
i
[|e
i
|
_
max
i
|e
i
|
_
|x|
1
.
Esto nos da una de las desigualdades buscadas y nos dice que
Id : (X, | |
1
) (X, | |)
es continua, de donde | | es una funcin continua sobre (X, | |
1
).
Puesto que S
(X,||
1
)
es un compacto (probamos este hecho ms ade-
lante), la funcin | |, que es estrictamente positiva en S
(X,||
1
)
alcanza
su mnimo ah al que llamamos y se tiene > 0. Por tanto
| |
1

1

| |
que era la otra desigualdad buscada.
Slo resta por tanto comprobar que efectivamente S
(X,||
1
)
es un
compacto. Consideremos la sucesin (x
k
)
kN
S
(X,||
1
)
donde para
todo k N
x
k
= (x
1
k
, . . . , x
n
k
).
1. ESPACIOS NORMADOS. ESPACIOS DE BANACH 15
Se tiene que |x
k
| =

n
i=1
[x
i
k
[ = 1 y por tanto para todo 1 i n
la sucesin (x
i
k
)
kN
est acotada. Podemos entonces, tomando subsuce-
siones repetidas veces, extraer una sucesin de ndices (k
l
)
lN
de manera
que para todo 1 i n la sucesin (x
i
k
l
)
lN
converge a x
i
. Entonces
lm
l
n

i=1
[x
i
k
l
x
i
[ = 0
y por tanto, llamando x = (x
1
, . . . , x
n
), se tiene que
lm
l
|x
k
l
x|
1
= 0.
Adems, puesto que x
k
l
S
(X,||
1
)
para todo l, se tiene que x S
(X,||
1
)
,
de donde se concluye que efectivamente S
(X,||
1
)
es compacto.
Corolario 1.9. Si X es un espacio normado e Y X es un
subespacio de dimensin nita, entonces Y est cerrado en X
Demostracin. Y es un espacio de dimensin nita (digamos n)
con la norma heredada de X. Por el teorema anterior esta norma es
equivalente a, por ejemplo, la norma del supremo, de manera que Y
es (isomorfo a)
n

. Sabemos que este espacio es completo (esto est


sugerido como ejercicio). La completitud no se conserva por homeo-
morsmos, pero s se mantiene por homeomorsmos uniformemente
continuos. Se ver al principio del siguiente captulo (y de hecho es un
ejercicio muy simple) que las aplicaciones lineales continuas son uni-
formemente continuas. Por tanto Y X es completo y se tiene que Y
debe ser cerrado en X.
El Teorema de Heine-Borel nos dice que los subconjuntos cerrados
y acotados de K
n
(con cualquiera de sus normas equivalentes) son com-
pactos. Para probar la falsedad de este resultado en el caso de espacios
de dimensin innita necesitamos un resultado interesante en s mismo.
Teorema 1.10 (Lema de Riesz, F. Riesz 1918). Sea X un espacio
normado y sea Y X un subespacio vectorial cerrado propio. Entonces
para todo 0 r < 1 existe x
r
S
X
tal que
r < dist(x
r
, Y ) 1.
Demostracin. Sea [z] X/Y tal que 1 > |[z]| > r. Sea ahora
un z [z] (es decir, un representante de [z]) tal que |z| 1 y sea
x
r
=
z
|z|
. Entonces se tiene que
dist(x
r
, Y ) =
dist(z, Y )
|z|
=
|[z]|
|z|
|[z]| > r.

16 1. ESPACIOS NORMADOS. ESPACIOS DE BANACH


El siguiente teorema es ahora fcil de probar
Teorema 1.11. Sea (X, | |) un espacio normado. Entonces son
equivalentes:
1. Todo conjunto cerrado y acotado de X es compacto
2. B
X
es compacto
3. X tiene dimensin nita
Demostracin. Claramente (1) implica (2). Para ver que (2) im-
plica (3), supongamos que X es de dimensin innita. Usando el Lema
de Riesz e induccin construimos una sucesin (x
n
) S
X
tal que
dist(x
n
spanx
1
, . . . , x
n1
) >
1
2
para todo n N. Entonces, para todo
n ,= m, dist(x
n
, x
m
) >
1
2
, por lo que la sucesin (x
n
) no puede tener
subsucesiones convergentes.
Veamos nalmente que (3) implica (1). Supongamos que dimX =
m, sea E X un conjunto cerrado y acotado y sea (x
n
)
n
E. Sea
e
1
, . . . , e
m
una base de X y para todo n N sea x
n
=
1
n
e
1
+
+
m
n
e
m
. Usando que (x
n
)
n
E y que E est acotado es fcil ver
que para todo 1 i m la sucesin (
i
n
)
n
K est acotada. As,
existe una subsucesin (n
l
)
l
tal que para todo 1 i m la sucesin
(
i
n
l
)
l
tiende a
i
K cuando l tiende a innito. Entonces x
n
l
tiende a
x =
1
e
1
+ +
n
e
n
(esto se puede ver estableciendo un isomorsmo
entre X y
n
1
, o
n

, que lleve los e


i
a los elementos de la base cannica).
Por ser E cerrado, se tiene que x E y con eso acabamos.
Continuamos el captulo dando una caracterizacin a menudo til
de los espacio de Banach en trminos de la convergencia de series.
En [16, Chapter 1. Notes and Remarks] aparece una gran cantidad
de informacin acerca de resultados profundos aunque elementales en
su formulacin relacionados con la sumabilidad en espacios normados,
incluso en espacios nito-dimensionales.
Dado un espacio normado y una sucesin (x
n
) X, podemos denir
la serie

n=1
x
n
como

n=1
x
n
= lm
m
m

n=1
x
n
,
al igual que hacamos con las series numricas. Si ese lmite existe
y vale x, decimos que la serie es convergente. Decimos que la serie

n
x
n
es absolutamente convergente si la serie numrica

n=1
|x
n
|
converge. Parece obvio que la convergencia absoluta debera implicar
1. ESPACIOS NORMADOS. ESPACIOS DE BANACH 17
la convergencia en norma de X. El problema podra ser la completitud.
De hecho, se tiene
Teorema 1.12. Un espacio normado X es completo si y slo si
toda serie absolutamente convergente es convergente.
Demostracin. Supongamos que X es un espacio de Banach y
sea

n
x
n
una serie absolutamente convergente. Entonces, dado > 0
existe n
0
N tal que para todos p, q n
0
q

n=p
|x
n
| < .
Por tanto, llamando s
m
=

m
n=1
x
n
, tenemos
|s
p
s
q
| =
_
_
_
_
_
q

n=p+1
x
n
_
_
_
_
_

n=p+1
|x
n
| < .
Es decir, (s
m
)
mN
es una sucesin de Cauchy y por tanto (s
m
) converge
a x X, ya que X es un espacio de Banach.
Recprocamente, supongamos que toda serie absolutamente conver-
gente es convergente, y sea (x
n
) X una sucesin de Cauchy. Veamos
que (x
n
) tiene una subsucesin convergente. Empezamos construyendo
inductivamente una subsucesin (x
n
m
)
m
(x
n
)
n
: Sea n
1
N tal que,
para todo n n
1
,
|x
n
1
x
n
| 2
1
.
Supuestos ya elegidos n
1
< n
2
< < n
m1
, elegimos n
m
de manera
que n
m
> n
m1
y tal que, para todo n n
m
,
|x
n
x
n
m
| 2
m
.
Denimos adems x
n
0
= 0 y denimos ahora la sucesin (y
i
)
iN
como
y
i
= x
n
i
x
n
i1
.
Entonces
x
n
m
=
m

i=1
y
i
y
|y
i
| = |x
n
i
x
n
i1
| 2
(i1)
.
Por tanto la serie

i
y
i
es absolutamente convergente y, por hiptesis,
convergente. Es decir

i
y
i
converge a x X. Puesto que x
n
m
=

m
i=1
y
i
, tenemos que x
n
m
converge a x y, por ser (x
n
) de Cauchy, de
aqu se sigue que x
n
converge a x, y por tanto X es completo.
18 1. ESPACIOS NORMADOS. ESPACIOS DE BANACH
La construccin de los
p
Los espacios de Banach clsicos son los
p
, los L
p
y los espacios
C(K). Vamos a ver que los
p
son espacios de Banach. Los espacios
C(K) ya estn estudiados en el ejemplo 1.2 y el estudio de los L
p
lo
haremos si los alumnos ya saben teora de la medida, o si nalmente les
enseamos la integral de Lebesgue. Para denir los espacios
p
y com-
probar que son espacios normados necesitamos primeramente estudiar
las desigualdades de Hlder y de Minkowski.
Primeramente necesitamos un lema.
Lema 1.13. Sean a 0, b 0, p, q (1, ) tales que
1
p
+
1
q
= 1.
Entonces
ab
a
p
p
+
b
q
q
.
Demostracin. Sea b > 0 y denamos la funcin
(a) =
a
p
p
+
b
q
q
ab
Es fcil ver que
t
(a) > 0 si a > b
q1
y que
t
(a) < 0 si 0 < a < b
q1
.
Por tanto alcanza su mnimo en a = b
q1
y (b
q1
) = 0. De ah se
sigue que
(a) =
a
p
p
+
b
q
q
ab 0 para todo a 0
y por lo tanto
ab
a
p
p
+
b
q
q
.

Teorema 1.14 (Desigualdad de Hlder). Sean (a


k
)
m
k=1
, (b
k
)
m
k=1

R, dos sucesiones nitas de trminos positivos, y sean p, q (1, ) dos
nmeros tales que
1
p
+
1
q
= 1. Entonces
m

k=1
a
k
b
k

_
m

k=1
a
p
k
_1
p
_
m

k=1
b
q
k
_1
q
.
Demostracin. Podemos suponer que a
k
,= 0 ,= b
k
para todo k.
Entonces, para todo 1 k m denimos
A
k
=
a
k
_

m
j=1
a
p
j
_1
p
y B
k
=
b
k
_

m
j=1
b
p
j
_1
p
1. ESPACIOS NORMADOS. ESPACIOS DE BANACH 19
y observamos que
m

k=1
A
p
k
=
m

k=1
B
p
k
= 1
Usando el Lema 1.13 tenemos que
A
k
B
k

A
p
k
p
+
B
q
k
q
y sumando esta desigualdad para todo k tenemos que
m

k=1
A
k
B
k

1
p
m

k=1
A
p
k
+
1
q
m

k=1
B
q
k
=
1
p
+
1
q
= 1,
y por tanto
m

k=1
a
k
_

m
j=1
a
p
j
_1
p

b
k
_

m
j=1
b
p
j
_1
p
1
lo que termina la demostracin.
Teorema 1.15 (Desigualdad de Minkowski). Sean dos sucesiones
(a
k
)
n
k=1
, (b
k
)
n
k=1
tales que, para todo k, a
k
0 y b
k
0, y sea 1 < p <
. Entonces
_
n

k=1
(a
k
+ b
k
)
p
_1
p

_
n

k=1
a
p
k
_1
p
+
_
n

k=1
b
p
k
_1
p
Demostracin. Si p = 1 el resultado es trivial. Sea 1 < p < y
sea q (1, ) tal que
1
p
+
1
q
= 1. Entonces, usando la desigualdad de
Hlder y el hecho de que (p 1)q = p tenemos
n

k=1
(a
k
+ b
k
)
p
=
n

k=1
(a
k
+ b
k
)
p1
(a
k
+ b
k
) =
=
n

k=1
(a
k
+ b
k
)
p1
a
k
+
n

k=1
(a
k
+ b
k
)
p1
b
k

_
n

k=1
(a
k
+ b
k
)
(p1)q
_1
q
_

a
p
k
_1
p
+
+
_
n

k=1
(a
k
+ b
k
)
(p1)q
_1
q
_

b
p
k
_1
p
=
20 1. ESPACIOS NORMADOS. ESPACIOS DE BANACH
=
_
n

k=1
(a
k
+ b
k
)
p
_1
q
_

a
p
k
_1
p
+
_
n

k=1
(a
k
+ b
k
)
p
_1
q
_

b
p
k
_1
p
Dividiendo todo por (

n
k=1
(a
k
+ b
k
)
p
)
1
q
, tenemos que
_
n

k=1
(a
k
+ b
k
)
p
_1
p
=
_
n

k=1
(a
k
+ b
k
)
p
_
1
1
q

a
p
k
_1
p
+
_

b
p
k
_1
p
como queramos comprobar.
El caso p = 1 de las desigualdades de Hlder y Minkowski se deja
como ejercicio.
Con esto ya podemos denir los espacios
p
,
n
p
y comprobar que
son espacios de Banach.
Definicin 1.16. Denimos
n

como el espacio vectorial n dimen-


sional de las n-uplas de nmeros reales o complejos (R
n
C
n
) dotado
con la norma del supremo ||

, donde para todo x = (x


1
, . . . , x
n
) R
n
C
n
, su norma del supremo se dene como:
|x|

= max
i
[x
i
[
Sobre el mismo espacio vectorial R
n
o C
n
podemos denir otras
normas. En particular tenemos
Definicin 1.17. Sea 1 p < . Denimos
n
p
como el espacio
vectorial R
n
C
n
dotado con la norma p | |
p
, denida como:
|x|
p
=
_
n

i=1
[x
i
[
p
_1
p
Observacin 1.18. Es sencillo ver que ||
p
verica las propiedades
(1) y (2) de la denicin de norma. La propiedad (3) (la desigualdad
triangular) es precisamente la desigualdad de Minkowski.
Es interesante representar las distintas bolas unidad de los espacios

2
p
para hacerse una idea de las distintas geometras de estos espacios.
En los ejercicios veremos que los espacios normados nito dimen-
sionales son completos, por lo que en particular
n
p
es un espacio de
Banach para todo 1 p y para todo n N.
Anlogamente podemos denir los espacios
p
(1 p ).
1. ESPACIOS NORMADOS. ESPACIOS DE BANACH 21
Definicin 1.19. Sea 1 p < . Denimos
p
como el espacio
vectorial de las sucesiones x := (x
i
)
iN
de nmeros (reales o complejos)
tales que

i=1
[x
i
[
p
< , dotado con la norma p denida por
|(x
i
)|
p
=
_

i=1
[x
i
[
p
_1
p
.
Observacin 1.20. Para ver que
p
est bien denido es necesario
ver que efectivamente es un espacio vectorial y que | | :
p
[0, )
es efectivamente una norma. Veamslo. La nica dicultad reside en
comprobar que, si (x
i
), (y
i
)
p
, entonces (x
i
) + (y
i
)
p
y |(x
i
) +
(y
i
)|
p
|(x
i
)|
p
+|(y
i
)|
p
:
Para todo n m, usando la desigualdad de Minkowski, se tiene
_
n

i=1
[x
i
+ y
i
[
p
_1
p

_
n

i=1
[x
i
[
p
_1
p
+
_
n

i=1
[y
i
[
p
_1
p

_
m

i=1
[x
i
[
p
_1
p
+
_
m

i=1
[y
i
[
p
_1
p
As pues, dejando tender m a innito se tiene que
_
n

i=1
[x
i
+ y
i
[
p
_1
p
|(x
i
)|
p
+|(y
i
)|
p
para todo n N, de forma que, al dejar que n tienda a innito tenemos
lo buscado.
Definicin 1.21. Denimos

como el espacio vectorial de las


sucesiones acotadas x := (x
i
)
iN
de nmeros (reales o complejos), dota-
do con la norma del supremo | |

denida por
|(x
i
)|

= sup
iN
[x
i
[.
Se ve fcilmente que

est bien denido, es decir, que es un espacio


vectorial y que | |

es en efecto una norma.


El espacio

es muy grande y tienen gran importancia sus sigu-


ientes subespacios cerrados.
Definicin 1.22. Denimos c como el espacio vectorial de las suce-
siones convergentes x := (x
i
)
iN
de nmeros (reales o complejos), dota-
do con la norma del supremo | |

, y denimos c
0
como el espacio
vectorial de las sucesiones x := (x
i
)
iN
convergentes a cero de nmeros
(reales o complejos), dotado de nuevo con la norma del supremo.
22 1. ESPACIOS NORMADOS. ESPACIOS DE BANACH
Todos los espacios denidos anteriormente son espacios de Banach.
Ese es el contenido de la siguiente proposicin.
Proposicin 1.23. Sea 1 p . Entonces
p
es un espacio de
Banach. Adems c y c
0
tambin son espacios de Banach.
Demostracin. Veamos primero el caso 1 p < . Consider-
emos (x
n
)
nN

p
una sucesin de Cauchy, donde x
n
= (x
n
i
)
i
. Dado
> 0 existe n
0
N tal que para todo n, m n
0
|x
n
x
m
| < , es decir
_

i=1
[x
n
i
x
m
i
[
p
_1
p
< .
En particular, para todo i N, se tiene que [x
n
i
x
m
i
[ < , y por
tanto (x
n
i
)
n
K es una sucesin de Cauchy. Usando la completitud
de K tenemos que para todo i N existe x
i
K tal que x
n
i
x
i
cuando n tiende a . Con esto ya tenemos el candidato a lmite, que
es x = (x
i
)
iN
. Veamos que x
p
y que x
n
tiende a x en
p
. Para
lo primero, por ser (x
n
)
n
una sucesin de Cauchy, sabemos que est
acotada, y por tanto existe C > 0 tal que, para todo n N, |x
n
|
p
p
C
es decir

i=1
[x
n
i
[
p
C.
Entonces, para todos j, n N se tiene que
j

i=1
[x
n
i
[
p
C.
Tomando lmites cuando n tiende a innito, se tiene que
j

i=1
[x
i
[
p
C,
y dejando ahora que j tienda a innito tenemos que

i=1
[x
i
[
p
C
y por tanto x
p
.
Veamos nalmente que x
n
tiende a x en | |
p
. Sea > 0. Sea n
0
N
tal que, para todo n, m n
0
,
|x
n
x
m
|
p
< .
Entonces, para todo j N tenemos que
1. ESPACIOS NORMADOS. ESPACIOS DE BANACH 23
_
j

i=1
[x
n
i
x
m
i
[
p
_
1
p
< .
Fijando j N, eligiendo n n
0
y dejando que m tienda a innito
obtenemos que
_
j

i=1
[x
n
i
x
i
[
p
_
1
p
< para todo j N, n n
0
.
Dejando ahora que j tienda a innito, obtenemos que
_

i=1
[x
n
i
x
i
[
p
_1
p
< para todo n n
0
y por tanto x
n
tiende a x en
p
.
Para ver que

es Banach, basta notar que

= B(N), el conjunto
de las funciones acotadas
f : N K.
Ya hemos visto en el Ejemplo 1.2 que ese espacio es completo. Tambin,
si los alumnos conocen suciente topologa, se puede ver que

=
C(N), las funciones continuas sobre la compacticacin de Stone-

Cech
de N con la mtrica usual. Tambin en el Ejemplo 1.2 vimos que este
espacio es un espacio de Banach.
Veamos nalmente que c y c
0
son espacios de Banach. Puesto que
ambos son subespacios de

y este es completo, slo tenemos que ver


que ambos son cerrados en

. Lo vemos para c, el caso de c


0
es algo
ms sencillo y se deja como ejercicio.
Sea (x
n
)
nN
c una sucesin tal que x
n
tiende a x

. Hemos
de ver que x c. Para todo n N sea l
n
= lm
i
x
n
i
. Veamos que
(l
n
)
n
es una sucesin de Cauchy. Sea > 0. Existe n
0
N tal que, para
todos n, m n
0
se tiene
|x
n
x
m
|

< .
Entonces para todo i N
[x
n
i
x
m
i
[ < .
Fijando n, m n
0
y dejando que i tienda a innito obtenemos que
[l
n
l
m
[ < .
24 1. ESPACIOS NORMADOS. ESPACIOS DE BANACH
Por tanto (l
n
)
n
es una sucesin de Cauchy y existe l K tal que (l
n
)
tiende a l. Veamos que lm
i
x
i
= l: sea > 0. Existe n
0
N tal que
para todo n n
0
|x
n
x| < y [l
n
l[ < .
Adems existe j N tal que para todo i j
[x
n
0
i
l
n
0
[ < .
Entonces, para todo i j
[x
i
l[ [x
i
x
n
0
i
[ +[x
n
0
i
l
n
0
[ +[l
n
0
l[ 3
y por tanto x
i
tiende a l y x c.
La construccin de los L
p
[0, 1]
Puesto que es posible que nuestros alumnos no conozcan la inte-
gral de Lebesgue, incluimos en esta memoria una breve descripcin sin
demostraciones de los resultados bsicos de la Teora de la Integral de
Lebesgue. Creemos que debera ser posible explicar en una o dos ho-
ras los resultados de esta Teora necesarios para poder construir los
espacios L
p
[0, 1].
Comenzamos con una denicin
Definicin 1.24. Sea un conjunto y sea T T() una coleccin
de subconjuntos de . Decimos que T es una -lgebra si
1. T.
2. Si A T entonces tambin su complementario A
c
T.
3. Si (A
n
)
nN
T entonces

nN
A
n
T.
Se sigue de la denicin que si T es una -lgebra y A, B T
entonces A B T, y si (A
n
)

n=1
T entonces tambin

n=1
A
n
T.
Se puede demostrar que dada T T() una familia de subconjun-
tos de , existe una -lgebra mnima (en el sentido de la inclusin) que
contiene a T. A esta -lgebra la denominamos la -lgebra generada
por T, y la escribimos (T).
No pretendemos desarrollar una Teora de la Medida abstracta, sino
simplemente denir y justicar parcialmente los resultados principales
de la Teora de la Medida de Lebesgue. Por lo tanto, de ahora en
adelante consideraremos = [0, 1] (todo es totalmente anlogo para
cualquier otro intervalo cerrado y acotado [a, b] R) y la -lgebra que
1. ESPACIOS NORMADOS. ESPACIOS DE BANACH 25
consideraremos ser B, la -lgebra generada por los conjuntos abiertos
de [0, 1]. A B se le denomina habitualmente la -lgebra de Borel.
Un par (, T), formado por un espacio y una -lgebra denida
sobre l se denomina un espacio de medida. Nuestro espacio de medida
ser siempre ([0, 1], B).
Definicin 1.25. Dado ([0, 1], B), un conjunto A [0, 1] se dice
medible si A B.
En Teora de la Medida es conveniente a menudo denir funciones
con valores en la recta real ampliada R = R + en
[0, ]. En el caso complejo, diremos que una funcin toma valores en
C si f es de la forma f = g + h, y tanto g como h toman valores en
R. Finalmente, si queremos denotar una funcin que toma valores en
R o C, segn el cuerpo sobre el que estemos trabajando, diremos que
f toma valores en K. Esta notacin no es estndar, pero creemos que
simplica algo la escritura de varios de los resultados siguientes.
Definicin 1.26. Una funcin f : [0, 1] R se dice medible
si para todo conjunto abierto A R y para todo a R se tiene que
f
1
(A), f
1
([, a)) y f
1
((a, +]) son medibles.
Definicin 1.27. Una funcin f : [0, 1] C se dice medible si
f = g + h, con f, g : [0, 1] R funciones medibles.
Se tiene la siguiente proposicin que no demostraremos.
Proposicin 1.28. Sea (f
n
)
nN
una sucesin de funciones f
n
:
[0, 1] R medibles. Entonces
1. g = sup
n
f
n
y h = lmsup
n
f
n
son medibles.
2. Si lm
n
f
n
(t) = f(t) para todo t [0, 1] entonces f es medible.
3. Si f, g : [0, 1] R son medibles entonces las funciones
maxf, g y mnf, g son medibles. En particular max0, f
y mn0, f son medibles.
Definicin 1.29. Una funcin f : [0, 1] K se dice simple si
existe n N, escalares a
1
, . . . , a
n
y conjuntos A
1
, . . . , A
n
B tales
que
f =
n

i=1
a
i

A
i
Claramente, toda funcin simple es medible.
Del siguiente resultado esbozamos la demostracin.
26 1. ESPACIOS NORMADOS. ESPACIOS DE BANACH
Teorema 1.30. Sea f : [0, 1] R una funcin medible. Entonces
existe una sucesin de funciones simples (s
n
)
n
tal que
1. 0 s
1
s
2

2. Para todo t [0, 1]
lm
n
s
n
(t) = f(t)
Demostracin. Para todo n N y para todo 1 i n2
n
sea
E
n,i
= f
1
__
i 1
2
n
,
i
2
n
__
y
F
n
= f
1
([n, ])
y sea
s
n
=
n2
n

i=1
i 1
2
n

E
n,i
+ n
F
n
.

Si bien slo vamos a utilizar la medida de Lebesgue, damos la deni-


cin general de medida (positiva contablemente aditiva)
Definicin 1.31. Dado un espacio de medida (, T), una medida
sobre dicho espacio es una funcin
m : T [0, ]
tal que
1. m() = 0
2. Para toda sucesin de conjuntos medibles (A
n
)
nN
dos a dos
disjuntos (esto es, A
n
A
m
= para todo m ,= n) se tiene
m
_

_
n=1
A
n
_
=

n=1
m(A
n
).
No es obvia la existencia de medidas no triviales. A continuacin
damos como teorema (sin demostracin) la existencia de la medida de
Lebesgue.
Teorema 1.32. En el espacio de medida ([0, 1], B) existe una nica
medida
: B [0, 1]
tal que para todo 0 a b 1
((a, b)) = ([a, b]) = b a.
A esta la denominamos medida de Lebesgue.
1. ESPACIOS NORMADOS. ESPACIOS DE BANACH 27
Ntese que para todo medida positiva m se tiene que si A, B son dos
conjuntos medibles con A B entonces m(A) m(B). La siguiente
proposicin (que tampoco probamos) nos dice que podemos aproximar
(en el sentido de la medida de Lebesgue) cualquier conjunto medible
desde dentro por compactos y desde fuera por abiertos.
Proposicin 1.33. La medida de Lebesgue es regular en el sigu-
iente sentido: para todo A [0, 1] y para todo > 0 existe un conjunto
cerrado (equivalentemente compacto) F A y un conjunto abierto
G A tales que
(G) (A) (F) + ,
equivalentemente
(A F) y (G A)
Necesitamos ahora denir integracin respecto de la medida de
Lebesgue. Comenzamos deniendo la integral de las funciones simples
de la nica forma razonable y la vamos extendiendo a las funciones
medibles ganando generalidad. No es difcil ver que cada una de las
deniciones va siendo coherente con las anteriores.
Definicin 1.34. Si s =

n
i=1
a
i

A
i
: [0, 1] R es una funcin
simple y E B es un conjunto medible, se dene la integral de s en E
respecto de como
_
E
sd =
n

i=1
a
i
(A
i
E).
Definicin 1.35. Si f : [0, 1] [0, ] es una funcin medible y
positiva, y E B es un conjunto medible, se dene la integral de f en
E respecto de como
_
E
fd = sup
s
_
E
sd,
donde el supremo se toma sobre todas las funciones simples s tales que
s f.
Ntese que la integral as denida puede valer .
Definicin 1.36. Si f : [0, 1] R es una funcin medible, se
denen f
+
y f

como
f
+
= maxf, 0 y f

= mnf, 0.
De la Proposicin 1.28 se sigue que tanto f
+
como f

son medibles
(y claramente positivas). Entonces, dado un conjunto medible E B
consideramos las dos integrales
28 1. ESPACIOS NORMADOS. ESPACIOS DE BANACH
_
E
f
+
d y
_
E
f

d.
Si al menos una de esas dos integrales es nita, se dene la integral de
f en E respecto de como
_
E
fd =
_
E
f
+
d
_
E
f

d.
Finalmente,
Definicin 1.37. Si f : [0, 1] C es una funcin compleja med-
ible y f = g + h, con g, h : [0, 1] R medibles, dado un conjunto
medible E B se dene la integral de f en E respecto de como
_
E
fd =
_
E
gd +
_
E
hd
si las integrales involucradas en la denicin existen.
Observacin 1.38. Se demuestra que si f es integrable Riemann
en un intervalo [a, b], entonces la integral de Riemann de f en [a, b] y
la integral de Lebesgue de f en [a, b] coinciden. Se demuestra tambin
que existen funciones no integrables Riemann que s son integrables
Lebesgue.
Resumimos en las dos siguientes proposiciones las propiedades bsi-
cas de la integral de Lebesgue que necesitaremos ms adelante, sin
demostracin. Necesitamos una denicin previa.
Definicin 1.39. Sea f : [0, 1] R una funcin medible. Deci-
mos que f es integrable si las dos integrales que aparecen en su deni-
cin son nitas. Anlogamente, si f : [0, 1] C es una funcin
medible, decimos que f es integrable si las dos integrales que aparecen
en su denicin son nitas.
Proposicin 1.40. Sea f, g : [0, 1] R y E B. Entonces
1. Si f es medible y acotada en E entonces f es integrable.
2. Si a f(x) b para todo x E entonces
a(E)
_
E
fd b(E).
3. Si f, g son integrables en E y f(x) g(x) para todo x E
entonces
_
E
fd
_
E
gd.
Proposicin 1.41. Sea f, g : [0, 1] K y E B. Entonces
1. ESPACIOS NORMADOS. ESPACIOS DE BANACH 29
1. Si f, g son integrables, entonces para todo , K
_
E
(f + g)d =
_
E
fd +
_
E
gd.
(Si bien es razonablemente inmediato que los escalares pueden
salir fuera del signo integral, para demostrar la aditividad de
la integral se suele usar el Teorema de la Convergencia Mon-
tona, que veremos ms adelante, o algn resultado equiva-
lente.)
2. Si f es medible y (E) = 0 entonces
_
E
fd = 0.
3. Si f es integrable en E y A B verica A E entonces f es
integrable en A.
4.

_
E
fd

_
E
[f[d.
De los resultados anteriores se deduce que los conjuntos de medida
nula son despreciables en la integracin:
Proposicin 1.42. Sean f : [0, 1] K y A, E B. Si A E y
(E A) = 0 entonces
_
E
fd =
_
A
fd
Este ltimo resultado hace que, si lo que nos interesa de las fun-
ciones sea su integral, parezca que tenga sentido considerar la siguiente
relacin de equivalencia. Dadas dos funciones medibles
f, g : [0, 1] K
decimos que f g si
(x [0, 1] tales que f(x) ,= g(x)) = 0.
Es claro que es una relacin de equivalencia y que si f g entonces
para todo E B
_
E
fd =
_
E
gd.
La siguiente notacin til est relacionada con lo anterior: Si una
propiedad P ocurre para todo x [0, 1] A siendo A un conjunto de
medida nula, decimos que P ocurre en casi todo punto, abreviado c.t.p.
En particular, f g si y slo si f(x) = g(x) c.t.p.
30 1. ESPACIOS NORMADOS. ESPACIOS DE BANACH
Los dos resultados que enunciamos a continuacin, sin demostracin,
son fundamentales en la teora de integracin de Lebesgue, y nos per-
mitirn intercambiar lmites e integrales en numerosas ocasiones.
Teorema 1.43 (de la Convergencia Montona). Sea E B. Sea
(f
n
) una sucesin de funciones medibles f
n
: [0, 1] [0, ] tales que
para todo x E
0 f
1
(x) f
2
(x)
Sea f : [0, 1] [0, ] la funcin denida como
f(x) = lm
n
f
n
(x) para todo x [0, 1].
Entonces
lm
n
_
E
f
n
d =
_
E
fd.
Teorema 1.44 (de la Convergencia Dominada). Sea E B. Sea
(f
n
) una sucesin de funciones medibles f
n
: [0, 1] [0, ] y sea
f : [0, 1] [0, ] tales que
f(x) = lm
n
f
n
(x) para todo x [0, 1].
(Ntese que f es medible como consecuencia de la Proposicin 1.28).
Si existe una funcin g : [0, 1] [0, ] integrable en E y tal que para
todo n N y para todo x [0, 1]
[f
n
(x)[ g(x)
entonces
lm
n
_
E
f
n
d =
_
E
fd.
Una vez que tenemos a nuestra disposicin los resultados bsicos de
la Teora de integracin de Lebesgue, podemos iniciar la construccin
de los espacios L
p
[0, 1].
Empezamos proponiendo como ejercicio la demostracin de las de-
sigualdades de Hlder y Minkowski en la versin integral.
Ejercicio 1.1 (Desigualdad de Hlder). Sean 1 < p, q < con
1
p
+
1
q
= 1, sean f, g : [0, 1] [0, ] medibles y sea E B. Entonces
_
E
fgd
__
E
f
p
d
_1
p
__
E
g
q
d
_1
q
.
1. ESPACIOS NORMADOS. ESPACIOS DE BANACH 31
Ejercicio 1.2 (Desigualdad de Minkowski). Sean 1 < p, q <
con
1
p
+
1
q
= 1, sean f, g : [0, 1] [0, ] medibles y sea E B.
Entonces
__
E
(f + g)
p
d
_1
p

__
E
f
p
d
_1
p
+
__
E
g
p
d
_1
p
.
Denimos ahora las normas p para las funciones medibles.
Definicin 1.45. Sea f : [0, 1] K una funcin medible y sea
1 p < . Se dene la norma p de f como
|f|
p
=
__
1
0
[f[
p
d
_
1
p
.
Veamos ahora el caso p = .
Definicin 1.46. Sea f : [0, 1] [0, ] una funcin medible.
Sea
S = R tales que (f
1
((, ])) = 0
Si S = hacemos = . Si S ,= , decimos que S es una cota
esencial de S y hacemos =nf S. Llamamos a el supremo esencial
de f.
Ahora, dada una funcin medible f : [0, 1] K, llamamos al
supremo esencial de [f[ y denimos la norma de f como
|f|

= .
Puesto que
f
1
((, ]) =

n=1
f
1
(( +
1
n
, ])
y dado que la unin numerable de conjuntos de medida 0 tiene medida
0, se sigue que el supremo esencial es una cota esencial y por lo tanto
f(x) |f|

c.t.p.
Ya podemos denir los espacios L
p
.
Definicin 1.47. Sea 1 p . Se dene el espacio vectorial
L
p
[0, 1] como
L
p
[0, 1] = f : [0, 1] K medibles tales que |f|
p
< .
Quisiramos ver ahora que el espacio L
p
[0, 1] con la norma p es
un espacio de Banach. Lamentablemente la norma p no es una norma
en L
p
[0, 1]. Lo nico que falla es que f puede ser distinta de 0 (en el
32 1. ESPACIOS NORMADOS. ESPACIOS DE BANACH
sentido que esperaramos de ser distinto de 0) mientras que |f|
p
= 0,
tmese por ejemplo
f(x) =
_
1 si x = 0
0 si x ,= 0.
La solucin es denir los espacios L
p
[0, 1] como el cociente de L
p
[0, 1]
por la relacin de equivalencia denida anteriormente.
Con esta denicin se tiene
Ejercicio 1.3. Sea 1 p . El espacio L
p
[0, 1] con la norma
| |
p
es un espacio normado.
Es algo ms complicado ver que L
p
[0, 1] es un espacio de Banach.
Siguiendo la costumbre habitual, al escribir no somos totalmente rig-
urosos en la distincin formal entre f L
p
[0, 1] y su clase [f] L
p
[0, 1].
Creemos que no hay confusin posible y que ser totalmente formalistas
confunde ms que aclara.
Teorema 1.48. Sea 1 p . El espacio L
p
[0, 1] con la norma
| |
p
es un espacio de Banach.
Demostracin. Consideramos primero el caso 1 p < . Sea
(f
n
)
n
L
p
[0, 1] una sucesin de Cauchy. Queremos ver que (f
n
) con-
verge a una funcin f L
p
[0, 1], y por ser (f
n
) una sucesin de Cauchy,
est claro que basta probar que (f
n
) tiene una subsucesin convergente.
As, tomando una subsucesin podemos suponer sin perdida de gener-
alidad que
|f
n+1
f
n
|
p

1
2
n
.
Sea f
0
= 0 y para todos n N, t [0, 1] sea
g
n
(t) =
n

j=0
[f
j+1
(t) f
j
(t)[
y
g(t) =

j=0
[f
j+1
(t) f
j
(t)[
Entonces, por la desigualdad triangular se tiene
|g
n
|
p

n

j=0
|f
j+1
f
j
|
p
|f
1
f
0
|
p
+
n

j=1
1
2
j
|f
1
|
p
+ 1.
Como adems la sucesin (g
n
) es creciente y converge a g del Teo-
rema de la Convergencia Montona se tiene que
|g|
p
= lm
n
|g
n
|
p
|f
1
|
p
+ 1
1. ESPACIOS NORMADOS. ESPACIOS DE BANACH 33
y por tanto g L
p
[0, 1], y en particular g es nita c.t.p.
Es decir, la serie

j=0
f
j+1
(t) f
j
(t) converge absolutamente c.t.p.
y por tanto converge c.t.p.
Para todo t [0, 1] sea entonces
f(t) =

j=0
f
j+1
(t) f
j
(t).
Como
f
n
(t) =
n1

j=0
f
j+1
(t) f
j
(t),
se tiene que
lm
n
f
n
(t) = f(t)
y
[f
n
(t)[
n1

j=0
[f
j+1
(t) f
j
(t)[ g(t) para todo t [0, 1].
Ahora el Teorema de la Convergencia Dominada nos dice que
_
1
0
[f[
p
d = lm
n
_
1
0
[f
n
[
p
d
_
1
0
g
p
d <
y por tanto f L
p
[0, 1].
En ese caso tambin [f[ + g L
p
[0, 1] y [f f
n
[
p
([f[ + g)
p
para todo n N. Por tanto, de nuevo el Teorema de la Convergencia
Dominada nos dice que
|f f
n
|
p
p
=
_
1
0
[f
n
f[
p
d 0
y por tanto f
n
f en | |
p
, lo que termina la demostracin en el caso
1 p < .
Si ahora p = , sea (f
n
) L

[0, 1] una sucesin de Cauchy. Para


todos n, m N sea
A
n
= t [0, 1] tales que [f
n
(t)[ |f|

y sea
B
n,m
= t [0, 1] tales que [f
m
(t) f
n
(t)[ |f
m
f
n
|

.
Si
F =
_

_
n=1
A
n

_
n,m=1
B
n,m
_
34 1. ESPACIOS NORMADOS. ESPACIOS DE BANACH
entonces m(F) = 0 y est claro que (f
n
) tiende uniformemente en F
c
a
una funcin f acotada. Por lo tanto f L

[0, 1] y |f
n
f|

0.
En muchas ocasiones la forma ms cmoda de estudiar un espacio
L
p
[0, 1] es considerarlo como el completado de C[0, 1] con la norma
p. Contamos eso a continuacin sin detallar las demostraciones, puesto
que eso nos llevara a internarnos en la Teora de la Integral de Lebesgue
ms de lo que consideramos adecuado para este curso.
Comenzamos enunciando sin demostracin el Teorema de Lusin. Al
presentarlo en clase podemos hacer un esquema de la demostracin,
utilizando la regularidad de y el Lema de Urysohn, aunque sin com-
pletar los detalles.
Teorema 1.49 (Lusin). Sea f : [0, 1] K una funcin integrable,
sea E B con f(x) = 0 si x , E y sea > 0. Entonces existe
g C[0, 1] tal que
(x; f(x) ,= g(x)) <
y
|g|

|f|

.
Necesitamos otro resultado, interesante en s mismo.
Proposicin 1.50. Sea S el conjunto de las funciones s : [0, 1]
K simples y sea 1 p < . Entonces S es denso en L
p
[0, 1].
Demostracin. Vemos primero el caso real. Sea f L
p
[0, 1], f
0, y sea (s
n
) S una sucesin tal que 0 s
1
s
2
y
lm
n
s
n
(x) = f(x) c.t.p.
Puesto que [f s
n
[
p
[f[
p
, del Teorema de la Convergencia Dominada
se sigue que
|f s
n
|
p
0.
Una vez probado el caso f 0 se puede probar el caso de f con valores
en R descomponiendo f = f
+
f

, y a continuacin se prueba el caso


de f con valores en C descomponiendo f = g + h.
Finalmente se tiene
Teorema 1.51. Sea 1 p < . Entonces (C[0, 1], | |
p
) es denso
en L
p
[0, 1].
Demostracin. La demostracin se sigue del resultado anterior
ms el Teorema de Lusin.
1. ESPACIOS NORMADOS. ESPACIOS DE BANACH 35
Finalmente podemos comentar que se puede demostrar que C[0, 1]
es separable y que L

[0, 1] no lo es, por lo que (C[0, 1], | |

) no puede
ser denso en L

[0, 1].
Prcticas sugeridas.
Ejercicio 1.4. Comprobar que todo espacio normado de dimensin
nita es completo.
Ejercicio 1.5. Sea (X, | |) un espacio normado. Entonces | | :
X [0, +) es una funcin continua.
Ejercicio 1.6. Para todo n N sea X
n
un espacio de Banach.
Probar que, para todo 1 p < , el espacio

p
(X
n
) = (x
n
)
nN
tales que x
n
X
n
para todo n N y

n=1
|x
n
|
p
<
es un espacio de Banach con la norma
|(x
n
)
nN
| =
_

n=1
|x
n
|
p
_1
p
.
Anlogamente probar que c(X
n
), c
0
(X
n
) y

(X
n
) con las deniciones
obvias tambin son espacios de Banach.
Ejercicio 1.7. En muchas ocasiones (en particular en la construc-
cin de los duales de estos espacios) necesitaremos una aproximacin
por un nmero nito de coordenadas a los elementos de
p
, c y c
0
.
Para esto resulta muy adecuado denir el espacio
c
00
= (x
n
)
nN
K
N
tales que existe n
0
N de manera que
para todo n n
0
x
n
= 0,
es decir el espacio de las sucesiones de escalares con un nmero nito de
trminos no nulos. Comprobar que, para todo 1 p , (c
00
, | |
p
)
es un espacio normado pero no es un espacio de Banach. Para todo
1 p < comprobar que c
00

p
y que c
00
es denso en
p
. Comprobar
tambin que c
00
c
0
y que c
00
es denso en c
0
.
Ejercicio 1.8. Aplicar el Teorema 1.12 para demostrar el recproco
de la Proposicin 1.4: Si X es un espacio normado, Y X es un
subespacio vectorial cerrado y X/Y es un espacio de Banach, entonces
X es un espacio de Banach.
36 1. ESPACIOS NORMADOS. ESPACIOS DE BANACH
Ejercicio 1.9. Bases de Schauder. [31, p. 133]
Ejercicio 1.10. Separabilidad. Denicin y propiedades bsicas.
Comprobar que
p
es separable si y slo si 1 p < , y que c
0
es
separable. Demostrar que C[0, 1] es separable.
Ejercicio 1.11. Probar que si 1 p L
p
[0, 1] es un espacio
de Banach.
Ejercicio 1.12. Dar condiciones necesarias y sucientes para que
se tenga la igualdad en las desigualdades de Hlder y Minkowski.
Ejercicio 1.13. Demostrar que si 1 q < p entonces la
inclusin
q

p
es propia.
Ejercicio 1.14. Encontrar una sucesin que pertenezca a c
0
pero
no a
p
para ningn p [1, ).
Ejercicio 1.15. Hacer un boceto de las bolas unidad en
2
p
con
p = 1,
3
2
, 2, 4, .
Ejercicio 1.16. Demostrar que si X tiene dimensin nita e Y
X es un subespacio vectorial propio entonces existe x S
X
tal que
d(x, Y ) = 1.
Ejercicio 1.17. Sea C
(n)
[0, 1] el conjunto de las funciones contin-
uas con derivada n-sima continua, con la norma
|f| = max
0kn
|f|

.
Demostrar que C
(n)
[0, 1] con esa norma es un espacio de Banach.
CAPTULO 2
Aplicaciones lineales continuas entre espacios
normados
Como ya hemos dicho, el objeto natural de trabajo del Anlisis Fun-
cional van a ser espacios vectoriales, en general de dimensin innita,
dotados de algn tipo de topologa vectorial (en este curso slo con-
sideramos espacios normados). Resulta pues natural que en el estudio
de estos espacios sea de gran importancia el estudio de las aplicaciones
lineales y continuas entre ellos, dado que estas son las aplicaciones que
preservan las dos estructuras (vectorial y topolgica) con las que es-
tamos trabajando. De hecho muchas de las aplicaciones del Anlisis
Funcional consisten precisamente en estudiar ciertas aplicaciones lin-
eales y continuas destacadas entre espacios normados.
Tras ver las caracterizaciones ms utilizadas de la continuidad de
una aplicacin lineal T : X Y , estudiamos la situacin cuando X
tiene dimensin nita y vemos que si X tiene dimensin innita siem-
pre existen aplicaciones lineales no continuas. Denimos seguidamente
cundo dos espacios son isomorfos o isomtricos.
Es fcil ver que L(X; Y ), el espacio de las operadores de X a Y
tiene estructura de espacio vectorial. Lo interesante es que tambin se
puede denir una norma que le convierte en espacio normado, completo
si Y lo es. En particular se sigue que el dual de todo espacio normado
es un espacio de Banach.
Para terminar, incluimos en este captulo un breve estudio de la
relacin entre formas lineales e hiperplanos, necesario para estudiar el
Teorema de Hahn-Banach en el siguiente captulo.
En la preparacin de este captulo hemos seguido principalmente
[31], [24] y [13].
Aplicaciones lineales continuas
Empezamos con el siguiente teorema
Teorema 2.1. Sean X, Y dos espacios normados, y sea T : X
Y una aplicacin lineal. Entonces, son equivalentes:
37
38 2. APLICACIONES LINEALES CONTINUAS
(i) T es continuo.
(ii) T es continuo en el origen.
(iii) Existe C > 0 tal que |T(x)| C|x| para todo x X
(iv) T es Lipschitziana, es decir, existe C > 0 tal que
|T(x) T(y)| C|x y|
para todos x, y X.
(v) T(B
X
) est acotado en Y .
(vi) ker(T) es un subespacio cerrado y la aplicacin lineal T :
X/ ker(T) Y denida como
T([x]) = T(x)
es continua.
Demostracin. Claramente (i) implica (ii).
Para ver que (ii) implica (iii), si T es continuo en el origen, entonces,
dado > 0 existe > 0 tal que, siempre que |x| se tiene
|T(x)| .
Entonces, para todo 0 ,= x X se tiene que
|T(x)| =
_
_
_
_
T
_
|x|

x
|x|
__
_
_
_
=
|x|

_
_
_
_
T
_
x
|x|
__
_
_
_
.
Como
_
_
_
_
x
|x|
_
_
_
_
=
entonces
|T(x)|
|x|

y C =

verica (iii).
Que (iii) implica (iv) se sigue de la linealidad de T, puesto que
|T(x) T(y)| = |T(x y)| C|x y|
Claramente (iv) implica (i).
Si (iii) es cierto, entonces, para todo x B
X
, |T(x)| C, luego
T(B
X
) est acotado y se tiene (v).
2. APLICACIONES LINEALES CONTINUAS 39
Recprocamente, si tenemos (v), y, por ejemplo |T(x)| C para
todo x B
X
, entonces para todo 0 ,= x X se tiene
|T(x)| = |x|
_
_
_
_
T
_
x
|x|
__
_
_
_
C|x|
lo que implica (iii)
Veamos que (i) implica (vi). Claramente, si T es continuo entonces
ker(T) es un subespacio cerrado. Por tanto ya vimos que X/ ker(T) es
un espacio normado con la norma inducida. Es conocido (y fcil de ver)
que la aplicacin T est bien denida y es lineal. Adems
|T([x])| = |T(x)| C|x|
si T es continuo.
Recprocamente, si se tiene (vi), existe C tal que
|T([x])| C|[x]|
y por tanto, para todo x X,
|T(x)| = |T([x])| |[x]| C|x|
ya que
|[x]| = nf
zker(T)
|x + z| |x|
por tanto (iii), y de ah (i), son ciertos.
Veamos como corolario (aunque es fcil probarlo directamente) que
en espacios de dimensin nita todo operador lineal es continuo.
Corolario 2.2. Sean X, Y dos espacios normados con dimX <
y sea T : X Y una aplicacin lineal. Entonces T es continuo.
Demostracin. Puesto que ya hemos visto anteriormente que en
los espacios de dimensin nita todas las normas son equivalentes,
suponemos sin perdida de generalidad X =
n
1
. Sea (e
i
)
n
i=1
la base
cannica de
n
1
y sea
M = sup
i
|T(e
i
)|.
Entonces, para todo x = (x
1
, . . . , x
n
) B

1
,
|T(x)| =
_
_
_
_
_
T
_

i
x
i
e
i
__
_
_
_
_
=
_
_
_
_
_

i
x
i
T(e
i
)
_
_
_
_
_

i
[x
i
[|T(e
i
)|
M

i
[x
i
[ M
40 2. APLICACIONES LINEALES CONTINUAS
y el teorema anterior nos garantiza que T es continuo.
Veamos que de hecho esa condicin caracteriza a los espacios de
dimensin nita.
Teorema 2.3. Sea X un espacio normado de dimensin innita
e Y un espacio normado no nulo. Entonces existe una aplicacin T :
X Y lineal no continua.
Demostracin. Sea (x
n
) B
X
una familia de vectores lineal-
mente independientes, consideremos una base B de X que contenga a
la sucesin (x
n
) y un vector 0 ,= y Y . Denamos ahora la aplicacin
lineal T : X Y en todos los elementos de la base B mediante
T(b) =
_
ny , si b = x
n
0 , si b ,= x
n
para todo n
y la extendemos por linealidad a todo X. Entonces claramente T(B
X
)
no est acotado, por lo que T no es continuo.
Desde el punto de vista de las categoras, siempre es importante
establecer alguna condicin de equivalencia entre los objetos de la
categora. En la categora de espacios de Banach, y de espacios nor-
mados, hay dos nociones que pueden jugar este papel, segn en que
aspectos de la teora deseemos jarnos.
Definicin 2.4. Dos espacios normados X e Y se dicen isomorfos
si existe una aplicacin lineal biyectiva y continua : X Y tal
que su inversa
1
(que sabemos por lgebra Lineal que es lineal) es
continua.
Veremos que del Teorema de la Aplicacin Abierta se sigue que si
es como en la denicin entonces
1
siempre verica lo pedido.
Observemos que si X e Y son isomorfos, entonces X es Banach si
y slo si Y lo es, ya que los homeomorsmos lineales ya vimos que son
uniformemente continuos.
Definicin 2.5. Dos espacios normados X e Y se dicen isomtri-
cos si existe una aplicacin lineal y biyectiva : X Y tal que, para
todo x X,
|(x)| = |x|.
Los isomorsmos conservan la estructura lineal y topolgica de los
espacios. Las isometras conservan adems la geometra de los espacios.
2. APLICACIONES LINEALES CONTINUAS 41
Se distingue entonces a menudo entre la Teora Isomtrica, ms preocu-
pada de problemas de Geometra de Espacios de Banach, y la Teora
Isomrca, ms preocupada de problemas que se suelen denominar es-
tructurales.
A continuacin damos algunos ejemplos de aplicaciones lineales y
continuas (operadores de ahora en adelante) o no continuas entre es-
pacios normados.
Ejemplo 2.6. Sea A = (a
i,j
)
i,jN
una matriz innita y sea 1 p <
.
Si p = 1, supongamos que (j) =

i=1
[a
i,j
[ < para todo j N
y que (j) 0 cuando j .
Si 1 < p < y
1
p
+
1
q
= 1, supongamos que

p
=
_
_

i=1
_

j=1
[a
i,j
[
q
_
p
q
_
_
1
p
< .
Entonces el operador
T
A
:
p

p
dado por
T
A
(x) =
_

j=1
a
i,j
x
j
_
j
(est bien denido y) es continuo.
Comencemos viendo el caso p = 1. Notemos que si (j) 0 en
particular se tiene que sup
j
(j) = C < . Entonces
|T
A
(x)|
1
=
_
_
_
_
_
_
_

j=1
a
i,j
x
j
_
i
_
_
_
_
_
_
1
=

i=1

j=1
a
i,j
x
j

j
[a
i,j
x
j
[ =
=

i
[a
i,j
[[x
j
[

j
[x
j
[

i
[a
i,j
[ =

j
[x
j
[(j) C|x|
1
,
por lo que T
A
es continuo.
Veamos ahora el caso 1 < p < . Ntese el uso de la desigualdad
de Hlder.
|T
A
(x)|
p
=
_
_
_
_
_
_
_

j=1
a
i,j
x
j
_
i
_
_
_
_
_
_
p

_
_
_
_
_
_
_

j=1
[a
i,j
x
j
[
_
i
_
_
_
_
_
_
p

42 2. APLICACIONES LINEALES CONTINUAS

_
_
_
_
_
_
_
_
_

j=1
[a
i,j
[
q
_1
q
_

j=1
[x
j
[
p
_1
p
_
_
i
_
_
_
_
_
_
p
=
= |x|
p
_
_
_
_
_
_
_
_
_

j=1
[a
i,j
[
q
_1
q
_
_
i
_
_
_
_
_
_
p
= |x|
p

p
,
y por tanto T
A
es continuo.
Veamos un anlogo continuo del ejemplo de arriba, lo que se conoce
como operadores integrales de Fredholm.
Ejemplo 2.7. Sea k(, ) : [0, 1] [0, 1] K una funcin a la
que de ahora en adelante nos referiremos como ncleo de Fredholm. El
ncleo k nos permite denir una aplicacin entre espacios funcionales
(de momento por precisar)
x T
k
(x)(s) =
_
1
0
_
1
0
k(s, t)x(t)dt
Se pueden estudiar las caractersticas de este operador en funcin del
ncleo k y del espacio en que lo denamos.
En este ejemplo, consideremos 1 < p . Sea X = L
p
[0, 1], sea
Y = L
q
[0, 1] con
1
p
+
1
q
= 1 y supongamos que k L
q
([0, 1] [0, 1]).
Entonces, para todo x X y s [0, 1] usando la versin integral de la
desigualdad de Hlder anlogamente a como lo hicimos en el Ejemplo
7.10 tenemos
[T
k
(x)(s)[
_
1
0
[k(s, t)[[x(t)[dt |x|
p
__
1
0
[k(s, t)[
q
dt
_
1
q
y por lo tanto
|T
k
(x)|
q
=
__
1
0
[T
k
(x)(s)[
q
ds
_
1
q

|x|
p
__
1
0
_
1
0
[k(s, t)[
q
dtds
_
1
q
= |x|
p
|k|
q
de donde se sigue la continuidad de T
k
: L
p
[0, 1] L
q
[0, 1].
Veamos algn ejemplo de operador lineal no continuo
Ejemplo 2.8. Sea C
1
[0, 1] el espacio de las funciones con derivada
continua, dotado de la norma del supremo, y sea
D : C
1
[0, 1] C[0, 1]
2. APLICACIONES LINEALES CONTINUAS 43
el operador derivacin. Entonces D es lineal pero no continuo, ya que,
si f
n
(t) = t
n
, entonces |f
n
| = 1 pero
|D(f
n
)| = |f
t
n
| = sup
t
|nt
n1
| = n
Ejemplo 2.9. Anlogamente, podemos denir la forma lineal
x
t
: C
1
[0, 1] K
dada por
x
t
(f) = f
t
(1)
y ver que no es continua. En efecto, veamos que ker x
t
= x
t1
0 no es
cerrado: sea f
n
= t
t
n
n
. Entonces f
t
n
(1) = 1
n
n
= 0 para todo n N, y
por tanto f
t
n
ker x
t
. Claramente f
n
tiende a f(t) = t pero f , ker x
t
.
Ya sabemos del lgebra Lineal que el espacio de operadores lineales
entre espacios vectoriales tiene a su vez estructura de espacio vectorial.
Es muy fcil ver que los operadores lineales y continuos forman un
subespacio vectorial de este (sin ms que notar que |(T + S)(x)| =
|T(x) + S(x)| |T(x)| + |S(x)| C
1
|x| + C
2
|x| = (C
1
+ C
2
)|x|
y que |T(x)| = [[|T(x)|). Denotaremos a este ltimo espacio por
L(X; Y ). Veamos que L(X; Y ) hereda de X e Y una estructura natural
de espacio normado.
Definicin 2.10. Dados dos espacios normados X e Y y un oper-
ador T : X Y denimos su norma como
|T| = sup
xB
X
|T(x)|
Es fcil ver que esa funcin es efectivamente una norma en L(X; Y ).
Lema 2.11. Dados dos espacios normados X e Y y un operador
T : X Y se tiene
|T| =nf C 0, tal que |T(x)| C|x| para todo x X =
= sup
xS
X
|T(x)|.
Demostracin. Llamemos I := nfC 0, tal que |T(x)|
C|x| para todo x X. Si C es tal que
|T(x)| C|x|
para todo x X, entonces
sup
x1
|T(x)| C
y por tanto |T| I.
44 2. APLICACIONES LINEALES CONTINUAS
Por otro lado, si 0 ,= x X, entonces
|T(x)| = |x||T
_
x
|x|
_
| |x| sup
zS
X
|T(z)|,
de donde
I sup
zS
X
|T(z)| |T| I.

Ya hemos visto que L(X; Y ) con la norma de operadores es un


espacio normado. Veamos adems que es un espacio de Banach si Y lo
es
Proposicin 2.12. Dados dos espacios normados X e Y , si Y
es un espacio de Banach entonces L(X; Y ) tambin es un espacio de
Banach.
Demostracin. La demostracin es esencialmente la misma de
siempre: consideramos una sucesin de Cauchy, le buscamos un can-
didato al lmite y nalmente vericamos que dicho candidato es efecti-
vamente un elemento del espacio y que la sucesin converge a l.
Sea (T
n
)
nN
L(X; Y ) una sucesin de Cauchy. Es fcil ver que,
para todo x X, (T
n
(x))
n
es una sucesin de Cauchy en Y . Por tanto,
puesto que Y es completo, existe y Y tal que (T
n
(x))
n
tiende a y.
Consideramos la aplicacin T : X Y denida por
T(x) = lm
n
T
n
(x).
Debemos ver que T es lineal y continua: la linealidad se sigue del
hecho de que la suma y el producto son continuos; veamos por ejemplo
la aditividad
T(x + y) = lm
n
T
n
(x + y) = lm
n
(T
n
(x) + T
n
(y)) = T(x) + T(y)
Para ver que T es continuo, por ser (T
n
) una sucesin de Cauchy, para
todo > 0 existe n
0
tal que para todo n, m 0, |T
n
T
m
| < .
Entonces, por la continuidad de la norma, para todo m n
0
(1) |T(x) T
m
(x)| = lm
n
|T
n
(x) T
m
(x)|
lm
n
|T
n
T
m
| |x| |x|
y por tanto, tomando = 1,
|T(x)| |T(x) T
m
(x)| +|T
m
(x)| (1 +|T
m
|)|x|
2. APLICACIONES LINEALES CONTINUAS 45
lo que implica que T es continuo. La convergencia de (T
n
) a T se sigue
de (1).
Probablemente sea interesante recalcar en algn momento que es
en general muy difcil calcular la norma de un operador, incluso para
muchos operadores aparentemente sencillos. Considrese por ejemplo
el caso de operadores T :
n
2

n
2
, dados por una matriz. Dada una
matriz, resulta un problema muy complejo y a menudo importante cal-
cular, o al menos estimar, su norma como operador entre esos espacios.
La siguiente proposicin es muy usada.
Proposicin 2.13. Sean X, Y, Z espacios normados, y sean T
L(X; Y ) y S L(Y ; Z). Entonces ST L(X; Z) y |ST| |S||T|
Demostracin. Basta observar que
|(S T)(x)| = |S(T(x))| |S||T||x|.

Dado un espacio normado X, llamamos X

:= L(X; K). Del Teore-


ma 2.12 se sigue que X

es siempre completo. Llamamos a X

el dual
topolgico, o simplemente dual de X. Del Teorema 2.3 se sigue que en
los espacios de dimensin innita el dual algebraico y el topolgico no
coinciden. Nosotros usaremos exclusivamente el dual topolgico, puesto
que es el que nos permitir utilizar la estructura topolgica del espacio.
Para poder estudiar en el prximo captulo las aplicaciones ge-
omtricas del Teorema de Hahn-Banach necesitaremos estar familiar-
izados con la nocin de hiperplano. Aprovechamos este momento para
introducir su denicin y probar algunos resultados elementales.
Sea X un espacio vectorial. Un hiperplano M X es un subespacio
vectorial de codimensin 1, es decir, tal que dim(X/M) = 1. Existe una
relacin biunvoca entre hiperplanos y (rectas de) formas lineales (no
necesariamente continuas).
Proposicin 2.14. Un subespacio vectorial M X es un hiper-
plano si y slo si existe una forma lineal no nula x
t
: X K tal que
M = ker x
t
.
Demostracin. Si M es un hiperplano, sea Q : X X/M la
aplicacin cociente cannica, y sea : X/M K un isomorsmo.
Entonces x
t
= Q : X K es una aplicacin lineal tal que ker x
t
=
M.
46 2. APLICACIONES LINEALES CONTINUAS
Recprocamente, si x
t
: X K es una forma lineal no nula,
llamando M = ker x
t
tenemos que la aplicacin cannica

x
t
: X/ ker x
t
K
dada por

x
t
([x]) = x
t
(x)
est bien denida, es lineal, inyectiva y sobreyectiva, y por tanto es-
tablece un isomorsmo lineal entre X/ ker x
t
y K, es decir,
dim(X/ ker x
t
) = 1.

Proposicin 2.15. Si X es un espacio normado y M X es un


hiperplano, entonces M es cerrado o denso.
Demostracin. Sea M, la clausura topolgica de M. Entonces
M es un subespacio vectorial (Proposicin 1.3) y se tiene que
M M X
Al ser dimX/M = 1, se tiene que, o bien M = M (M es cerrado) o
bien M = X (M es denso).
A la vista de la divisin de los hiperplanos en densos y cerrados, y de
la relacin entre hiperplanos y formas lineales, la siguiente proposicin
resulta natural.
Proposicin 2.16. Sea X un espacio de Banach y x
t
: X K
una forma lineal. Entonces ker x
t
es cerrado si y slo si x
t
es continua.
Demostracin. Claramente si x
t
es continua entonces ker x
t
=
x
t1
(0) es cerrado.
Recprocamente, supongamos que ker x
t
es cerrado. Sea
Q : X X/ ker x
t
la aplicacin cociente, que es continua (Ejercicio 2.2). Sea
: X/ ker x
t
K
un isomorsmo algebraico, que siempre es continuo por ser una apli-
cacin lineal entre espacios de dimensin nita (Corolario 2.2). En-
tonces
f = Q : X K
es continua y ker f = ker x
t
. Veamos nalmente que existe K tal
que x
t
= f, y por tanto x
t
es continua.
2. APLICACIONES LINEALES CONTINUAS 47
Sea x
0
X tal que f(x
0
) = 1. Entonces x
t
(x
0
) = ,= 0 (porque los
ncleos de f y x
t
coinciden). Sea ahora x X y sea a = f(x). Entonces
f(x ax
0
) = f(x) a1 = 0
y por tanto
x ax
0
ker f = ker x
t
,
es decir
x
t
(x ax
0
) = 0
de donde x
t
(x) = ax
t
(x
0
), es decir
x
t
(x) = f(x)x
t
(x
0
) = f(x) para todo x X.

Ejemplo 2.17. Veamos un ejemplo de un hiperplano denso: con-


sideremos la sucesin (e
n
) c
0
. Sea x
0
c
0
, x
0
= (1,
1
2
,
1
3
, . . .), es decir
x
0
n
=
1
n
. Es fcil ver que x
0
e
n
; n N es un sistema linealmente
independiente. Sea entonces B = x
0
e
n
; n N b
i
; i I una
base de Hamel. Entonces todo x c
0
se puede escribir en forma nica
como
x =
0
x
0
+

n
e
n
+

i
e
i
(donde slo una cantidad nita de coecientes son no nulos). Denimos
x
t
: c
0
K como x
t
(x) =
0
. As denido x
t
es una forma lineal
obviamente no nula puesto que x
t
(x
0
) = 1. Claramente c
00
ker x
t
y
puesto que c
00
es denso en c
0
se tiene que ker x
t
es denso.
Prcticas sugeridas
Ejercicio 2.1. Dos espacios normados X, Y son isomorfos si y
slo si existe una aplicacin lineal y biyectiva : X Y y existen
, > 0 tales que, para todo x X,
|x| |(x)| |x|.
Ejercicio 2.2. Sea X un espacio normado, Y X un subespacio
vectorial cerrado. Entonces la aplicacin cociente
Q : X X/Y
es continua.
Ejercicio 2.3. Sea 1 q < p . Probar que la identidad formal
i :
q

p
es continua.
48 2. APLICACIONES LINEALES CONTINUAS
Ejercicio 2.4. [22, p. 103] Si (F
n
) L(C[0, 1]) es una sucesin
de operadores positivos tal que
lm
n
F
n
(f) = f
para f = 1, x, x
2
entonces
lm
n
F
n
(f) = f
para todo f C[0, 1].
Ejercicio 2.5. Sea H = L
2
y sea C
(1)
el conjunto de las funciones
con derivada continua. Sea t [0, 1] y sea D
t
: C
(1)
K la forma
dada por
D
t
(f) = f
t
(t).
Demostrar que D
t
no se puede extender con continuidad a L
2
[0, 1].
Ejercicio 2.6 (Matriz de Hilbert). Probar que la matriz A =
(a
i,j
)
i,jN
con
a
i,j
=
1
i + j
dene un operador acotado T :
2

2
con |T| .
CAPTULO 3
Teoremas de Hahn-Banach
El Teorema de Hahn-Banach (o ms bien los Teoremas de Hahn-
Banach) juegan un papel fundamental en la teora de Espacios Nor-
mados, as como en la teora de Espacios Localmente Convexos. En su
forma analtica, el Teorema de Hahn-Banach nos permite extender for-
mas lineales denidas sobre subespacios y dominadas por funcionales
sublineales o seminormas. Esto nos garantiza que los espacios duales
son, en primer lugar, no vacos, y en segundo lugar, sucientemente ri-
cos como para determinar importante informacin estructural acerca de
los espacios. En su forma geomtrica, el Teorema de Hahn-Banach nos
permite separar por medio de hiperplanos determinados pares de con-
juntos del espacio y esto tiene una importancia fundamental, como se
ver en las aplicaciones. Las dos formas del teorema estn relacionadas
ya que, como vimos en el captulo anterior, hay una estrecha relacin
entre formas lineales e hiperplanos. Nosotros demostraremos en primer
lugar la forma analtica, y de ah deduciremos la forma geomtrica. El
lector interesado en leer una presentacin de estos teoremas en otro
orden puede consultar por ejemplo [44], [26] o [31].
La demostracin de la versin analtica del Teorema que presen-
tamos es la ms habitual, y creemos que tambin la ms elegante. En
todo el captulo hay que distinguir entre el caso real y el caso complejo,
y as lo hacemos. Como consecuencia de la forma analtica del teorema
obtenemos la existencia de formas normantes
Antes de la presentacin de las formas geomtricas del teorema,
hemos incluido una breve presentacin de los Espacios Localmente
Convexos y de los Espacios Vectoriales Topolgicos. No es estricta-
mente necesario hacerlo as, y de hecho en este curso slo utilizamos
una versin del Teorema de separacin de Hahn-Banach en Espacios
Localmente Convexos en la demostracin del Teorema de Goldstine en
el Captulo 8, demostracin que no es indispensable para la asignatura
como comentaremos en su momento.
49
50 3. TEOREMAS DE HAHN-BANACH
El motivo de que hayamos incluido esa pequea seccin sobre Espa-
cios Localmente Convexos es doble: por un lado nos permite presentar
el Teorema de separacin de Hahn-Banach en su contexto ms gen-
eral, y por otro lado nos sirve de excusa para informarle al alumno
de la existencia de tales espacios y que no todo espacio funcional es
normable. De todas formas, podemos prescindir de esa seccin y hacer
una presentacin algo menos general de los Teoremas de separacin de
Hahn-Banach tal y como se hace por ejemplo en [24] o [31].
As como el Teorema de extensin de Hahn-Banach es esencialmente
nico en su enunciado, existen numerosas versiones ms o menos gen-
erales o especializadas del Teorema de separacin. La aparentemente
ms general y de la que se siguen muchas de las otras es nuestro Teo-
rema 3.22, debido a Mazur. A continuacin hemos presentado algunos
de sus corolarios ms tiles, pero hay muchas otras versiones posibles
algunas de las cuales se pueden sugerir como ejercicios.
Seguimos con un estudio de la reexividad en espacios normados y
el estudio del dual del espacio cociente.
En este captulo comienzan a ser muy relevantes las aplicaciones.
Las de los Teoremas de Hahn-Banach son muy numerosas. Hemos in-
cluido algunas de las clsicas, pero se podran aadir muchas otras.
Debido simplemente a nuestro inters personal en este momento hemos
hecho una pequea resea de algunas aplicaciones a la Teora Econmi-
ca de estos teoremas, pero probablemente no seran las ms adecuadas
para contar en clase dado que requieren bastantes deniciones adi-
cionales.
Para la presentacin del Teorema de extensin de Hahn-Banach
hemos seguido principalmente [8]. La breve introduccin a los Espa-
cios Localmente Convexos sigue [13] aunque tambin nos hemos servi-
do de [44] y [26]. Seguimos de nuevo [8] para introducir las versiones
geomtricas del Teorema de Hahn-Banach. El lector interesado esen-
cialmente en versiones geomtricas del teorema para espacios normados
puede consultar [24, Ejercicios al nal del Captulo 2].
El Teorema de Extensin de Hahn-Banach
Aunque en este curso usaremos el Teorema de Hahn-Banach mas
frecuentemente en el contexto de los espacios normados, el teorema
sigue siendo cierto, y se utiliza a menudo en el contexto de espacios
vectoriales topolgicos localmente convexos. Incluso en el contexto de
los espacios normados, las versiones geomtricas del teorema necesitan
3. TEOREMAS DE HAHN-BANACH 51
de las nociones de seminorma y funcional sublineal. Es por ello que
antes de enunciar y demostrar el teorema de Hahn-Banach debemos
denir seminormas y funcionales sublineales.
Definicin 3.1. Sea X un espacio vectorial. Una funcin
p : X [0, +)
es una seminorma si verica
1. p(x) = [[p(x), para todo x X y K.
2. p(x + y) p(x) + p(y), para todos x, y X.
Es decir, una seminorma es exactamente igual a una norma salvo
en que puede haber elementos x ,= 0 tales que p(x) = 0.
Aunque volveremos sobre ello ms adelante, adelantamos que un
espacio localmente convexo ser un espacio vectorial cuya topologa
vendr denida por medio de una familia de seminormas.
Necesitamos tambin denir funcionales sublineales.
Definicin 3.2. Sea X un espacio vectorial sobre R o C. Un fun-
cional sublineal p sobre X es una aplicacin p : X R positivamente
homognea y subaditiva, es decir p verica
p(x) = p(x) para todo R
+
, y para todo x X
p(x + y) p(x) + p(y) para todos x, y X
Claramente, toda seminorma es un funcional sublineal, aunque no
al revs. En particular una norma es un funcional sublineal.
Puesto que el Lema de Zorn es parte esencial de la demostracin
del Teorema de Hahn-Banach, incluimos aqu su enunciado.
Lema 3.3 (Lema de Zorn). Sea A ,= un conjunto ordenado, en
el que todo subconjunto totalmente ordenado tiene una cota superior
(en ese caso se dice que A es inductivo). Entonces todo elemento de A
est mayorado por un elemento maximal (un elemento a es maximal si
a b implica a = b)
Las demostraciones (y los enunciados) del Teorema de Hahn-Banach
son levemente distintos en el caso real y el caso complejo. Merece la
pena mencionar que si bien el teorema en el caso real lo probaron
independientemente Hahn (1926) y Banach (1927), la demostracin del
caso complejo no apareci hasta 1938 en un artculo de Bohnenblust y
Sobczyk.
Teorema 3.4 (Hahn-Banach). Sea X un espacio vectorial real,
p : X R un funcional sublineal, M X un subespacio vectorial de
52 3. TEOREMAS DE HAHN-BANACH
X y f : M R una aplicacin lineal dominada por p, es decir, para
todo x M,
f(x) p(x).
Entonces existe una aplicacin lineal f : X R que extiende a f
(es decir, tal que f(x) = f(x) para todo x M) y que sigue estando
dominada por p, es decir, para todo x X,
f(x) p(x).
Demostracin. Demostraremos primero que podemos extender
f a una dimensin ms. A continuacin el Lema de Zorn operar su
magia.
Sea e X M, sea G = M [e]. Entonces todo elemento y G
admite una nica descomposicin y = x +e, con x M, R. Para
todo y G descompuesto en la forma indicada denimos
f(y) = f(x) + c
Claramente f : G R es lineal y extiende a f. Slo nos falta de-
mostrar que podemos elegir c = f(e) de manera que f p en G, es
decir de manera que
(2) f(y) = f(x) + c p(x + e) = p(y)
para todo y G.
Sabemos por hiptesis que (2) es cierta para = 0. Para > 0,
dividiendo por , la ecuacin (2) se puede escribir como
f
_
x

_
+ c p
_
x

+ e
_
y por tanto queremos que
c p
_
x

+ e
_
f
_
x

_
es decir
c nf
x
1
M
p(x
1
+ e) f(x
1
)
Para < 0, dividiendo por , la ecuacin (2) se puede escribir
como
f(x
2
) c p(x
2
e), con x
2
=
x

,
y por tanto queremos que
c sup
x
2
M
f(x
2
) p(x
2
e)
3. TEOREMAS DE HAHN-BANACH 53
Es decir, podremos elegir un c que cumpla lo pedido si y slo si
sup
x
2
M
f(x
2
) p(x
2
e) nf
x
1
M
p(x
1
+ e) f(x
1
)
Esto es lo mismo que pedir que, para todo x
1
, x
2
M,
f(x
2
) p(x
2
e) p(x
1
+ e) f(x
1
)
es decir,
f(x
1
) + f(x
2
) p(x
1
+ e) + p(x
2
e).
Pero por hiptesis sabemos que
f(x
1
) + f(x
2
) = f(x
1
+ x
2
) p(x
1
+ x
2
) = p(x
1
+ e + x
2
e)
p(x
1
+ e) + p(x
2
e)
Por tanto efectivamente existe algn c (en general ms de uno) que
verica lo pedido. Ntese que la no unicidad de c hace que tampoco la
extensin sea nica en general.
Ahora es cuando el lema de Zorn entra en accin: Consideramos el
conjunto de pares
A := (H, f
H
), donde H X es un subespacio vectorial
que contiene a M y f
H
: H R es una aplicacin lineal
que extiende a f y est dominada por p.
Podemos denir un orden natural para nuestro problema en A como
(H
1
, f
H
1
) (H
2
, f
H
2
) si H
1
H
2
y f
H
2
extiende a f
H
1
.
Con este orden A es inductivo: en efecto, sea (H
i
, f
H
i
)
iI
un con-
junto totalmente ordenado. En ese caso el par (H, f
1
) denido como
_
H =
i
H
i
f
1
(x) = f
H
i
(x) si x H
i
pertenece a A y es una cota superior de (H
i
, f
H
i
)
iI
Por tanto, puesto que A es inductivo el elemento (M, f) A es-
t mayorado por un elemento maximal, llammosle (L, f
L
). Si L no
fuera el espacio total X, usando la primera parte de la demostracin
podramos extender f a un subespacio L
t
estrictamente mayor que L
de manera que (L, f
L
) no sera maximal, una contradiccin. Por tanto
L = X y f = f
L
es la extensin buscada de f a todo X.
54 3. TEOREMAS DE HAHN-BANACH
Observacin 3.5. Merece la pena hacer aqu un par de observa-
ciones. En primer lugar que en la demostracin del Teorema no hemos
usado ninguna topologa, por lo que en particular f no tiene por qu ser
continua ni F tiene por qu ser cerrado en X (puesto que ni siquiera
hemos denido ninguna topologa en X)
En segundo lugar mencionar que este Teorema, con toda su poten-
cia que tendremos ocasin de aplicar ms adelante, tiene tambin una
limitacin que no conviene olvidar. El procedimiento de obtencin de f
es no constructivo, y adems f no es nica, ni tenemos forma de elegir
bien una extensin entre todas las posibles (en particular no podemos
en general seleccionar linealmente una extensin). Cuando veamos las
versiones geomtricas del teorema, ser natural plantearse la relacin
entre la unicidad de la extensin y la geometra del espacio. Hay varios
resultados en esta direccin. Un ejemplo de lo que se puede esperar es el
Teorema de Taylor-Foguel que nos dice que, dado un espacio de Banach
X, para todo Y X y para todo y

Y

y

tiene una extensin de


Hahn-Banach nica si y slo si X

es estrictamente convexo, es decir,


si dados f
1
, f
2
S
X
,
_
_
f
1
+f
2
2
_
_
< 1.
Hemos seguido la presentacin ms habitual del Teorema. Para cier-
tas aplicaciones puede ser interesante observar que el Teorema, con
(esencialmente) la misma demostracin, sigue siendo cierto si a p le
pedimos tan slo que sea convexa ([1]). Es fcil ver que todo funcional
sublineal es convexo.
Damos a continuacin como corolario la versin compleja del teo-
rema.
Teorema 3.6 (Hahn-Banach). Sea X un espacio vectorial real o
complejo, p : X [0, +) una seminorma, M un subespacio vecto-
rial de X y f : M K (con K = R C segn X sea real o complejo)
una forma lineal dominada por p en mdulo, es decir, para todo x M,
[f(x)[ p(x).
Entonces existe una extensin

f : X K de f tambin dominada
por p en mdulo.
Demostracin. Supongamos primero que X es un espacio vec-
torial real. Puesto que las seminormas son funcionales sublineales, el
teorema anterior nos dice que existe una extensin de f,

f : X R
tal que

f(x) p(x) para todo x X. Entonces, cambiando x por x
se tiene que

f(x) p(x)
3. TEOREMAS DE HAHN-BANACH 55
es decir

f(x) p(x)
y por tanto
[

f(x)[ p(x)
para todo x X.
Si X es un espacio vectorial complejo, tambin lo es real, de manera
que si f : M C es una forma lineal, podemos denir la forma real
g(x) := '(f)(x) : M R
que podemos extender a g con [g(x)[ p(x) para todo x X por los
razonamientos anteriores.
Ahora, la linealidad compleja de f nos dice que si f(x) = +
entonces
f(x) = + ,
es decir, que para todo x M,
(f)(x) = g(x) = '(f)(x)
Por tanto, la forma

f(x) := g(x) g(x) : X C


extiende a f. Para ver que

f est dominada en mdulo por p, dado
x X, sea = arg

f(x). Entonces
[

f(x)[ = e


f(x) =

f(e

x) = '

f(e

x) =
= g(e

x) p(e

x) = p(x)
lo que demuestra el teorema.
El teorema de Hahn-Banach tiene muchsimas consecuencias y apli-
caciones en al Anlisis Funcional. La primera de ellas es que, puesto
que los espacios de dimensin nita tienen funcionales controlados por
la norma, todo espacio normado (y tambin todo espacio localmente
convexo) tiene dual topolgico no vaco. El siguiente corolario nos dice
que no slo el dual es no vaco, sino que hay sucientes elementos para
normar cada vector de X. Enunciamos y demostramos el resultado en
el caso ms general de seminormas.
Corolario 3.7. Sea X un espacio vectorial real o complejo, p una
seminorma en X y x X. Entonces existe una forma lineal f : X
K tal que, para todo y X,
[f(y)[ p(y)
56 3. TEOREMAS DE HAHN-BANACH
y tal que
f(x) = p(x).
En particular, si X es un espacio normado entonces dado x X
existe una forma lineal y continua f de norma 1 tal que
f(x) = |x|
Demostracin. Tomamos M = [x] y denimos f : M K
como
f(x) = p(x)
para todo K. Entonces el Teorema 3.6 nos dice que existe

f :
X K que cumple lo pedido.
El siguiente resultado tambin es una consecuencia del teorema.
Nos proporciona una dualidad entre la norma de los elementos de X
y los de X

.
Corolario 3.8. Sea X un espacio normado. Entonces, para todo
x X,
|x| = sup
fB
X

[f(x)[
Demostracin. En efecto,
[f(x)[ |f||x|
y por tanto
sup
fB
X

[f(x)[ |x|
Por otro lado, tomando f
0
un funcional de norma 1 normante de x,
como en el corolario anterior, tenemos
|x| = [f
0
(x)[ sup
fB
X

[f(x)[

Espacios Localmente Convexos


El contexto natural para las versiones geomtricas de los teoremas
de Hahn-Banach es el de los espacios localmente convexos. Por ello
antes de enunciar y probar dichas versiones geomtricas dedicaremos
esta seccin a la denicin y breve estudio de los Espacios Localmente
Convexos (ELC) en particular.
Los espacios vectoriales topolgicos (EVT) son la generalizacin
natural de los espacios normados en el siguiente sentido: a menudo es
3. TEOREMAS DE HAHN-BANACH 57
necesario estudiar un espacio vectorial con una topologa bien rela-
cionada con la estructura lineal (en el sentido que vamos a precisar
a continuacin) pero dicho espacio no es normado. Buena parte -pero
no toda- de la teora (y de la intuicin) de espacios normados se puede
trasladar sin excesiva dicultad a este nuevo contexto
Empezamos con una denicin.
Definicin 3.9. Un espacio vectorial topolgico X es un espacio
vectorial en el que se ha denido una topologa que hace que
1. La aplicacin suma + : X X X es continua.
2. La aplicacin producto : KX X es continua.
Ejercicio 3.1. Sea X un EVT, y sea x X. Probar que la apli-
cacin T
x
: X X dada por
T
x
(y) = x y
es un homeomorsmo (es decir, es biyectiva y continua en ambos sen-
tidos).
Ya hemos visto que los espacios normados son EVTs.
No vamos a estudiar en esta memoria los EVTs en general, puesto
que entendemos que dicho estudio es excesivamente especializado y que
probablemente sea mejor que el contacto inicial de los alumnos con el
Anlisis Funcional ocurra en el contexto ms razonable de los espa-
cios de Banach. S queremos en esta seccin estudiar sucintamente un
tipo especial de EVTs, los Espacios Localmente Convexos (ELC), ya
que son el contexto natural para los Teoremas geomtricos de Hahn-
Banach. Adems, dado un espacio de Banach X, tanto (X, w) como
(X

, w

), que estudiaremos ms adelante, son ELCs. Por ello deni-


mos a continuacin Espacios Localmente Convexos, y a continuacin
probamos algunas propiedades bsicas de los ELC que usamos ms
adelante. Cuando estas propiedades lo sean de todos los EVT, no pedi-
remos la convexidad en el enunciado.
Los ELCs son aquellos EVT en los que cada punto posee una base
de entornos formada por conjuntos convexos. Veremos ms adelante
cmo a partir de un conjunto convexo se puede denir su funcional
de Minkowski que ser una seminorma, de manera que probaremos la
equivalencia entre tener una base de entornos convexos y tener una
topologa denida por seminormas. Elegimos de momento esta ltima
como la denicin de ELC, por parecernos ms intuitiva inicialmente.
58 3. TEOREMAS DE HAHN-BANACH
Definicin 3.10. Sea X un espacio vectorial y sea T una familia
de seminormas denidas sobre X. Sea T la topologa denida en X que
tiene como subbase los conjuntos
W
(x
0
;p;)
:= x X tales que p(x x
0
) <
para todo x
0
X, p T y > 0, de manera que un conjunto U X
es abierto si y slo si para todo x
0
X existe un entorno de x
0
W
(x
0
;p
1
,...,p
n
;)
:= x X tales que p
i
(x x
0
) < para todo 1 i n
de manera que W
(x
0
;p
1
,...,p
n
;)
U. Si adems la topologa T es Haus-
dor, decimos que X es un Espacio Localmente Convexo.
Es un ejercicio (y como tal lo proponemos) vericar que todo ELC
es un EVT. Tambin es un ejercicio vericar que si X es un espacio vec-
torial con la topologa dada por una familia de seminormas T, entonces
X es Hausdor si y slo si

p1
x X tales que p(x) = 0 = 0.
Parafraseando [1], if we had our way, all topological vector spaces
would be Hausdor. Hacemos nuestra tal armacin, puesto que no
tendremos ocasin de trabajar con EVTs no separados. En nuestras
deniciones, si bien no le pedimos a los EVT que sean separados, s se
lo pedimos a los ELC.
Recordemos la denicin de conjunto convexo.
Definicin 3.11. Sea X un espacio vectorial. Un conjunto A X
es convexo si para todos a, b A, el segmento
[a, b] = a + (1 )b; [0, 1]
est contenido en A
Es fcil ver que A es convexo si y slo si para todos a
1
, . . . , a
n
A
y para todos
1
, . . . ,
n
[0, 1] con

n
i=1

i
= 1 se tiene
n

i=1

i
a
i
A.
Tambin es fcil ver que la interseccin de conjuntos convexos es
convexo. Puesto que adems un espacio vectorial X siempre es convexo,
tiene sentido denir la envoltura convexa de un conjunto.
3. TEOREMAS DE HAHN-BANACH 59
Definicin 3.12. Sea X un espacio vectorial y sea A X. Se
dene la envoltura convexa de A, que denotamos co(A), como la inter-
seccin de todos los conjuntos convexos que contienen a A. Si adems
X es un EVT, se dene la envoltura convexa y cerrada de A, y la
denotamos como co(A), como la interseccin de todos los conjuntos
convexos y cerrados que contienen a A.
Ejercicio 3.2. Dado A X, demostrar que
co(A) =
n

i=1

i
a
i
; n N, a
1
, . . . , a
n
A,
1
, . . . ,
n
[0, 1].
Ejercicio 3.3. Sean X, Y espacios vectoriales. Si T : X Y es
una aplicacin lineal y C Y es convexo entonces T
1
(C) tambin es
convexo.
Necesitamos dos deniciones ms.
Definicin 3.13. Sea X un espacio vectorial. Un subconjunto A
X se dice equilibrado si A A para todo K tal que [[ 1.
Obviamente todos los conjuntos equilibrados contienen al origen.
La idea de su denicin es una cierta simetra respecto al origen.
Definicin 3.14. Sea X un espacio vectorial. Un subconjunto A
X se dice absorbente si para todo x X existe un > 0 tal que x A
para todo K tal que [[ > .
Adems, si a A, A se dice absorbente en a si Aa es absorbente.
Ejercicio 3.4. Dado un espacio vectorial X y una seminorma p :
X [0, ), comprobar que el conjunto
A = x X tales que p(x) < 1
es convexo, absorbente y equilibrado y que adems A es absorbente en
todos sus puntos.
El ejercicio anterior muestra que la bola unidad abierta asociada
a una seminorma es siempre un conjunto convexo, absorbente en todos
sus puntos y equilibrado. Lo interesante es que el recproco tambin es
cierto.
Proposicin 3.15. Sea X un espacio vectorial y sea A X un
subconjunto convexo, absorbente en todos sus puntos y equilibrado. En-
tonces existe una nica seminorma p : X [0, ) tal que A = x
X tales que p(x) < 1.
60 3. TEOREMAS DE HAHN-BANACH
Demostracin. Denimos p : X [0, ) como
p(x) =nft tales que t 0 y x tA.
Tenemos que ver que p est bien denida, que es una seminorma y
que A = x X tales que p(x) < 1. Puesto que A es absorbente,
para todo x X existe t > 0 tal que x tA, por lo que p est bien
denida. Veamos que p es una seminorma. Claramente p(0) = 0 puesto
que 0 A. Veamos que p(x) = [[p(x). Podemos suponer que ,= 0.
Usando que A es equilibrado tenemos que
p(x) =nft 0; x tA =nft 0; x
t

A =
=nft 0; x

A =nft 0; x
t
[[
A =
= [[nf
t
[[
0; x
t
[[
A = [[p(x).
Para terminar de comprobar que p es una seminorma nos resta ver
la propiedad triangular. Observemos como paso previo que si , 0
y a, b A entonces
a + b = ( + )
_

+
a +

+
b
_
( + )A
por la convexidad de A. Ahora, sean x, y X con p(x) = y p(y) = .
Sea > 0. Por denicin de p,
x ( + )A
e
y ( + )A.
Por tanto
x + y ( + )A + ( + )A = ( + + 2)A
y de aqu se sigue que
p(x + y) + + 2 = p(x) + p(y) + 2.
Puesto que esto es cierto para todo > 0 se tiene la desigualdad trian-
gular.
Veamos ahora que
A = x X tales que p(x) < 1.
Si p(x) = < 1 entonces para todo < < 1 se tiene que x A A
(la ltima inclusin se sigue de que A es equilibrado). Por tanto
x X tales que p(x) < 1 A.
3. TEOREMAS DE HAHN-BANACH 61
Para la otra inclusin, sea a A. Entonces p(a) 1. Puesto que A
es absorbente en todos sus puntos, en particular lo es en a y por tanto
existe > 0 tal que
a + a = y A.
Entonces a =
y
1+
y de aqu se sigue que
p(a) =
1
1 +
p(y)
1
1 +
< 1.
Slo falta por tanto ver la unicidad de p. Sea q : X [0, ) tal
que A = x X tales que q(x) < 1.
Veamos que p(x) q(x) para todo x X. La otra desigualdad se
prueba anlogamente. Sea x X y sea = q(x). Para todo > 0
q
_
x
+
_
=

+
< 1.
Por tanto
x
+
A y de aqu se sigue que
p(x)
+
= p
_
x
+
_
< 1.
Por tanto
p(x) < + = q(x) + .
Puesto que esto se tiene para todo > 0, se sigue que p q.
El resultado anterior nos invita a formular una denicin. Ntese
previamente que slo hemos usado que A fuera absorbente en ca-
da punto para obtener el menor estricto en la relacin A = x
X tales que p(x) < 1. En particular ntese que p tambin es una
seminorma si slo le pedimos a A que sea absorbente, equilibradoy
convexo.
Definicin 3.16. Sea A X un conjunto convexo, absorbente y
equilibrado. Entonces podemos denir el funcional de Minkowski de A,
que a menudo llamaremos j, como la funcin j : X [0, ) dada
por
j(x) =nf tales que 0 y x A.
Del resultado anterior se sigue que en ese caso el funcional de
Minkowski es una seminorma.
Lema 3.17. Si X es un EVT y A X es un abierto entonces A es
absorbente en todos sus puntos.
Demostracin. Para todo a A, se tiene que A a es entorno
de 0, y por tanto es absorbente.
62 3. TEOREMAS DE HAHN-BANACH
Tal y como prometimos al principio de la seccin, probamos que un
ELC es precisamente un EVT con una base de entornos convexos.
Proposicin 3.18. Sea X un EVT y sea
| = A X tales que A es abierto, convexo y equilibrado .
Entonces X es un ELC si y slo si | es una base de entornos del
origen.
Demostracin. Si X es un ELC, segn la Denicin 3.10, los
entornos de la forma W
(x
0
;p
1
,...,p
n
;)
:= x X tales que p
i
(x x
0
) <
para todo 1 i n, que son claramente abiertos, convexos y equi-
librados forman una base entornos del origen.
La otra implicacin es ms fcil.
Ms adelante necesitaremos este resultado.
Proposicin 3.19. Sea X un EVT. Sean A X un subconjunto
compacto y B X un subconjunto cerrado con A B = . Entonces
existe un entorno abierto del origen U tal que A + U y B + U no se
intersecan.
Proposicin 3.20. Sea X un EVT y f : X K una forma
lineal. Entonces son equivalentes:
(i) f es continua.
(ii) f es continua en algn punto.
(iii) f es continua en el origen.
(iv) ker f es cerrado.
Demostracin. La equivalencia entre (i), (ii) y (iii) se sigue del
hecho de que las traslaciones en los EVTs son homeomorsmos.
Claramente (iii) implica (iv).
Veamos ahora que (iv) implica (iii). Supongamos que ker f es cer-
rado y sea (x

)
A
X una red tal que x

0. Sea tambin u X
tal que f(u) = 1. Si f(x

) , 0 entonces podemos suponer (pasando a


una subred en caso necesario) que existe > 0 tal que [f(x

)[ > para
todo . Sea
y

= u
f(u)
f(x

)
x

3. TEOREMAS DE HAHN-BANACH 63
y observemos que para todo se tiene que y

ker f. Adems clara-


mente y

u. Por ser ker f cerrado se tiene que u ker f en con-


tradiccin con que f(u) = 1. Por tanto se ha de tener que f(x

) 0,
lo que nos dice que f es continua en el origen.
Como ya vimos en la forma analtica del Teorema de Hahn-Banach,
las seminormas y los funcionales sublineales juegan un papel destacado
en su enunciado y demostracin. Ya hemos visto que dado un EVT,
las seminormas estn asociadas a los conjuntos convexos, absorbentes
y equilibrados. El siguiente resultado, cuya demostracin no incluimos
por ser totalmente anloga a la de la Proposicin 3.15 nos dice que si
no le pedimos al conjunto que sea equilibrado lo que obtenemos no es
una seminorma sino un funcional sublineal. Recurdese que ya hemos
visto que los conjuntos abiertos son absorbentes.
Proposicin 3.21. Sea X un EVT y sea A X un subconjun-
to abierto y convexo que contiene al origen. Entonces el funcional de
Minkowski de A, j
A
: X [0, ) denido como
j
A
(x) =nf tales que 0 y x A
es un funcional sublineal, y A = x X tales que j
A
(x) < 1.
Antes de enunciar el teorema de Hahn-Banach en forma geomtri-
ca, debemos comprobar que los resultados bsicos referidos a hiper-
planos que probamos para espacios normados siguen siendo ciertos en
el contexto de EVTs. En particular se prueba anlogamente al caso
de espacios normados que si X es un EVT y H X es un hiperplano
entonces H es cerrado o denso. Que un hiperplano es el ncleo de una
forma lineal sigue siendo obviamente cierto puesto que ah los nicos
razonamientos involucrados eran algebraicos, no topolgicos. Por lti-
mo necesitaremos que si H es un hiperplano ncleo de una forma lineal
f entonces H es cerrado si y slo si f es continua. Esto ya lo hemos
probado en la Proposicin 3.20.
Teoremas de separacin de Hahn-Banach
El siguiente resultado se debe a Mazur.
Teorema 3.22 (Hahn-Banach, forma geomtrica). Sea X un espa-
cio vectorial topolgico (real o complejo), A X un conjunto convexo
abierto no vaco, L un subespacio afn de X que no interseca a A.
Entonces existe un hiperplano afn cerrado H que contiene a L y no
interseca a A.
64 3. TEOREMAS DE HAHN-BANACH
Demostracin. Demostramos primero el caso real. Suponemos
que 0 A (si no es as, podemos considerar A a y L a para
cualquier a A).
Por ser A un abierto convexo que contiene al origen es absorbente, y
podemos denir su funcional de Minkowski p : X [0, ). Ya hemos
visto que p es un funcional sublineal y que A = x X tales que p(x) <
1.
Sea x
0
L y consideremos el subespacio vectorial F = L x
0
.
Entonces F es el subespacio vectorial generado por L. Puesto que, por
hiptesis, 0 , L, L es un hiperplano afn de F.
Por tanto existe una forma (no necesariamente continua) f : F
R tal que L = x F; f(x) = 1. Por tanto f(x) p(x) para todo
x L (porque para todo x L se tiene que x , A y por tanto
p(x) 1). Veamos que tambin f(x) p(x) para todo x F: Sea
x F. Si f(x) 0, automticamente se sigue que f(x) p(x). Si,
en cambio f(x) > 0 consideramos el elemento
x
f(x)
F. Puesto que
f
_
x
f(x)
_
= 1, se tiene que
x
f(x)
L y entonces
p(x)
f(x)
= p
_
x
f(x)
_
1,
de donde
f(x) p(x).
Entonces podemos aplicar la forma analtica del Teorema de Hahn-
Banach, caso real, puesto que p es un funcional sublineal, y tenemos
que existe una forma lineal

f : X R que extiende f y que est
dominada por p (

f(x) p(x) para todo x X).
Sea ahora el hiperplano afn H := x X;

f(x) = 1. Claramente
contiene a L, puesto que

f extiende a f. Adems H A = porque
si y H entonces p(y)

f(y) = 1 y por tanto y , A. Para acabar,
veamos que H es cerrado: Es un ejercicio comprobar que, al igual que
ocurra con los hiperplanos vectoriales, un hiperplano afn es cerrado
o denso en X. Pero H no puede ser denso puesto que no interseca al
abierto A.
Ahora supongamos que X es un espacio vectorial complejo. Usando
los razonamientos anteriores podemos hallar un hiperplano afn real H
0
que contiene a L y que no interseca a A. Sabemos que existe x
0
L
tal que H
0
= x
0
+ H
t
0
, con H
t
0
un hiperplano vectorial real. Entonces
H
t
:= H
t
0
H
t
0
es un hiperplano vectorial complejo y H = x
0
+ H
t
es
un hiperplano afn complejo que no interseca a A y que contiene a L,
3. TEOREMAS DE HAHN-BANACH 65
ya que L = x
0
+ L
t
, donde L
t
es un espacio vectorial complejo y por
tanto L
t
= L
t
.
Se pueden dar ahora una amplia variedad de corolarios de la forma
geomtrica del Teorema. Propondremos alguno a continuacin y otros
irn en los ejercicios. Es conveniente mencionar que muy a menudo es
necesario fabricarse teoremas de separacin tipo Hahn-Banach a me-
dida para el problema en que se est trabajando. Esto ocurre a menudo,
por ejemplo, en Economa Matemtica, donde los teoremas geomtri-
cos de Hahn-Banach se usan profusamente pues son parte esencial de la
demostracin de los Teoremas del Estado del Bienestar sobre los que se
basa casi toda la Teora Econmica y Financiera moderna. Un ejemplo
(entre muchos) de esta construccin ad hoc de un teorema no trivial
tipo Hahn-Banach se puede ver en [43].
Corolario 3.23. Sea X un EVT real, A X un subconjunto
abierto convexo no vaco y B X un subconjunto convexo no vaco tal
que A B = . Entonces existe una forma lineal y continua f X

y
un R tales que, para todos x A, y B,
f(x) < f(y),
es decir, existe un hiperplano cerrado afn (f
1
()) que separa A y
B.
Demostracin. El conjunto
C = A B := a b; a A, b B =
bB
(A b)
(no A B!!) es abierto, por ser unin de abiertos, y convexo (si c
1
=
x
1
y
1
C, c
2
= x
2
y
2
C, entonces c
1
+ (1 )c
2
= x
1
+
(1 )x
2
y
1
+(1 )y
2
C por la convexidad de A y B). Adems
C es no vaco y no contiene al origen. Usando el teorema podemos
encontrar un hiperplano afn cerrado que pasa por el origen (es decir, un
hiperplano vectorial cerrado) que no corta a C. Usando la convexidad
de C podemos encontrar f X

tal que f(x) < 0 para todo c C.


Por tanto, para todo x A, y B
f(x y) = f(x) f(y) < 0
y por tanto, si llamamos
= nf
yB
f(y)
tenemos, para todo x A, y B
f(x) f(y)
66 3. TEOREMAS DE HAHN-BANACH
pero ahora, como A es abierto, se tiene que f(x) < para todo x A,
ya que si no fuera as, existira x
0
A tal que f(x
0
) = . Supongamos
por ejemplo que > 0. Por la continuidad del producto, existira > 0
tal que (1 + )x
0
A, y tendramos que f((1 + )x
0
) . Si < 0 se
razona anlogamente.
El mismo corolario admite una versin compleja
Corolario 3.24. Sea X un EVT complejo, A X un subconjunto
abierto convexo y equilibrado y B X un subconjunto convexo no
vaco tal que A B = . Entonces existe una forma (compleja) lineal
y continua f X

y un > 0 tales que, para todos x A, y B,


[f(x)[ < [f(y)[,
es decir, existe un hiperplano cerrado afn (f
1
()) que separa A y
B.
Demostracin. [8, Corollary 4, p. 32]
Finalmente damos las versiones real y compleja de un teorema de
separacin bastante utilizado. Lo enunciamos y probamos en el caso de
espacios localmente convexos.
Corolario 3.25. Sea X un ELC real. Sea A X un subconjunto
compacto, convexo no vaco y B X un subconjunto cerrado convexo
no vaco que no interseca a A. Entonces existe un hiperplano afn cer-
rado que separa estrictamente A y B, esto es, existe f X

y R
tales que
sup
xA
f(x) < < sup
yB
f(y)
Demostracin. Por la Proposicin 3.19 sabemos que existe un
entorno abierto del origen U tal que A + U y B + U cumplen las
hipotesis del Corolario 3.23 y por tanto existen f X

y R tales
que, para todos x A + U, y B + U,
f(x) < < f(y),
la ltima desigualdad es estricta porque B es abierto, razonando como
lo hicimos en el Corolario 3.23 para A.
La versin compleja del Corolario es la siguiente
Corolario 3.26. Sea X un ELC complejo, Hausdor. Sea A X
un subconjunto compacto, convexo equilibrado no vaco y B X un
subconjunto cerrado convexo no vaco que no interseca a A. Entonces
3. TEOREMAS DE HAHN-BANACH 67
existe un hiperplano afn cerrado que separa estrictamente A y B, esto
es, existe f X

y > 0 tales que para todos x A, y B,


[f(x)[ < < [f(y)[
Demostracin. [8, Corollary 6 p. 33]
Reexividad
Notemos que si tenemos un espacio normado X podemos denir
una aplicacin cannica
J : X X

dada por la relacin


J(x)(x

) = x

(x)
para todos x X, x

. Es un sencillo ejercicio comprobar que J


est bien denida (es decir, que para todo x X la aplicacin J(x) :
X

K es lineal y continua). Adems, del Teorema de Hahn-Banach


se sigue que
|J(x)| = sup
x

B
X

[J(x)(x

)[ = sup
x

B
X

[x

(x)[ = |x|
y por tanto J es una isometra (no necesariamente sobreyectiva).
Si X es un espacio de Banach se sigue fcilmente que J(X) X

es un espacio cerrado. Si X no es un espacio de Banach, entonces


J(X) X

es un subespacio cerrado en X

. Puesto que X

s es un
espacio de Banach se sigue que J(X) es la complecin de X (en X

).
Esta complecin es nica en el siguiente sentido:
Si X
1
es otro espacio de Banach tal que X X
1
y X es denso en
X
1
entonces X
1
y J(X) son isomtricos. Vemoslo. Sea
: J(X) X
1
la aplicacin dada por
(J(x)) = x.
es claramente una isometra y por tanto se extiende por densidad a
una isometra

: J(X) X
1
.
Slo nos falta comprobar que

es sobreyectiva. Sea x
1
X
1
. Puesto
que X es denso en X
1
, existe una sucesin (x
n
)
n
X tal que x
n
x
1
.
68 3. TEOREMAS DE HAHN-BANACH
Entonces la sucesin (J(x
n
))
n
J(X) es de Cauchy y por tanto existe
y J(X) tal que J(x
n
) y. Entonces

(J(x
n
))

(y)
y tambin

(J(x
n
)) x
1
por lo que x
1
=

(y) lo que termina la demostracin.
Decimos que un espacio X es reexivo si la aplicacin cannica
J : X X

es sobreyectiva. Veremos ms adelante que si 1 < p <


entonces
p
y L
p
[0, 1] son espacios reexivos. En cambio, veremos
tambin que c
0
y
1
no son reexivos.
Un espacio es reexivo si la inyeccin cannica J : X X

es sobreyectiva. En [27], James muestra un ejemplo de un espacio no


reexivo X tal que existe una isometra sobreyectiva (no la cannica,
naturalmente) : X X

.
Dual de un espacio cociente
En la siguiente seccin probamos dos resultados tiles a menudo.
Ambos pueden ser considerados simples ejercicios y, con alguna sug-
erencia, pueden ser resueltos por los propios alumnos.
Proposicin 3.27. Sea X un espacio de Banach y M X un
subespacio vectorial cerrado. Si M

= x

tales que x

(x) =
0 para todo x M entonces
X

/M

= M

donde el igual en la lnea de arriba denota que ambos espacios son


linealmente isomtricos.
Demostracin. Sea : X

/M

la aplicacin dada por


([x

]) = x

[
M
,
es decir, para todo m M,
([x

])(m) = x

(m).
est bien denida y es lineal.
Si ([x

]) = 0 entonces x

y por tanto [x

] = 0, por lo que
es inyectiva.
3. TEOREMAS DE HAHN-BANACH 69
Adems, dado m

, podemos extenderlo por Hahn-Banach a


x

. Es claro que ([x

]) = m

por lo que es sobre.


Veamos que es una isometra. Sea x

y sea m

. En-
tonces
|([x

])| = |([x

+ m

])| = sup
mB
M
|x

(m) + m

(m)| |x

+ m

|,
y tomando nmos en m

se tiene que
|([x

])| |[x

]|.
Para la otra desigualdad, dado x

, sea m

dado por
m

= X

[
M
y sea y

una extensin de Hahn-Banach de m

con la misma
norma (|y

| = |m

|). Entonces [x

] = [y

], m

= ([x

]) = ([y

]) y
|([x

])| = |m

| = |y

| |[y

]| = |[x

]|,
lo que termina la demostracin.
Proposicin 3.28. Sea X un espacio de Banach y M X un
subespacio vectorial cerrado. Entonces
(X/M)

= M

donde de nuevo el signo igual denota que los espacios son linealmente
isomtricos.
Demostracin. Sea Q : X X/M la aplicacin cociente y sea
: (X/M)

la aplicacin dada por


(y

) = y

Q.
Veamos en primer lugar que termina en M

: Para todo m M
y para todo y

(X/M)

,
(y

)(m) = y

Q(m) = y

(0) = 0.
Es claro que es lineal. Veamos que es una isometra:
Para todo y

(X/M)

,
|(y

)| = |y

Q| |Q||y

| = |y

|
lo que nos da una de las desigualdades.
Para la otra, dado y

(X/M)

y dado > 0 sea [x] X/M tal


que |[x]| < 1 y tal que
[y

[x][ |y

| .
70 3. TEOREMAS DE HAHN-BANACH
Por la denicin de la norma cociente existe m M tal que
|x + m| 1
y entonces
|(y

)| = |y

Q| [(y

Q)(x + m)| =
= [(y

Q)(x)| = [y

([x])[ |y

| ,
y de aqu se sigue que, para todo y

(X/M)

,
|(y

)| |y

|
por lo que es una isometra (y, en consecuencia, inyectiva).
Slo falta ver que es sobreyectiva. Para ello, sea m

. Sea
y

: X/M K la forma dada por


y

([x]) = m

(x).
y

est bien denida, es lineal, y es continua porque, para todo m M,


[y

([x])[ = [m

(x)[ = [m

(x + m)[ |m

||x + m|
y por tanto, tomando nmos en m, se tiene que
[y

(x)[ |m

||[x]|.
Por tanto y

(X/M)

, y claramente (y

) = m

, lo que termina la
demostracin.
Aplicaciones
Las aplicaciones de los Teoremas de Hahn-Banach tanto a prob-
lemas del Anlisis Funcional como a problemas ms aplicados son
demasiado numerosos para incluir en esta memoria ni siquiera una
cantidad signicativa de ellos. Nos limitaremos a mencionar algunas
de estas aplicaciones.
Lmites de Banach: Demostrar que existe un funcional lineal L
denido sobre

tal que
|L| = 1
Si x = (x
n
) c entonces L(x) = lm
n
x
n
Si x = (x
n
)

y x
n
0 para todo n N entonces L(x) 0.
Si x = (x
n
)

y llamamos x
t
= (x
2
, x
3
, . . .) entonces
L(x
t
) = L(x)
3. TEOREMAS DE HAHN-BANACH 71
Esta es una de las aplicaciones ms clsicas del Teorema de Hahn-
Banach y se puede consultar en muchos libros. Dos presentaciones ade-
cuadas para este curso pueden ser las de [13, p. 82] o [24, Ejercicio 19,
p.39].
Existencia de medidas sobre el crculo unidad invariantes
por rotaciones: S. Banach estaba intentando resolver este problema
cuando prob el Teorema de Hahn-Banach (y a continuacin lo aplic
para resolver el problema). Se puede ver una presentacin por ejemplo
en [22].
Sea T = z C tales que [z[ = 1. Entonces se tiene.
Teorema 3.29. Existe una medida nitamente aditiva invariante
por rotaciones denida sobre la -lgebra de todas las partes de T. Es
decir, existe una funcin
m : T(T) [0, 1]
tal que
m(T) = 1.
Para todos S, T T(T) con S T = se tiene
m(S T) = m(S) + m(T).
Para toda rotacin f : T T (dada por f(z) = ze

) y para
todo S T(T) se tiene
m(f(S)) = m(S).
Aplicaciones Econmicas Aunque probablemente no se puedan
desarrollar en clase porque nos separara excesivamente del ncleo de
la asignatura, si los alumnos mostraran inters en ello se podran co-
mentar aqu algunas aplicaciones de los Teoremas de Hahn-Banach a la
Matemtica Econmica. Hay una gran cantidad de tales aplicaciones,
mencionaremos aqu brevemente las dos que consideramos principales.
La primera son los Teoremas del Estado del Bienestar, o Teoremas de
Arrow-Debreu, demostrados en 1954 ([3] que merecieron un Premio No-
bel y que forman la base de la Teora Econmica moderna. La segunda
es el llamado Teorema Fundamental de la Valoracin de Activos, uno
de los pilares de la Matemtica Financiera, y pieza clave en los trabajos
sobre la valoracin de opciones de Black y Scholes [9, 10] y [34], que
tambin resultaron premiados con un Premio Nobel.
72 3. TEOREMAS DE HAHN-BANACH
Teoremas del estado del bienestar. Se pueden consultar en
cualquier texto riguroso de Microeconoma. Una presentacin
atractiva para un matemtico se puede ver por ejemplo en
[17]. No podemos introducir ahora toda la notacin necesaria
para enunciar y demostrar los teoremas, pero mencionaremos
simplemente que en uno de los pasos se separan dos conjuntos
por el Teorema de separacin de Hahn-Banach, y la forma
del dual asociada al hiperplano separador se puede interpretar
como un vector de precios que a cada vector de bienes le
asocia su valor.
Teorema fundamental de la valoracin de activos. Este
es uno de los resultados ms utilizados en las tcnicas de val-
oracin de Derivados Financieros, y nos dice que la ausencia
de arbitraje, una hiptesis muy razonable econmicamente so-
bre los mercados es equivalente a la existencia de una medida
en cierto espacio de probabilidad con respecto de la cual cier-
tos procesos de precios forman una martingala. Si bien las
formas nito-dimensionales del teorema son razonablemente
simples, las formas ms generales son de una gran compleji-
dad, no econmica sino matemtica. El lector interesado en
este tema puede consultar por ejemplo [14] y la bibliografa
que all aparece.
Teorema bipolar Sea X un espacio normado y sea M X y
N X

dos conjuntos no vacos. Denimos la polar de M como


M
0
= x

tales que [x

(x)[ 1 para todo x M


y la polar de N como
N
0
= x X tales que [x

(x)[ 1 para todo x

N.
Obviamente hay una cierta incoherencia en la denicin tal y como la
hemos presentado, puesto que N
0
debiera ser un subconjunto de X

;
para ser ms estrictos debiramos haber llamado a N
0
algo as como
prepolar, y haber utilizado una notacin como
0
N o algo as, como se
hace en algunos textos. En realidad el contexto natural para la nocin
de polar de un conjunto es el de los pares duales, y en ese contexto
se reduce la ambigedad, pero no vamos a incluir en esta memoria la
nocin de par dual, por lo que nos limitamos a presentar estas ideas en
el contexto de los espacios normados.
Ejercicio 3.5. Para todos M y N como en la denicin M
0
y N
0
son conjuntos cerrados, convexos y equilibrados.
3. TEOREMAS DE HAHN-BANACH 73
La nocin de polar est muy relacionada con la nocin de anulador
de un subespacio (que se corresponde con el subespacio ortogonal en
espacios de Hilbert).
Definicin 3.30. Sea M X un subespacio. Entonces denimos
su anulador M

como
M

= x

tales que x

(x) = 0 para todo x M


Ejercicio 3.6. Si M X es un subespacio entonces M
0
= M

Ejercicio 3.7. Si B A entonces A


0
B
0
.
Con esto ya podemos probar el siguiente teorema, conocido como
Teorema (de la) bipolar. Tambin es vlido en el contexto ms amplio
de par dual.
Teorema 3.31. Sea X un espacio normado, M X. Entonces
M
00
:= (M
0
)
0
= coe(M)
donde coe(M) es la envoltura convexa equilibrada y cerrada de M.
Demostracin. Sea A =
iI
A
i
donde A
i
; i I son todos los
conjuntos cerrados convexos y equilibrados que contienen a M. Hay
que probar que M
00
= A.
En primer lugar, puesto que M es cerrado convexo y equilibrado se
tiene que A M
00
.
Para el otro contenido, sea x
0
, A. Como A es cerrado y convexo,
usando el Teorema de Hahn-Banach se tiene que existe x

, R
y > 0 tales que, para todo a A
'x

(a) < < + < 'x

(x
0
).
Por ser A convexo y equilibrado sabemos que 0 A, y por tanto
0 = x

(0) < .
Entonces, sustituyendo x

por
x

se tiene que existe > 0 tal que, para


todo a A,
'x

(a) < 1 < 1 + < 'x

(x
0
).
Si a A y x

(a) = [x

(a)[e

entonces e

a A (por ser A equili-


brado) y por tanto
[x

(a)[ = 'x

(e

a) < 1 < 1 + < 'x

(x
0
).
Puesto que eso ocurre para todo a A, se sigue que x

A
0
. Notemos
que M A. Por tanto A
0
M
0
, as que x

M
0
. Pero esto implica que
x
0
, M
00
. Es decir, hemos probado que el complementario de A est
contenido en el complementario de M
00
, es decir, que M
00
A.
74 3. TEOREMAS DE HAHN-BANACH
El lector de estas notas interesado en aplicaciones econmicas de
los teoremas de Hahn-Banach, puede consultar [4] para ver una curiosa
aplicacin del Teorema Bipolar a lo que Delbaen llama coherent risk
measures, medidas del riesgo de un agente econmico coherentes en
cierto sentido.
Prcticas sugeridas
Ejercicio 3.8. Comprobar que todo ELC es un EVT.
Ejercicio 3.9. Sea X un EVT y x
0
X. Entonces la aplicacin
t
x
0
: X X dada por t
x
0
(x) = x + x
0
es biyectiva, continua y de
inversa continua (es decir, es un homeomorsmo). Como consecuencia,
los trasladados de una base de entornos del origen forman una base de
entornos de cualquier punto. Anlogamente, para todo 0 ,= K la
aplicacin h

: X X denida por h

(x) = x tambin es un
homeomorsmo.
Ejercicio 3.10. Sea X un espacio vectorial complejo. Si F
1
: X
R es una forma lineal considerando X como espacio vectorial real, de-
mostrar que F : X C denida como
F(x) = F
1
(x) F
1
(x)
es una forma lineal sobre X considerado como espacio vectorial com-
plejo.
Ejercicio 3.11. Si X es un espacio de Banach de dimensin in-
nita, demostrar que existen conjuntos convexos C
1
y C
2
tales que
C
1
C
2
= X, C
1
C
2
= y tanto C
1
como C
2
son densos en X. Sug-
erencia: Considrese una forma lineal no continua y los dos semies-
pacios en que divide a X.
Ejercicio 3.12. Demostrar que un espacio de Banach X es reex-
ivo si y slo si X

es reexivo.
Ejercicio 3.13. Demostrar que si un espacio de Banach X es re-
exivo entonces todo subespacio cerrado Y X tambin es reexivo.
CAPTULO 4
El Teorema de Baire y sus consecuencias: El
Principio de Acotacin Uniforme y el Teorema de
la Grca Cerrada.
Los Teoremas de la Aplicacin Abierta, de la Grca Cerrada, el
de Banach-Steinhaus y el principio de Acotacin uniforme, junto con el
Teorema de Hahn-Banach ya estudiado forman la base de la Teora de
Espacios de Banach y son herramientas indispensables para el estudio
de los operadores entre espacios de Banach. Los estudiamos en este
captulo. Todos ellos tienen en comn el que su demostracin ms usual
los deriva del Teorema de Baire. Quizs merezca la pena mencionar
que varias de las consecuencias de estos teoremas se pueden demostrar
directamente por mtodos de joroba deslizante.
La importancia de estos resultados se maniesta en que en todo el
resto de la memoria se usarn frecuentemente.
Una posibilidad a la hora de presentar estos resultados es utilizar el
lenguaje de las categoras de Baire para obtener con poco ms esfuerzo
versiones algo ms nas de estos resultados. Sin embargo, dado que
no vamos a necesitar esas versiones ms nas en este curso, hemos
optado por no usar dicho lenguaje de categoras para mantener una
presentacin ms simple.
El Principio de Acotacin Uniforme nos informa acerca de la posi-
bilidad de garantizar que una familia de operadores es uniformemente
acotada sabiendo que es puntualmente acotada. Antes de enunciar
formalmente ese teorema veamos otro resultado muy conocido y til, el
Teorema de Arzela-Ascoli, que tambin nos habla, aunque en otro con-
texto, de la posibilidad de obtener una acotacin uniforme partiendo
de una acotacin puntual.
Seguimos el captulo estudiando las otras dos aplicaciones antes
mencionadas del Teorema de Baire al Anlisis Funcional, los Teoremas
de la Grca Cerrada y de la Aplicacin Abierta.
75
76 4. PAU Y TEOREMA DE LA GRFICA CERRADA
Como una primera aplicacin de estos resultados estudiamos las
proyecciones en espacios de Banach, lo que nos da pie a denir sube-
spacios complementados. No es sencillo en general demostrar si un de-
terminado subespacio est o no complementado, ni es obvio presentar
ejemplos de subespacios no complementados sin usar resultados profun-
dos. Por ello, mostramos en este captulo que c
0
no est complementado
en

.
Terminamos el captulo viendo otro de los teoremas fundamentales
del Anlisis Funcional, aunque su demostracin no est relacionada con
los anteriores, el Teorema de Stone-Weierstrass. Por su importancia en
muchas aplicaciones, creemos necesario enunciarlo y probarlo en este
curso. Su principal aplicacin en este curso la veremos en el Captulo
11, cuando se utilizar para estudiar Anlisis de Fourier en el marco
de la Teora de espacios de Hilbert.
Con el paso del tiempo han ido apareciendo diferentes demostra-
ciones de este teorema fundamental. Entre ellas, una demostracin di-
recta, utilizando los polinomios de Bernstein, una demostracin uti-
lizando el Teorema de Korovkin (vase por ejemplo [31, p. 40]), o una
demostracin razonablemente corta, pero que utiliza el Teorema de
Alaoglu, el Teorema de Krein-Milman y la descripcin del dual de
C(K) como el espacio de medidas de Radon (vase por ejemplo [13,
p. 145]). Finalmente hemos elegido la demostracin seguida en [12, p.
137] puesto que no utiliza resultados profundos adicionales y a la vez
resulta, creemos, razonablemente instructiva y sencilla.
Al igual que ocurra con el Teorema de Hahn-Banach, son innu-
merables las aplicaciones de los resultados de este captulo al Anlisis
Funcional y a otras reas de la matemtica. Presentamos brevemente
al nal del captulo algunas de las aplicaciones que nos han parecido
ms adecuadas para este curso.
Para la preparacin de este captulo hemos seguido sobre todo [31]
y [24], con la adicin de [12] para el estudio del Teorema de Stone-
Weierstrass.
El Teorema de Ascoli-Arzela
Antes de enunciar y probar el Teorema de Ascoli-Arzela y el Prin-
cipio de Acotacin Uniforme veamos que responde a una pregunta nat-
ural. Observemos que una familia de funciones continuas de un espacio
mtrico en otro puede ser puntualmente acotada sin serlo uniforme-
mente. Por ejemplo, consideremos las funciones f
n
C([0, 1])
4. PAU Y TEOREMA DE LA GRFICA CERRADA 77
f
n
(t) =
_
n
2
t , si 0 t
1
n
1
t
, si
1
n
< t 1
Esta familia est puntualmente acotada ya que f
n
(0) = 0 para
cada n N y [f
n
(t)[
1
t
para todo t (0, 1]. Pero en cambio no estn
uniformemente acotadas (es decir, no admiten una misma cota valida
para todo n y para todo t) ya que f
n
(
1
n
) = n para todo n N.
Precisamente el Teorema de Ascoli-Arzela nos da condiciones su-
cientes para poder garantizar la acotacin uniforme.
Vamos a trabajar en C(T), el espacio de las funciones continuas
denidas sobre un compacto T que suponemos metrizable.
Previamente necesitamos unas deniciones.
Ya sabemos que toda funcin f C(T) es de hecho uniformemente
continua, donde la uniformidad es en los t T. A un subconjunto
E C(T) le podemos pedir que sea equicontinuo en t, y esto va a ser
una continuidad uniforme en f E.
Definicin 4.1. Sea T un espacio compacto metrizable y sea E
C(T). E se dice equicontinuo en t T si para todo > 0 existe > 0
tal que para todo f E, si d(s, t) < entonces [f(t) f(s)[ < .
Definicin 4.2. E C(T) se dice acotado en t T si
sup
fE
[f(t)[ < .
Teorema 4.3 (Ascoli-Arzela). Sea T un espacio compacto metriz-
able y sea E C(T). Supongamos que E es un conjunto acotado y
equicontinuo en t para todo t T. Entonces E C(T) es precompacto
(en particular E est acotado en la norma del supremo, es decir E est
uniformemente acotado). Por tanto, toda sucesin de E contiene una
subsucesin uniformemente convergente.
Demostracin. Tenemos que probar que E es precompacto. Sea
> 0. Por ser E equicontinuo en todo punto, para todo t T existe

t
> 0 tal que para todo f E y para todo s B(t,
t
)
[f(s) f(t)[ < .
El conjunto B(t,
t
); t T es un recubrimiento abierto de T y
por ser T compacto existen t
1
, . . . , t
n
T tales que, llamando
i
=
t
i
,
T =
n
i=1
B(t
i
,
i
).
Por ser E acotado en todo punto, existen
1
, . . . ,
n
> 0 tales que
[f(t
i
)[
i
para todo f E, y para todo 1 i n.
78 4. PAU Y TEOREMA DE LA GRFICA CERRADA
Sea ahora = max
i

i
+ . Entonces para todo t T existe i
1, . . . , n tal que para todo f E
[f(t)[ = [f(t) f(t
i
) + f(t
i
)[ +[f(t
i
)[
y por tanto E est uniformemente acotado en T, es decir E est acotado
en C(T). Veamos que de hecho E es precompacto. Sea
D

= k K tales que [k[


y para toda f E sea
e(f) = (f(t
1
), . . . , f(t
n
)) D
n

.
El Teorema de Heine-Borel nos dice que D
n

es un conjunto compacto
de K
n
(con cualquiera de sus normas). Consideramos por ejemplo la
norma en K
n
. Por ser D
n

compacto existe una cantidad nita de


bolas V
1
, . . . , V
m
de radio que cubren D
n

.
Ahora, para cada 1 j m, si
V
j
e(f); f E , =
elijamos un f
j
E tal que e(f
j
) V
j
.
Vamos entonces a mostrar que las bolas de centros f
j
y radio 5
forman un recubrimiento de E:
Sea f E. Puesto que los V
j
forman un recubrimiento de D
n

sabemos que existe j 1, . . . , m tal que e(f) V


j
. Puesto que
e(f
j
) V
j
y dado que el radio de V
j
es , sabemos que
|e(f) e(f
j
)|

<
es decir, para todo i 1, . . . , n
[f(t
i
) f
j
(t
i
)[ < 2.
Ahora, dado t T, existe i 1, . . . , n tal que t B(t
i
,
i
) y por
lo tanto
[f(t)f
j
(t)[ [f(t)f(t
i
)[+[f(t
i
)f
j
(t
i
)[+[f
j
(t
i
)f
j
(t)[ < +2+.
Es decir
|f f
j
|

4 < 5,
lo que prueba que E es precompacto.
La ltima armacin es inmediata: al ser C(T) un espacio de Ba-
nach, la clausura de E, E es un subconjunto compacto. Por tanto toda
sucesin en E tiene una subsucesin convergente (a un elemento de
E).
4. PAU Y TEOREMA DE LA GRFICA CERRADA 79
El Teorema de Banach-Steinhaus y el Principio de Acotacin
Uniforme
Antes de estudiar el Principio de Acotacin Uniforme y el Teorema
de Banach-Steinhaus, empezaremos recordando a los alumnos el Teo-
rema de Baire, que suponemos ya conocido. Si no lo fuera, podramos
incluir una demostracin.
Teorema 4.4 (Baire, 1899). Sea X un espacio mtrico. Entonces
la interseccin de una familia nita de subconjuntos abiertos densos de
X es denso (y abierto) en X.
Si adems X es completo, entonces la interseccin de una familia
contable de subconjuntos abiertos densos de X en densa en X.
Notemos que, tomando complementarios, el Teorema de Baire dice
que en un espacio mtrico completo la unin contable de cerrados con
interior vaco tiene interior vaco.
Podemos pensar el teorema de Ascoli-Arzela como una herramien-
ta que nos da condiciones sucientes para que en una familia de fun-
ciones continuas escalares denidas sobre un compacto metrizable, la
acotacin puntual implique la acotacin uniforme.
Nos preguntamos ahora cul es la situacin para una familia de
aplicaciones lineales entre espacios de Banach. La respuesta nos la da
el
Teorema 4.5 (Principio de Acotacin Uniforme). Sea X un es-
pacio de Banach, Y un espacio normado y sea T un subconjunto de
L(X; Y ) puntualmente acotado, es decir, tal que para todo x X el
conjunto T(x); T T Y est acotado. Entonces existe C > 0 tal
que
sup
TT
|T| = sup
TT ,xB
X
|T(x)| C.
Demostracin. Para todo n N, sea
D
n
= x X; |T(x)| > n para algn T T .
Puesto que, toda T T es continua, y | | tambin es una funcin
continua, tenemos que | | T : X [0, +) es continua, por lo que
D
T
n
:= (| | T)
1
(n, ) = x X tales que |T(x)| > n
es un abierto. Puesto que D
n
=
TT
D
T
n
, se sigue que D
n
es un abierto.
80 4. PAU Y TEOREMA DE LA GRFICA CERRADA
Sea x X. Por hiptesis, existe n
0
N tal que |T(x)| n
0
para todo T T . Por tanto x , D
n
0
y se sigue que
nN
D
n
= , en
particular
nN
D
n
no puede ser denso en X.
Ahora el Teorema de Baire nos dice que existe m
0
N tal que D
m
0
no es denso. Por tanto existe a X y r > 0 tal que B(a, r) D
m
0
= .
Esto quiere decir que T(B(a, r)) m
0
para todo T T , y veremos
que a partir de ah es fcil acabar sin mas que trasladar la bola al
origen.
Sea x B
X
, T T . Entonces
|T (rx + a)| m
0
(puesto que rx + a B(a, r)) de forma que
|T(x)| =
1
r
|T (rx)| =
1
r
|T (rx + a) T(a)|

1
r
(|T (rx + a)| +|T(a)|)
2m
0
r
.
Es decir
sup
xB
X
,TT
|T(x)| = sup
TT
|T|
2m
0
r
.

Observacin 4.6. Probablemente sea conveniente hacer algunos


comentarios al teorema.
En primer lugar, obsrvese que el conjunto E
0
= x X; tales que
T est acotado en x es un subespacio vectorial. Por tanto, puesto que
todo subespacio vectorial de X o bien es X o bien tiene interior vaco, se
tiene que o bien el interior de E
0
es vaco (es decir, el complementario
de E
0
es denso), o bien E
0
= X. Por lo tanto, o bien T est acotado
en todo punto, o para todo x en un conjunto denso de X, T no est
acotado en x. Esta es la formulacin del Teorema que sigue por ejemplo
[8].
En realidad, usando la notacin de categoras de Baire, se puede
ver fcilmente que la demostracin vale igual suponiendo que X es un
espacio normado y T est puntualmente acotada en un conjunto de
segunda categora.
Corolario 4.7. Sea X un espacio de Banach, Y un espacio nor-
mado y sea (T
n
)
nN
L(X; Y ) una sucesin tal que para todo x X
la sucesin (T
n
(x))
n
es convergente en Y . Denamos la aplicacin
T : X Y como
T(x) = lm
n
T
n
(x)
4. PAU Y TEOREMA DE LA GRFICA CERRADA 81
entonces
(i) (Teorema de Banach Steinhaus, 1927) T L(X; Y ) y
|T| sup
n
|T
n
| < .
(ii) Sea B X un conjunto compacto. Entonces (T
n
(x))
n
converge
a T(x) uniformemente en x B
Demostracin. (i) T es lineal, de nuevo por la continuidad de la
suma y el producto y el escalares. Para ver T es continuo, empecemos
notando que, para todo x X, el conjunto |T
n
(x)|; n N est
acotado, porque (T
n
(x))
n
es una sucesin convergente. El Principio de
Acotacin Uniforme nos dice ahora que el conjunto |T
n
|; n N est
acotado y por tanto existe el sup
n
|T
n
|. Entonces, para todo x B
X
,
|T(x)| = lm
n
|T
n
(x)| sup
n
|T
n
| <
De forma que T es continua y
|T| sup |T
n
| <
(ii) Sea > 0. Puesto que B es precompacto existen x
1
, . . . , x
m
B
tales que
B B(x
1
, ) B(x
m
, )
Puesto que T
n
(x
i
) T(x
i
) para todo 1 i m, existe n
0
N tal
que
|T
n
(x
i
) T(x
i
)| <
para todo n n
0
y para todo 1 i m. Sea ahora x B y sea x
i
tal
que |x x
i
| < . Entonces, para todo n n
0
, se tiene
|T
n
(x) T(x)| |T
n
(x x
i
)| +|(T
n
T)(x
i
)| +|T(x
i
x)|
(|T
n
| +|T|)|x x
i
| +|T
n
(x
i
) T(x
i
)| (2 sup
n
|T
n
| + 1).

Observacin 4.8. Observemos en primer lugar con un ejemplo


que no podemos prescindir de la hiptesis de la completitud de X ni
en el Principio de Acotacin Uniforme ni en el Teorema de Banach-
Steinhaus. Consideremos X = c
00
con la norma del supremo (en ese
caso su completado es c
0
) y para todo n N sea, para todo x c
00
T
n
(x) =
n

j=1
x(j)
82 4. PAU Y TEOREMA DE LA GRFICA CERRADA
Entonces T
n
c

00
= L(c
00
; K) y |T
n
| = n para todo n N. Sea x c
00
y sea i
x
N tal que x(j) = 0 para todo j i
x
. Entonces, para todo
n N,
[T
n
(x)[ i
x
|x|

<
y en cambio el conjunto |T
n
|; n N no est acotado. Esto muestra
que el resultado del Principio de Acotacin Uniforme no es cierto en
este caso. Para ver que el Teorema de Banach-Steinhaus tambin falla,
deniendo la aplicacin lineal T

: c
00
K como
T

(x) =

j=1
x(j)
se tiene que T
n
(x) T

(x) para todo x c


00
, pero T

no es continuo
porque T

(1,
(n)
. . ., 1, 0, 0, . . .) = n y (1,
(n)
. . ., 1, 0, 0, . . .) B
c
00
para todo
n N.
Ahora es fcil probar como consecuencia que un conjunto B E
est acotado si y slo si est dbilmente acotado, y que un operador
es continuo si y slo si es dbilmente continuo, donde el sentido de
dbilmente acotado y dbilmente acotado es el dado por el enunciado
siguiente.
Teorema 4.9. Sean X e Y espacios normados y sean B X un
subconjunto y T : X Y una aplicacin lineal. Entonces
(i) B es acotado si y solo si para todo x

(B) est acotado


en K
(ii) T es continua si y slo si y

T es continua para todo y

Y

.
Demostracin. Es fcil ver que si B est acotado tambin est
dbilmente acotado. Para la otra implicacin, supongamos que para
todo x

, x

(B) K est acotado. Sea J : X X

la inmersin
cannica de X en su bidual. Entonces el conjunto J(x); x B es
un subconjunto de X

= L(X

; K) donde X

es un espacio de Ba-
nach. Adems, para todo x

el conjunto [J(x)(x

)[; x B =
[x

(x)[; x B est acotado. Por lo tanto, el Principio de Acotacin


Uniforme nos dice que el conjunto |J(x)|; x B est acotado, y
como |J(x)| = |x|, se sigue que B est acotado.
Para la parte (ii), de nuevo una implicacin est clara. Para la otra,
supongamos que T es dbilmente continua. Sea B = T(x); x B
X

Y . Entonces para todo y

Y

, y

(B) = (y

T)(B
X
) est acotado,
de forma que la parte (i) nos dice que B est acotado, es decir T(B
X
)
est acotado, es decir T es continuo.
4. PAU Y TEOREMA DE LA GRFICA CERRADA 83
Los Teoremas de la Grca Cerrada y de la Aplicacin
Abierta
Al igual que hicimos al introducir el Principio de Acotacin Uni-
forme y el Teorema de Banach-Steinhaus, antes de introducir los Teo-
remas de la Grca Cerrada y de la Aplicacin Abierta reexionaremos
brevemente acerca de la pregunta natural a que los Teoremas dan re-
spuesta.
Sean X e Y espacios mtricos y sea T : X Y una aplicacin. T
es continua si y slo si es continua por sucesiones, es decir, si para toda
sucesin (x
n
) X tal que x
n
x X se tiene que T(x
n
) T(x).
Por otro lado, T se dice cerrada si para toda sucesin (x
n
) X tal
que x
n
x X y T(x
n
) y Y se tiene que y = T(x).
Se sigue inmediatamente de la denicin que una aplicacin contin-
ua es cerrada. En cambio el recproco no es cierto, como lo prueba el
siguiente ejemplo. Sea X = Y = R y sea
T(x) =
_
1
x
, si x ,= 0
0 , si x = 0
Ejercicio 4.1. Demostrar que T es cerrada si y slo si la grca
de T, esto es, el conjunto
Gr(T) := (x, T(x)) X Y ; x X
es cerrado en X Y con la topologa producto.
Notemos tambin que si una aplicacin cerrada T es biyectiva, en-
tonces su inversa T
1
tambin es cerrada, ya que que su grca es la
misma (una est en X Y y la otra en Y X, pero ambos espacios
son claramente homeomorfos). En cambio la inversa de una aplicacin
biyectiva y continua no tiene por qu ser continua, como lo prueba
el siguiente ejemplo: Sea X = [0, 2), Y = z C; [z[ = 1 y sea
T : X Y la aplicacin dada por
T(x) = e
ix
T es continua y biyectiva pero T
1
no es continua.
Vamos a ver ahora cul es la situacin para aplicaciones lineales
entre espacios de Banach. Seguimos [8] para la demostracin.
84 4. PAU Y TEOREMA DE LA GRFICA CERRADA
Teorema 4.10 (Grca Cerrada). Sean X, Y espacios de Banach,
y sea T : X Y una aplicacin lineal. Entonces T es cerrada si y
slo si T es continua.
Demostracin. Ya hemos comentado que si T es continua en-
tonces es cerrada. Veamos la otra implicacin. Probamos primero el
siguiente Aserto, en el que slo usaremos la linealidad de T
Aserto: Para todo > 0 existe > 0 tal que
B
X
T
1
(B
Y
)
donde B
X
= x X, tales que |x| y anlogamente B
Y
Demostracin del Aserto: Sea > 0. Consideramos los conjuntos
G
n
= nT
1
_

2
B
Y
_
.
La unin
nN
G
n
es un recubrimiento de X, y por tanto tambin lo
es
nN
G
n
. (Observese que todava no sabemos que T sea continua, lo
nico que usamos es que T est bien denida). Por el Teorema de Baire
(tomando complementarios) existe n
0
N tal que G
n
0
tiene interior no
vaco, y por tanto contiene una bola cerrada de radio > 0 y centro
x
0
, lo que implica que si |x x
0
| entonces x n
0
T
1
_

2
B
Y
_
. Es
fcil ver que esto es equivalente a que
x
n
0
T
1
_

2
B
Y
_
y por lo tanto tambin se verica, para todo n n
0
,
(3)
x
n
T
1
_

2
B
Y
_
.
Adems, existe n
1
N tal que, para todo n n
1
,
(4)
x
0
n
T
1
_

2
B
Y
_
(esto simplemente porque T(x
0
) Y )
Sea entonces n
2
= maxn
0
, n
1
y sea =

n
2
. Entonces para todo
y B
Y
se tiene
y =
n
2
y + x
0
x
0
n
2
=
n
2
y + x
0
n
2

x
0
n
2
.
Ahora, puesto que |n
2
y + x
0
x
0
| , la ecuacin (3) implica que
n
2
y + x
0
n
2
T
1
_

2
B
Y
_
.
4. PAU Y TEOREMA DE LA GRFICA CERRADA 85
Por otro lado, de la ecuacin (4) se sigue que
x
0
n
2
T
1
_

2
B
Y
_
y ahora es fcil ver que
y T
1
(B
Y
),
es decir
B
Y
T
1
(B
Y
)
lo que termina la demostracin del Aserto.
Dado un > 0, llamemos () a (uno de los) que nos proporciona
el Aserto, y observemos que para cada > 0 siempre podemos suponer
que () < .
Continuamos con la demostracin del Teorema. Suponemos que T
es cerrada, y vamos a probar que, para todo > 0
T
_

2
_
B
X
_
B
Y
lo que probar que T es continua en el origen, y por tanto en todo
punto.
As, jamos > 0 y x
_

2
_
B
X
. Elijamos x
1
T
1
(

2
B
Y
), con
x x
1
(

4
)B
X
,
(esto es, |xx
1
| (

4
)). A continuacin elegimos x
2
T
1
(

4
B
Y
) con
x x
1
x
2
(

8
)B
X
,
y procedemos inductivamente para construir una sucesin (x
i
)
iN
tal
que x
i
T
1
(

2
i
B
Y
) y
x
i

j=1
x
j
(

2
i+1
)B
X
,
y por tanto,
x
i

j=1
x
j
T
1
_

2
i+1
B
X
_
.
Entonces T(x
i
)

2
i
B
Y
y por tanto la sucesin z
n
=

n
i=1
T(x
i
) es
una sucesin de Cauchy que converge a un punto z Y . Puesto que
z
n
B
Y
para todo n, se tiene que z B
Y
.
Adems x

n
j=1
x
j
(

2
n+1
)B
X


2
n+1
B
X
(porque () < ) y
por tanto la sucesin t
n
=

n
j=1
x
j
converge a x. Puesto que T(t
n
) =
86 4. PAU Y TEOREMA DE LA GRFICA CERRADA
z
n
y puesto que T es cerrada, tenemos que T(x) = z, por lo que
x T
1
(B
Y
), lo que prueba que (

2
)B
X
T
1
B
Y
y termina la
demostracin del teorema.
Observacin 4.11. Ntese que el teorema dice que si X e Y cumplen
las hiptesis, entonces para probar que una aplicacin T sea continua
basta comprobar que si (x
n
)
nN
X es una sucesin convergente a 0
tal que la sucesin (T(x
n
))
nN
Y converge a y Y entonces y = 0.
Esta es la forma en que se usa el Teorema en muchas aplicaciones.
El siguiente corolario es de gran importancia en mltiples aplica-
ciones.
Corolario 4.12. Sean X, Y dos espacios de Banach y sea T :
X Y un operador (aplicacin lineal y continua) biyectiva. Entonces
T
1
: Y X es continua.
Demostracin. Si T es continua, su grca es cerrada. Pero la
grca de T
1
es la misma que la de T (una en X Y y la otra en
Y X) por lo que la grca de T
1
tambin es cerrada. Ahora el
Teorema de la Grca Cerrada nos garantiza que T
1
es continua.
Continuamos el captulo estudiando el Teorema de la Aplicacin
Abierta.
Definicin 4.13. Dados dos espacios topolgicos X e Y , una apli-
cacin T : X Y se dice abierta si la imagen de cualquier abierto
es un abierto.
Claramente, si la aplicacin T es inyectiva, T es abierta si y slo si
T
1
es continua.
Entonces tenemos el siguiente resultado.
Teorema 4.14. Sean X, Y espacios normados y T : X Y una
aplicacin lineal. Entonces T es una aplicacin abierta si y slo si existe
> 0 tal que
B
Y
T(B
X
),
es decir, para todo y Y existe x X con |x| |y| de manera que
T(x) = y. En particular, las aplicaciones abiertas son sobreyectivas.
Demostracin. Supongamos que T es una aplicacin abierta. En-
tonces transforma entornos abiertos de 0 en entornos abiertos de 0. Sea
U
X
= x X tales que |x| < 1.
4. PAU Y TEOREMA DE LA GRFICA CERRADA 87
Entonces existe > 0 tal que
U
Y
T(U
X
) T(B
X
).
Sea 0 <
t
< . Entonces

t
B
Y
U
Y
T(U
X
) T(B
X
).
Ahora es fcil ver que
=
1

t
cumple lo pedido.
Para la otra implicacin, supongamos que
B
Y
T(B
X
)
y sea G X un abierto. Veamos que T(G) es un abierto.
Sea a G. Por ser G abierto existe > 0 tal que
a + B
X
G.
Entonces
T(a + B
X
) = T(a) + T(B
X
) T(G)
y por lo tanto
T(a) + B
Y
T(G)
por lo que T(G) es un abierto.
Teorema 4.15. Sean X, Y espacios normados.
1. Si Z X es un subespacio cerrado, entonces la aplicacin
cociente
Q : X X/Z
es continua y abierta.
2. Sea T : X Y una aplicacin lineal tal que ker(T) E es
cerrado. Sea T : X/ ker(T) Y la aplicacin lineal dada por
T([x]) = T(x).
Entonces T es abierta si y slo si T es abierta.
Demostracin. Para probar (1), en primer lugar Q es continuo
porque |[Q(x)[| = |[[x][| |x|. Para ver que Q es abierta, utilizamos
el resultado anterior. Sea un > 0 cualquiera y sea [x] X/Z. Entonces
nf|x + z|; z Z = |[[x][| < (1 + )|[[x][|
por tanto existe z
0
Z tal que |x + z
0
| < (1 + )|[[x][|. Puesto que
Q(x + z
0
) = [x] , podemos hacer = 1 + en el teorema anterior y
obtenemos que Q es abierta.
88 4. PAU Y TEOREMA DE LA GRFICA CERRADA
Para probar (2), puesto que ker(T) es cerrado, sea Q : X
X/ ker(T) la aplicacin cociente. Entonces para todo A X,
T(A) = T(Q(A)).
Puesto que Q es abierta, se sigue trivialmente que si T es abierta
tambin lo es T.
Por otro lado, para todo A X/ ker(T), se tiene que
T(A) = T(Q
1
(A))
ya que Q es sobre.
Puesto que Q es continuo por (1), se sigue que si T es abierta T
tambin lo es.
Finalmente ya tenemos todas las herramientas para probar
Teorema 4.16 (Teorema de la Aplicacin Abierta, Banach 1932).
Sean X, Y espacios de Banach y sea T : X Y una aplicacin
cerrada y sobre. Entonces T es continua y abierta.
Demostracin. Por el Teorema de la Grca Cerrada, T es con-
tinua. Por tanto ker(T) = T
1
(0) es cerrado y la aplicacin
T : X/ ker(T) Y
denida por
T([x]) = T(x)
es continua (esto lo vimos en el Teorema 2.1, punto (v)). Podramos
tambin no incluirlo en aquel Teorema y demostrarlo ahora.) Por tan-
to T es cerrada. T es inyectiva (esto es lgebra) y, puesto que T es
sobre tambin lo es T (ms lgebra). Por tanto T es una aplicacin
lineal cerrada y biyectiva. As pues la aplicacin inversa T
1
tambin
es lineal cerrada y biyectiva (es cerrada porque la grca es la misma
cambiando el orden de los pares). Puesto que Y y X/ ker(T) son es-
pacios de Banach, tenemos que T
1
es continua, es decir T es abierta.
Por el Teorema anterior T es abierta.
Proyecciones en un espacio de Banach
Definicin 4.17. Sea X un espacio normado e Y X un subespa-
cio vectorial cerrado. Un operador (lineal y continuo) P : X X es
una proyeccin sobre Y si P
[
Y
: Y Y es la identidad y si P(x) Y
para todo x X.
4. PAU Y TEOREMA DE LA GRFICA CERRADA 89
Definicin 4.18. Sea X un espacio normado e Y X un sube-
spacio vectorial cerrado. Y se dice complementado en X si existe una
proyeccin de X sobre Y .
Ejercicio 4.2. Sea X un espacio normado, Y X un subespacio
vectorial cerrado y P : X X un operador (lineal y continuo).
Probar que P es una proyeccin si y slo si P
2
= P, P(x) Y para
todo x X y P : X Y es sobreyectiva.
Proposicin 4.19. Sea X un espacio de Banach e Y X un
subespacio vectorial cerrado. Y est complementado en X si y slo si
existe un subespacio vectorial cerrado Z X tal que X = Y Z en el
sentido algebraico, es decir, X = Y + Z e Y Z = 0. En ese caso
se dice que Z es un complementario de Y en X.
Demostracin. Supongamos que existe Z como en el enunciado.
Tanto Y como Z son espacios de Banach y por tanto (Y Z, | |
1
) es
un espacio de Banach, donde
|(y, z)|
1
= |y| +|z|.
Denimos
: Y Z X
como
(y, z) = y + z.
es claramente lineal. Tambin es continua (considerando la norma 1
en Y Z) ya que
|(y, z)| = |y + z| |y| +|z| = |(y, z)|
1
.
Adems es inyectiva: (y, z) = 0 implica y + z = 0 lo que a su vez
implica que y = z = 0 por ser X = Y Z.
Tambin es sobreyectiva: para todo x X, x se puede escribir
como x = y + z (de forma nica) y (y, z) = x.
Por los Teoremas de la Grca Cerrada (o de la Aplicacin abierta)
sabemos que
1
es continua. De hecho se tiene, aunque no lo usamos
explcitamente, que X es isomorfo a (Y Z, | |
1
); es un ejercicio com-
probar que podemos sustituir la norma 1 por, por ejemplo, cualquier
norma p.
Notemos adems que la aplicacin
1
: Y Z Y Z dada por

1
(y, z) = y es claramente una proyeccin.
Denimos entonces la aplicacin
P =
1

1
: X Y Z y Z X
Es ahora muy fcil comprobar que P es una proyeccin sobre Y .
90 4. PAU Y TEOREMA DE LA GRFICA CERRADA
Supongamos ahora que Y X est complementado, y sea P :
X X una proyeccin. Sea Z = ker P. Z es claramente cerrado.
Adems, si y Y Z se tiene que P(y) = y (porque y Y ) y que
P(y) = 0 (porque y Z) y por tanto y = 0, luego Y Z = 0.
Por otro lado, sea x X, sea y = P(x) Y y sea z = x y.
Entonces
P(z) = P(x) P(y) = y y = 0,
por tanto z Z y x = y + z, por lo que X = Y + Z.
Ejercicio 4.3. Sea Y X un subespacio complementado, sea P :
X X una proyeccin y sea Z = ker P un complementario de Y en
X. Entonces Z est complementado en X y
Id P : X X
es una proyeccin. Sugerencia: Usar el Ejercicio 4.2.
Desde el punto de vista puramente algebraico, todo subespacio vec-
torial est complementado. Se puede probar, dado un espacio de Ba-
nach X, que todo subespacio cerrado Y X est complementado si
y slo si X es Hilbert. En cambio, dado un subespacio cerrado Y de
un espacio de Banach X, no es en general nada sencillo averiguar si
Y est complementado o no. El artculo [38] (al menos partes de l)
puede ser ledo por no especialistas que deseen aprender algo acerca de
este problema.
En esta lnea, con tcnicas elementales se puede probar que c
0
no
est complementado en

. Por claridad, extraemos parte de la de-


mostracin como lema previo.
Lema 4.20. Existe una familia T de subconjuntos de N tal que
card(T) = card(R)
Para todo F T se tiene que card(F) =
0
Para todos F
1
, F
2
T
card(F
1
F
2
) <
0
Demostracin. Sea i : N Q una biyeccin. Para todo r R
elijamos una sucesin (q
r
n
)
n
Q tal que q
r
n
r. Sea
: R N
N
la aplicacin dada por
(r) =
_
i
1
(q
r
n
)
_
n
.
4. PAU Y TEOREMA DE LA GRFICA CERRADA 91
Es fcil ver que es inyectiva. Sea ahora
T = (r); r R.
No es difcil ver que T cumple lo pedido.
Teorema 4.21. c
0
no est complementado en

Demostracin. [24, Thm. 95].


Veamos que los cocientes por subespacios complementados son ex-
actamente lo que esperaramos que fueran.
Proposicin 4.22. Sea X un espacio de Banach, Y X un sube-
spacio complementado de manera que X = Y Z. Entonces (X/Y ) =
Z, donde la igualdad quiere decir que ambos espacios son isomorfos
(pero no necesariamente isomtricos).
Demostracin. Sea : Z X/Y dada por (z) = [z]. Fcil-
mente se ve que es lineal y continua. Es inyectiva porque si [z] = 0
entonces z Y , y puesto que Y Z = 0, se sigue que z = 0. Es
sobreyectiva porque si [x] X/Y y x = y + z, con y Y y z Z, en-
tonces (z) = [x]. Por tanto es biyectiva. Los Teoremas de este captulo
nos dicen ahora que es un homeomorsmo lineal
Veamos ahora que los subespacios nito dimensionales estn siem-
pre complementados. Aprovechamos la ocasin para denir las Bases
de Auerbach.
Definicin 4.23. Dado un espacio de Banach Y con dimY = n <
una base de Auerbach de Y es un sistema e
1
, . . . , e
n
, e

1
, . . . , e

tal que e
1
, . . . , e
n
es una base de Y , e

1
, . . . , e

n
Y

, para todos
1 i, j n
e

i
(e
j
) =
ij
y, para todos 1 i, j n,
|e
i
| = |e

j
| = 1.
(Obsrvese que de aqu se sigue en particular que e

1
, . . . , e

n
son lin-
ealmente independientes, y por tanto una base de Y

, ya que si

i
e

i
= 0
entonces para todo j

i
e

i
(e
j
) =
j
= 0.
92 4. PAU Y TEOREMA DE LA GRFICA CERRADA
Ejercicio 4.4. [24, Thm 89] Sea X un espacio de Banach, Y X
un subespacio n-dimensional. Entonces existen vectores e
1
, . . . , e
n

Y , e

1
, . . . , e

n
X

tales que e
1
, . . . , e
n
, e

1[
Y
, . . . , e

n[
Y
es una base
de Auerbach. Por lo tanto Y est complementado en X y existe una
proyeccin P de X sobre Y con |P| n.
Existe una versin de este resultado garantizando la existencia de
una forma dbil de Bases de Auerbach (no de proyecciones acotadas)
en todo espacio de Banach separable [36].
Por otro lado, del Teorema de Kadets-Snobar se sigue que todo
espacio n-dimensional admite una proyeccin de norma menor o igual
que

n (ver, por ejemplo, [16, Theorem 4.18]).
El Teorema de Stone-Weierstrass
Sea K un espacio compacto, y C(K, R) el espacio de Banach de
las aplicaciones continuas f : C(K) R, dotado de la norma del
supremo. Observemos en primer lugar que C(K, R) tiene adems de la
estructura de espacio de Banach, una estructura natural de lgebra.
Recordamos al lector la denicin de lgebra.
Definicin 4.24. Sea A un espacio vectorial sobre K. Decimos que
A es un lgebra si existe un producto : AA A denido en l tal
que (A, +, ) es un anillo y adems
(x y) = (x) y para todos x, y A, K.
Siguiendo la costumbre habitual escribiremos xy en lugar de x y
siempre que no haya riesgo de confusin. Claramente C(K, R) con el
producto punto a punto es un lgebra.
Decimos que un conjunto A C(K, R) es reticulado si para todos
f, g A las funciones sup(f, g) e nf(f, g) pertenecen a A, donde estas
funciones se denen como
sup(f, g)(t) := maxf(t), g(t) para todo t K,
y anlogamente la funcin nf(f, g). Es un ejercicio vericar que si
f, g C(K, R) entonces sup(f, g) C(K, R) e nf(f, g) C(K, R).
Antes de probar el Teorema de Stone-Weierstrass necesitamos dos
lemas. El primero es
Lema 4.25. Sea A C(K, R) un conjunto reticulado. Entonces
f A (la clausura topolgica de A) si y slo si para todos s, t K, f
4. PAU Y TEOREMA DE LA GRFICA CERRADA 93
es el lmite sobre s, t de funciones de A, es decir, dados s, t K y
dado > 0 existe g
s,t
A tal que
[f(s) g
s,t
(s)[ < y [f(t) g
s,t
(t)[ < .
Demostracin. Una de las implicaciones est clara. Para la otra,
supongamos que para todos s, t K, f es el lmite sobre s, t de
funciones de A. Sea > 0 y sea g
s,t
A tal que
(5) [f(s) g
s,t
(s)[ < y [f(t) g
s,t
(t)[ < .
Denamos
W
s,t
= u K tales que g
s,t
(u) < f(u) + .
Como la funcin (g
s,t
f) es continua, se tiene que W
s,t
K es
abierto y usando (5) es muy fcil ver que contiene a t. Por tanto, jado
s K la familia de conjuntos W
s,t
; t K forma un recubrimiento
abierto de K. Al ser K compacto podemos extraer de este recubrim-
iento un subrecubrimiento nito W
s,t
i
; 1 i n.
Denamos ahora
g
s
=nf
i
(g
s,t
i
).
Puesto que A es un conjunto reticulado se tiene que g
s
A. Para todo
u K, existe i 1, . . . , n tal que u W
s,t
i
, es decir g
s,t
i
(u) < f(u)+
y por tanto
g
s
< f + , es decir g
s
(u) < f(u) + para todo u K.
Adems
g
s
(s) > f(s) ,
ya que, para todo t K, se tiene que [f(s) g
s,t
(s)[ < .
Denamos ahora
W
s
= u K tales que g
s
(u) < f(u) .
Como la funcin (g
s
f) es continua, se tiene que W
s
K es abier-
to y es fcil ver que contiene a s. Por tanto la familia de conjuntos
W
s
; s K forman un recubrimiento abierto de K. De nuevo por ser
K compacto podemos extraer de este recubrimiento un subrecubrim-
iento nito W
s
j
; 1 j m.
Denamos ahora
g = sup
j
(g
s
j
).
De nuevo por ser A es un conjunto reticulado se tiene que g A.
Adems se tiene
f < g < f +
94 4. PAU Y TEOREMA DE LA GRFICA CERRADA
y puesto que esto se puede hacer para todo > 0 se sigue que f A.
Necesitamos un lema ms. Previamente recordemos que si A es
un lgebra entonces decimos que A
t
A si A
t
con las operaciones
inducidas tambin es un lgebra. Obviamente, al igual que ocurra con
subespacios vectoriales, basta con que sea cerrada para las operaciones
del lgebra.
Lema 4.26. Sea A C(K, R) una sublgebra cerrada (en el sentido
topolgico). Entonces A es un conjunto reticulado.
Demostracin. Observando que
sup(f, g) =
f + g +[f g[
2
y
nf(f, g) =
f + g [f g[
2
queda claro que basta demostrar que si f A entonces [f[ A
Para ello vamos a ver que [f[ es el lmite uniforme de polinomios
en f de la forma

m
n=1
a
n
f
n
.
Es fcil ver que podemos suponer sin perdida de generalidad que
|f| 1.
Notemos que para todo > 0 y para todo x R, se tiene
0 (x
2
+
2
)
1
2
[x[
Por otro lado, para todo x [1, 1] se tiene
x
2
+
2
= 1 +
2
+ (x
2
1) = (1 +
2
)(1 + u),
donde
u =
x
2
1
1 +
2
.
Adems, para todo x [1, 1]
[u[ <
1
1 +
2
< 1.
Por lo tanto, la serie de Taylor de (1+u)
1
2
converge uniformemente
a (1 + u)
1
2
en el intervalo [1, 1]. Es decir, existe un polinomio P(x)
tal que, para todo x [1, 1],
[(x
2
+
2
)
1
2
P(x)[ .
En particular se tiene que
[P(0)[ 2.
4. PAU Y TEOREMA DE LA GRFICA CERRADA 95
Finalmente, si hacemos Q = P P(0), se tiene
|Q(f) [f[| 4.

Necesitamos una denicin ms.


Definicin 4.27. Sean K, Y dos conjuntos y sea A Y
K
un con-
junto de aplicaciones de K en Y . Decimos que A separa puntos de K
si para todos s, t K existe f A tal que f(s) ,= f(t).
Finalmente podemos enunciar y probar el resultado principal de
este captulo. Probamos primeramente el caso real.
Teorema 4.28 (Stone-Weierstrass). Sea A C(K, R) una subl-
gebra tal que
1. A separa puntos de K.
2. Para todo t K existe f A tal que f(t) ,= 0.
Entonces A = C(K, R).
Demostracin. Por los lemas anteriores, basta ver que para todos
s, t K con s ,= k y para todos , R existe f A tal que f(s) =
y f(t) = . Puesto que A separa puntos sabemos que existe g A tal
que g(s) ,= g(t). Supongamos inicialmente que g(s) ,= 0 ,= g(t). Veamos
entonces que existen a, b R tales que
f = ag + bg
2
cumple lo pedido. Para esto tenemos que solucionar el sistema
_
= ag(s) + bg
2
(s)
= ag(t) + bg
2
(t)
y sabemos que este sistema tiene solucin si
g(s)g
2
(t) g(t)g
2
(s) = g(s)g(t)(g(s) g(t)) ,= 0.
Por hiptesis sabemos que esta condicin se cumple, y por tanto existen
a y b que cumplen lo pedido.
Si la funcin g que separa s y t cumpliera que, por ejemplo, g(s) = 0,
entonces por las hiptesis sobre A existe otra funcin g
t
A tal que
g
t
(s) ,= 0. Entonces podemos elegir un > 0 sucientemente pequeo
de manera que
g
tt
= g + g
t
separa s y t y no se anula en ninguno de ellos, y podemos aplicarle
ahora a g
tt
los razonamientos anteriores.
96 4. PAU Y TEOREMA DE LA GRFICA CERRADA
Los siguientes corolarios son a menudo cmodos de aplicar.
Corolario 4.29. Si f
i
; i I C(K, R) es un conjunto que
separa puntos de K y si para todo t K existe f
i
tal que f
i
(t) ,= 0,
entonces toda funcin f C(K, R) se puede escribir como el lmite
uniforme de polinomios (sin trmino constante) en los elementos f
i
.
Demostracin. Los polinomios en esas funciones forman una sub-
lgebra que verica las condiciones del Teorema 4.28
Corolario 4.30. Sea A C(K, R) una sublgebra tal que
1. A separa puntos de K.
2. 11 A, donde 11 es la funcin constantemente igual a 1.
Entonces A = C(K, R).
Corolario 4.31 (Weierstrass, 1885). En C([0, 1], R) los polinomios
son densos.
Veamos nalmente la versin compleja del Teorema.
Corolario 4.32. Sea K un conjunto compacto y sea A C(K, C)
una sublgebra (sobre C) tal que
1. A separa puntos.
2. Para todo x K existe f A tal que f(x) ,= 0
3. Para todo f A, tambin f A, donde f(t) = f(t), el conju-
gado complejo de f(t).
Entonces A = C(K, C), donde A denota la clausura topolgica de
A.
Demostracin. Sea
A
r
= '(f); f A.
Es fcil ver que A
r
es una sublgebra (sobre R) de C(K, R) y que
verica las condiciones 1 y 2 del Teorema 4.28 (Si f(x) ,= f(y) entonces
'(f)(x) ,= '(f)(y) '(f)(x) ,= '(f)(y); anlogamente, si f(x) ,= 0
entonces '(f)(x) ,= 0 '(f)(x) ,= 0). Por lo tanto A
r
= C(K, R),
donde A
r
es la clausura topolgica de A
r
y se ve fcilmente que entonces
A = A
r
+ A
r
= C(K, C).

Aplicaciones
Al igual que con el Teorema de Hahn-Banach, hay multitud de
aplicaciones de los teoremas de este captulo al Anlisis Funcional o a
otras reas del Anlisis. Citamos aqu alguna de ellas.
4. PAU Y TEOREMA DE LA GRFICA CERRADA 97
Existencia de solucin de ecuaciones diferenciales: Una apli-
cacin del Teorema de Ascoli-Arzela que se puede ver en [22, p. 30] es
el siguiente teorema.
Teorema 4.33. Sea f : R
2
R una funcin continua en un
abierto D. Entonces, para todo (x
0
, y
0
) D la ecuacin diferencial
y
t
= f(x, y)
admite una solucin local (no necesariamente nica) que pasa por el
punto (x
0
, y
0
).
Divergencia de la serie de Fourier de funciones continuas:
Se tiene el siguiente resultado.
Teorema 4.34. Existe un conjunto denso E C[, ] tal que
para toda f E la serie de Fourier de f diverge en el 0.
Es bastante frecuente encontrar esta aplicacin en los libros. Una
buena referencia para este curso puede ser [31, p. 144] o [24, ej 39 p.
59].
En [22, p. 78] se puede ver una aplicacin similar a la divergencia
de polinomios interpoladores.
Mtodos de Integracin: En [31, p. 146] se puede ver una apli-
cacin de los resultados de este captulo a la convergencia de mtodos
de integracin.
Condensacin de singularidades: La idea general de un princi-
pio de condensacin de singularidades es ver condiciones bajo las cuales
se puede garantizar que, si para todo punto t
0
en un conjunto denso
existe una funcin f
0
en un conjunto dado de funciones T que tiene
una singularidad en t = t
0
entonces existe f F tal que f tiene una
singularidad en todo t S. En el caso particular de operadores entre
espacios de Banach se tiene el siguiente resultado.
Teorema 4.35. Sea X un espacio de Banach y sea Y un espacio
normado. Sea (T
m
n
)
n,mN
L(X; Y ) una sucesin (indexada por n y
m) tal que para todo m N existe x
m
X tal que
lmsup
n
|T
m
n
(x
m
)| = +.
Entonces existe x X tal que para todo m N
lmsup
n
|T
m
n
(x)| = +.
98 4. PAU Y TEOREMA DE LA GRFICA CERRADA
Se puede ver por ejemplo en [22].
Propiedad de lifting de
1
. Se tiene el siguiente resultado
Teorema 4.36. Sean X e Y espacios de Banach y Q : X Y
un operador sobreyectivo. Entonces, para todo operador T :
1
Y
existe un operador

T :
1
X tal que QT = T.
Se puede ver por ejemplo en [8, Ex. 3, p. 15].
Todo espacio de Banach separable es isomorfo a un cociente
de
1
, o todo espacio de Banach es isomorfo a un cociente de

1
() para algn . Se puede ver por ejemplo en [24, Theorem 91].
Soluciones de ecuaciones diferenciales. Sean X, Y espacios de
Banach y T L(X; Y ) un operador sobre, es decir para todo y F la
ecuacin
(6) T(x) = y
tiene solucin en X. Por los Teoremas de la Aplicacin Abierta, existe
R tal que, para todo y Y existe una solucin x de (6) con
|x| |y|.
En muchas ocasiones la posibilidad de acotar la norma de una solu-
cin de la ecuacin en trminos de la norma del trmino independiente
es importante. Por ilustrar esa importancia consideremos la siguiente
situacin. Supongamos, y este es el caso a menudo, que T no es slo
sobre sino biyectiva, de forma que los teoremas de este captulo nos
garantizan que T
1
tambin es lineal y continua. En ocasiones, aunque
(6) tenga solucin para todo trmino independiente y, slo sabemos
calcular una solucin x cuando y pertenece a un cierto conjunto denso
D F. Si ahora y F D, podemos hallar una sucesin (y
n
) D tal
que y
n
y y una sucesin (x
n
) E tal que T(x
n
) = y
n
. Obviamente
la pregunta es si podemos garantizar que x
n
x donde x es el nico
elemento de X tal que T(x) = y.
La respuesta es s, ya que
|x x
n
| = |T
1
(y) T
1
(y
n
)| |T
1
||y y
n
| 0
De forma que tiene sentido decir que x
n
es una solucin aproxima-
da de (6) y adems vemos que x varia continuamente con respecto
a y. Estos razonamientos son los que estn detrs de ciertas tcnicas
de perturbacin utilizadas en la resolucin de ecuaciones diferenciales:
4. PAU Y TEOREMA DE LA GRFICA CERRADA 99
perturbamos el trmino independiente levemente (de y a y
n
) y obten-
emos una solucin aproximada x
n
que est sucientemente cerca de x
(en cierta norma) si y
n
est sucientemente cerca de y (en otra cierta
norma).
Vemoslo con un ejemplo. Sea la ecuacin diferencial lineal de orden
m con coecientes variables dada por
(7) a
m
(t)x
(m)
(t) + + a
1
(t)x
t
(t) + a
0
(t)x(t) = y(t) t [0, 1]
donde a
j
C[0, 1] para todo 1 j m y a
m
(t) ,= 0 para todo
t [0, 1].
Consideramos las condiciones iniciales
(8) x(0) = x
t
(0) = = x
(m1)
(0) = 0
Se demuestra en los cursos elementales de ecuaciones diferenciales que
para todo y C[0, 1] existe una nica solucin de (7) que verica
(8). Consideramos los espacios vectoriales Y = C[0, 1], X = x
C
(m)
[0, 1] tales que x(0) = x
t
(0) = = x
(m1)
(0) = 0. Y es un
espacio de Banach con la norma del supremo, y se puede comprobar
que si dotamos a X de la norma
|x| = |x|

+|x
t
|

+ +|x
(m)
|

entonces tambin X es un espacio de Banach. Denimos ahora la apli-


cacin
T : X Y
dada por
T(x) = a
m
x
(m)
+ + a
1
x
t
+ a
0
x.
Es sencillo vericar que T es lineal y continua. Del teorema de
existencia y unicidad antes mencionado se sigue que T es biyectiva.
Supongamos que tenemos un mtodo para solucionar (7) (es de-
cir, para calcular T
1
(y)) cuando y es un polinomio. Del Teorema de
Weierstrass se sigue que los polinomios son densos en Y . Sea ahora
y
0
Y y sea (p
n
) Y una sucesin de polinomios que converge a y
0
.
Entonces si x
n
es la solucin de (7) cuando el trmino independiente
es p
n
, tenemos que x
n
x
0
, donde x
0
es la nica solucin de (7) verif-
icando las condiciones iniciales (8) cuando el trmino independiente es
y
0
. En particular esto implica que
x
n
||

x
0
,
x
t
n
||

x
t
0
100 4. PAU Y TEOREMA DE LA GRFICA CERRADA
.
.
.
x
(m)
n
||

x
(m)
0
Prcticas sugeridas
Ejercicio 4.5. Sea X un espacio de Banach. Entonces una base
algebraica suya tiene cardinal nito o no contable.
Solucin: Supongamos que X admitiera una base contable (e
i
)
iN
.
Para cada n N sea F
n
el subespacio generado por e
1
, . . . , e
n
. F
n
es cerrado por ser de dimensin nita y puesto que (e
i
)
iN
es una base
algebraica, se tiene que X =
nN
F
n
. Por el Teorema de Baire tiene
que existir n
0
N tal que F
n
0
tiene interior no vaco. Pero si F
n
0
contiene una bola abierta entonces contiene tambin una bola abierta
centrada en el origen de manera que F
n
0
es un entorno del origen, lo
que implica que X = F
n
0
, una contradiccin con que la dimensin de
X sea innita.
Ejercicio 4.6. Sean X, Y, Z espacios normados, con X o Y espa-
cio de Banach. Sea
F : X Y Z
una aplicacin bilineal separadamente continua (esto es, para cada x
X la aplicacin lineal F
x
: Y Z dada por
F
x
(y) = F(x, y)
es continua, y anlogamente en la segunda variable. Entonces F es
continua, es decir existe > 0 tal que para todo (x, y) X Y
|F(x, y)| |x||y|
Ejercicio 4.7. Los Teoremas de la Aplicacin Abierta y de la Gr-
ca Cerrada fallan si no hay completitud. [31, Example 10.7, p.177]
Ejercicio 4.8. Sea X un espacio normado en el que consideramos
dos normas | |
1
y | |
2
. Si X es un espacio de Banach con ambas y
si existe C R
+
tal que | |
1
C| |
2
entonces las dos normas son
equivalentes.
Ejercicio 4.9. Sean X, Y espacios de Banach, T : X Y
un operador sobreyectivo. Entonces Y es isomorfo a un cociente de
X, en concreto a X/ ker T. Sugerencia Considerar el operador :
X/ ker T Y dado por ([x]) = T(x) y vericar que es biyectivo y
continuo. A continuacin aplicar el Teorema de la Aplicacin Abierta.
4. PAU Y TEOREMA DE LA GRFICA CERRADA 101
Ejercicio 4.10. Sea E un espacio compacto y sea (f
i
)
n
i=1
C(E, R)
un conjunto que separa puntos de E. Demostrar que E es homeomorfo
a un subconjunto de R
n
Ejercicio 4.11. Demostrar que el lgebra generado por 1, x
2
es
densa en C[0, 1] pero no en C[1, 1].
CAPTULO 5
Espacios duales y operadores traspuestos
En varios libros de Anlisis Funcional se puede leer que el Anli-
sis Funcional es sobre todo dualidad y teora espectral. Sin entrar en
la posible exactitud de esa frase, no cabe duda que la nocin de es-
pacio dual es fundamental en muchas de las aplicaciones del Anlisis
Funcional: muchas propiedades estructurales y geomtricas slo se en-
tienden si se formulan o se estudian en trminos del dual, y las nociones
de convergencia dbil y convergencia dbil

necesitan de la denicin de
espacio dual para su misma denicin. Una vez denido el dual de un
espacio, dado un operador entre espacios normados tiene sentido denir
su traspuesto, lo que hacemos en este captulo puesto que necesitamos
esa nocin para poder desarrollar la Teora Espectral de operadores
compactos en los captulos siguientes.
Dedicamos adems buena parte de este captulo a estudiar los duales
de algunos espacios de Banach clsicos: c
0
,
p
, L
p
[0, 1] (1 p < )
y C[0, 1]. El estudio del dual de este ltimo nos lleva a hacer una pe-
quea presentacin de la integral de Riemann-Stieltjes y las funciones
de variacin acotada. Si los alumnos conocieran Teora de la Medi-
da estudiaramos directamente la representacin del dual de C(K) en
trminos de medidas de Radon.
El estudio de los duales de
p
y c
0
lo hemos basado sobre todo en
[24], el del dual de C[0, 1] en [5] y el de los duales de L
p
en [42] y [31].
Espacio dual
Comenzamos recordando la denicin de espacio dual.
Definicin 5.1. Sea X un espacio normado. Entonces
X

:= L(X; K) = T : X K; T lineal y continuo


Como ya vimos anteriormente todo espacio dual es un espacio de
Banach, por la completitud de K.
En este sentido, veamos que si partimos de un espacio normado
no completo o de su completado, el dual es el mismo. Notemos que si
103
104 5. ESPACIOS DUALES Y OPERADORES TRASPUESTOS
dos espacios de Banach X, Y , o dos espacios normados en general, son
isomtricos (es decir existe una aplicacin : X Y lineal, biyectiva
y continua tal que |(x)| = |x| para todo x X) entonces todas
las estructuras de X e Y que conocemos y utilizamos (su estructura
conjuntista, su estructura vectorial y su norma, y junto con ella su
estructura topolgica) se conservan, por lo que no podemos distinguir
un espacio del otro. Por tanto, a partir de ahora diremos que X = Y
si X e Y son isomtricos.
Proposicin 5.2. Sea X
0
X un subespacio denso. Entonces
X

0
= X

con la identicacin natural.


Demostracin. Sea : X

0
el operador restriccin, es
decir
(T) = T
[
X
0
Claramente es lineal y continuo, con
|(T)| |T|
De la densidad de B
X
0
en B
X
se sigue que de hecho
|(T)| = |T|.
Slo nos falta ver que es sobre. Sea T
0
X

0
. El Teorema de
Hahn-Banach nos garantiza que existe T X

(con |T| = |T
0
|) tal
que (T) = T
0
, lo que termina la demostracin.
Estudiamos a continuacin los duales de los espacios de Banach
clsicos que hemos visto hasta ahora.
Proposicin 5.3 (Riesz). c

0
=
1
Demostracin. Dado x

0
, para cada n N denimos x

n
=
x

(e
n
) donde e
i
= (0, . . . , 0,
(i)
1 , 0, 0, . . . , ). A continuacin Denimos el
operador
: c

0

1
de la siguiente forma
(x

) = (x

n
)
nN
Veamos que esta bien denido: si x

n
= x

(e
n
) = [x

n
[e

n
, para
todo m N, consideramos el vector
v
m
= (e

1
, e

2
, . . . , e

m
, 0, 0, . . .)
. Puesto que v
m
B
c

0
se tiene que
x

(v
m
) = x

_
m

n=1
e

n
e
n
_
=
m

n=1
e

n
x

(e
n
) =
m

n=1
[x

n
[ |x

|.
5. ESPACIOS DUALES Y OPERADORES TRASPUESTOS 105
Por tanto

n=1
[x

n
[ existe y es menor o igual que |x

|. Esto prueba
que est bien denido y puesto que claramente es lineal, los razon-
amientos anteriores tambin prueban que es continuo y || 1.
Vamos a usar ahora que el completado de (c
00
, | |

) es c
0
, por lo
que para todo x

0
, se tiene que |x

| = sup
xB
c
00
x

(x).
Dado x

0
, para todo x B
c
00
, suponiendo x
j
= 0 para todo
j m, se tiene
x

(x) = x

_
m

n=1
x
n
e
n
_
=
m

n=1
x
n
x

n
|x|

n=1
[x

n
[

n=1
[x

n
[

n=1
[x

n
[ = |(x

)|
por lo que |x

| |(x

)| y esa era la desigualdad que nos faltaba


para probar que es una isometra. Al ser isometra automticamente
es inyectiva y slo nos resta comprobar que es sobre.
Sea a = (a
n
)
1
. Consideramos la forma x

0
dada por
x

(x) =

n=1
a
n
x
n
Claramente, para todo x c
0
la serie

n=1
a
n
x
n
es absolutamente
convergente, puesto que

n=1
[a
n
x
n
[ sup
n
[x
n
[

n=1
[a
n
[ = |x|

|a|
1
.
Por tanto x

es efectivamente un elemento de c

0
y claramente (x

) =
a.
Proposicin 5.4 (Riesz).

1
=

Demostracin. Dado x

1
denimos x

n
= x

(e
n
), y denimos
el operador
:

como
(x

) = (x

n
)
Veamos que est bien denido.
|(x

)| = sup
n
[x

n
[ = sup
n
[x

(e
n
)[ |x

|
por lo que est bien denido y, puesto que es claramente lineal, tam-
bin es continuo.
106 5. ESPACIOS DUALES Y OPERADORES TRASPUESTOS
Anlogamente a la demostracin anterior, vamos a usar ahora que
el completado de (c
00
, | |
1
) es
1
, por lo que para todo x

1
, se tiene
que |x

| = sup
xB
c
00
x

(x).
Dado x


1
, para todo x = (x
n
) B
c
00
, suponiendo x
j
= 0 para
todo j m, se tiene
x

(x) = x

_
m

n=1
x
n
e
n
_
=
m

n=1
x
n
x

n
sup
n
[x

n
[
m

n=1
[x
n
[
|(x

)|
m

n=1
[x
n
[ |(x

)|

n=1
[x
n
[ |(x

)|
por lo que |x

| |(x

)| y es una isometra. Slo resta compro-


bar que es sobre.
Sea a = (a
n
)

. Consideramos la forma x

1
dada por
x

(x) =

n=1
a
n
x
n
Claramente, para todo x
1
la serie

n=1
a
n
x
n
es absolutamente
convergente, puesto que

n=1
[a
n
x
n
[ sup
n
[a
n
[

n=1
[x
n
[ = |a|

|x|
1
.
Por tanto x

es efectivamente un elemento de

1
y claramente (x

) =
a.
Proposicin 5.5 (Riesz). Sean 1 < p, q < tales que
1
p
+
1
q
= 1.
Entonces

p
=
q
.
Demostracin. Dado x

p
, para cada n N denimos x

n
=
x

(e
n
).
Entonces denimos el operador
:

p

q
de la siguiente forma
(x

) = (x

n
)
nN
Veamos que esta bien denido: Dado x

p
, para todo m N,
consideramos el vector v
m
= ([x

1
[
q1
e

1
, . . . , [x

m
[
q1
e

m
, 0, 0, . . .).
Notemos que si
1
p
+
1
q
= 1 entonces
1
p
=
q1
q
por lo que p(q 1) = q.
As
5. ESPACIOS DUALES Y OPERADORES TRASPUESTOS 107
m

n=1
[x

n
[
q
=
m

n=1
[x

n
[
q1
[x

n
[ =
m

n=1
[x

n
[
q1
e

n
x

(e
n
) =
= x

_
m

n=1
[x

n
[
q1
e

n
e
n
_
= x

(v
m
) |x

||v
m
|
p
=
= |x

|
_
m

n=1
[x

n
[
p(q1)
_1
p
= |x

|
_
m

n=1
[x

n
[
q
_1
p
Hemos probado que
m

n=1
[x

n
[
q
|x

|
_
m

n=1
[x

n
[
q
_1
p
y de aqu se sigue que
_
m

n=1
[x

n
[
q
_1
q
=
_
m

n=1
[x

n
[
q
_
1
1
p
|x

|.
Por tanto est bien denido y es lineal y continuo con || 1.
Vamos a usar ahora que el completado de (c
00
, | |
p
) es
p
, por lo
que para todo x

p
, se tiene que |x

| = sup
xB
c
00
x

(x).
Dado x

p
, para todo x B
c
00
, suponiendo x
j
= 0 para todo
j m, se tiene, utilizando la desigualdad de Hlder, que
x

(x) = x

_
m

n=1
x
n
e
n
_
=
m

n=1
x
n
x

n
|x|
p
|(x

)|
q
por tanto |x

| |(x

)| y es una isometra. Veamos que es


sobreyectiva:
Sea a = (a
n
)
q
. Consideramos la forma x

p
dada por
x

(x) =

n=1
a
n
x
n
De nuevo por la desigualdad de Hlder, para todo x
p
la serie

n=1
a
n
x
n
es absolutamente convergente, puesto que

n=1
[a
n
x
n
[ |(a
n
)|
q
|(x
n
)|
p
.
Por tanto x

es efectivamente un elemento de

p
y claramente (x

) =
a.
108 5. ESPACIOS DUALES Y OPERADORES TRASPUESTOS
El dual de C[0, 1]
Vamos a describir el dual de C[0, 1] como cierto subconjunto de
las funciones de variacin acotada, utilizando la Integral de Riemann-
Stieltjes, que quizs nuestros alumnos ya conozcan. Si no la conocen,
esta puede ser una buena oportunidad para presentarla y que se famil-
iaricen con sus propiedades bsicas.
Comenzamos deniendo funciones de variacin acotada.
Definicin 5.6. Una funcin g : [0, 1] R se llama de variacin
acotada si su variacin V (g) es nita, es decir, si
V (g) := sup
0<x
0
...<x
n
=1
_
n

i=1
[g(x
i
) g(x
i1
)[
_
< .
Es muy fcil ver que con las operaciones puntuales, el conjunto
de las funciones de variacin acotada BV [0, 1] es un espacio vectorial.
Dado que V (g) = 0 implica que g es constante, es tambin fcil ver que
|g|
v
= [g(0)[ + V (g)
dene una norma sobre BV [0, 1].
Recordemos la denicin de la Integral de Riemann-Stieltjes.
Dadas f, g : [0, 1] R, para cada particin de [0, 1] = 0 = x
0
<
< x
n
= 1 y para cada vector = (
1
. . . ,
n
), con
i
[x
i1
, x
i
]
denimos una suma de Riemann-Stieltjes de f respecto de g como
S(f, , ; g) =
n

i=1
f(
i
)(g(x
i
) g(x
i1
))
La funcin f se dice integrable Riemann-Stieltjes respecto de g,
y escribimos f 1(g), si existe el lmite de la red S(f, ,

; g)

cuando recorre el conjunto dirigido de todas las particiones de [0, 1]


y

es una eleccin ja de para cada . A este lmite si existe lo


llamaremos
_
1
0
fdg
y se puede demostrar que no depende de la eleccin de .
Las siguientes propiedades de la integral de Riemann-Stieltjes son
fciles de probar y se pueden ver por ejemplo en [2].
5. ESPACIOS DUALES Y OPERADORES TRASPUESTOS 109
El conjunto 1(g) de las funciones integrables Riemann-Stieltjes
respecto de g es un espacio vectorial y la aplicacin
T
g
: 1(g) R
denida como
T
g
(f) =
_
1
0
fdg
es una forma lineal.
Aditividad respecto al intervalo: Para todo c (0, 1) y para
todo f 1(g),
_
1
0
fdg =
_
c
0
fdg +
_
1
c
fdg
Linealidad respecto al integrador. Si f 1(g)1(h), entonces
f 1(g + h) para todo , R y
_
1
0
fd(g + h) =
_
1
0
fdg +
_
1
0
hdg
Si g es de variacin acotada entonces toda funcin continua
f es integrable. En ese caso no es necesario tomar el lmite
en la red, sino que se puede considerar una sucesin ()
n
de
particiones que verique
|
n
| := max[x
i
x
i1
[; 1 i n 0.
Adems siempre
S(f, , ; g) |f|

|g|
v
por lo que

_
1
0
fdg

|f|

|g|
v
Por tanto, la aplicacin T
g
: C[0, 1] R es una forma
lineal y continua para cada g BV [0, 1], y |T
g
| V (g)
|g|
v
. Lo interesante ahora es que tambin podemos proceder
en sentido opuesto.
Teorema 5.7 (Riesz). Si T : C[0, 1] R es una forma lineal y
continua, entonces existe g BV [0, 1] con g(0) = 0 tal que T = T
g
y
|T| = |g|
v
110 5. ESPACIOS DUALES Y OPERADORES TRASPUESTOS
Demostracin. Sea T C[0, 1]

. Si existiera g BV [0, 1] con


g(0) = 0 tal que T = T
g
, tendramos
g(x) = g(x) g(0) =
_
x
0
dg =
_
1
0

[0,x]
dg = T(
[0,x]
)
y parece que bastar con denir
g(x) = T(
[0,x]
).
El problema es que en general
[0,x]
, C[0, 1], por lo que la expresin
T(
[0,x]
) no tiene sentido. Ser el Teorema de Hahn-Banach quien acu-
da en nuestra ayuda. Notemos que C[0, 1] es un subespacio vectorial
(incluso cerrado) de B[0, 1], las funciones reales acotadas denidas so-
bre [0, 1]. Podemos entonces extender T a una forma T B[0, 1]

con
|T| = |T|. Denamos las funciones
x
como

x
=
[0,x]
si x ,= 0
y

0
= 0.
Ahora ya podemos denir g : [0, 1] R como
g(x) = T(
x
) para todo x [0, 1]
Veamos que g cumple lo que esperamos de ella.
En primer lugar, g BV [0, 1]. En efecto, para toda particin
= 0 = x
0
< x
1
< < x
n
= 1
tenemos
n

i=1
[g(x
i
) g(x
i1
)[ =
n

i=1

i
(g(x
i
) g(x
i1
)) =
= T
_
n

i=1

i
_

x
i

x
i1
_
_
|T|
_
_
_
_
_
n

i=1

i
_

x
i

x
i1
_
_
_
_
_
_
=
= |T|
_
_
_
_
_
n

i=1

(x
i1
,x
i
]
_
_
_
_
_
= |T|,
donde
i
= sgn(g(x
i
) g(x
i1
)).
Por lo tanto g BV [0, 1] y V (g) |T|. Como adems g(0) = 0,
tenemos
V (g) = |g|
v
|T|.
Veamos ahora que T = T
g
. Sea f C[0, 1]. Denamos
f
n
(t) =
n

k=1
f
_
k
n
_
(k
n
(t) (k1)
n
(t)).
5. ESPACIOS DUALES Y OPERADORES TRASPUESTOS 111
Por ser f uniformemente continua se tiene que f
n
tiende a f en la
norma uniforme (| |

). Por lo tanto
T(f
n
) =
n

k=1
f
_
k
n
__
g
_
k
n
_
g
_
(k 1)
n
__
T(f) = T(f).
Pero, por las propiedades que hemos enunciado y la denicin de la
integral de Riemann-Stieltjes se tiene que tambin
T(f
n
) =
n

k=1
f
_
k
n
__
g
_
k
n
_
g
_
(k 1)
n
__

_
1
0
fdg.
En consecuencia
T(f) =
_
1
0
fdg,
es decir, T = T
g
y por lo visto anteriormente |T| = |g|
v
.
Ya hemos visto que todo elemento de C[0, 1]

se puede ver como


una funcin g BV [0, 1]. Nos gustara ahora para que la identicacin
fuera perfecta tener unicidad en la representacin. Eso no es posible,
ya que como sabemos las extensiones por Hahn-Banach en general no
son nicas, y para cada extensin de T tenemos una g diferente. Lo
que vamos a hacer es identicar un cierto subespacio de BV [0, 1] como
el dual de C[0, 1]. Para ello necesitaremos algunos resultados previos
Proposicin 5.8. Sea g BV [0, 1]. Entonces T
g
= 0 si y slo si,
para todo c (0, 1),
g(0) = g(1) = g(c
+
) = g(c

),
donde
g(c
+
) = lm
xc
+
g(x)
y anlogamente g(c

).
Demostracin. Sea g tal que T
g
= 0 y sea c (0, 1). Para cada
h > 0 tal que c + h 1 denimos
f
h
(t) =
_
_
_
1 si 0 t c
0 si c + h t 1
y lineal en el resto
Entonces
T
g
(f
h
) = 0 =
_
1
0
f
h
dg = g(c) g(0) +
_
c+h
c
f
h
dg
112 5. ESPACIOS DUALES Y OPERADORES TRASPUESTOS
Integrando por partes se tiene
_
c+h
c
f
h
dg = g(c) +
1
h
_
c+h
c
g(t)dt
luego
g(0) =
1
h
_
c+h
c
g(t)dt g(c
+
)
cuando h 0.
Anlogamente se prueba que g(1) = g(c

). Finalmente tomando
f = 11 se tiene que 0 = g(1) g(0).
Recprocamente, si g es como en la hiptesis, entonces g es con-
stante en sus puntos de continuidad, que forman un conjunto denso de
[0, 1] (pues su complementario es a lo sumo numerable). Tomando una
sucesin de particiones con norma tendiendo a 0 formadas exclusiva-
mente por puntos de continuidad de g, las correspondientes sumas son
todas nulas, y por tanto su lmite
_
1
0
fdg tambin lo es.
Definicin 5.9. Una funcin g NBV [0, 1] se llama normalizada
si g(0) = 0 y g es continua por la derecha en todo punto c (0, 1).
Llamamos NBV [0, 1] al subespacio vectorial de BV [0, 1] formado por
las funciones normalizadas.
Proposicin 5.10. Si g
1
, g
2
NBV [0, 1] cumplen que T
g
1
g
2
= 0,
entonces g
1
= g
2
Demostracin. Por ser g
1
, g
2
NBV [0, 1] se tiene
g
1
(0) = g
2
(0) = 0.
Por lo tanto (g
1
g
2
)(0) = 0. Por la Proposicin 5.8 se tiene g
1
(1)
g
2
(1) = g
1
(0) g
2
(0) y por tanto g
1
(1) = g
2
(1), y anlogamente para
todo c (0, 1) usando la continuidad por la derecha de g
i

Proposicin 5.11. Para cada g BV [0, 1] existe una nica fun-
cin g NBV [0, 1] tal que T
g g
= 0. Adems V ( g) V (g).
Demostracin. La unicidad se sigue de la proposicin anterior.
En cuanto a la existencia, basta con denir
g(c) =
_
_
_
0 si c = 0
g(1) g(0) si c = 1
g(c
+
) g(0) si c (0, 1)
Veamos que g cumple lo pedido. Sea c (0, 1). Sea > 0. Existe
> 0 tal que si c < s < c + entonces
5. ESPACIOS DUALES Y OPERADORES TRASPUESTOS 113
[g(c
+
) g(s)[ .
Por lo tanto, tomando (t
n
) (s, c + ) una sucesin decreciente que
converge a s (esto lo escribimos (t
n
) s) y dejando que n tienda a
innito se tiene que, para todo s (c, c + ),
[g(c
+
) g(s
+
)[ .
De aqu se sigue que
lm
s
n
c
g(s
n
) = g(c)
y por lo tanto g es continua a la derecha en todo c [0, 1).
De forma anloga se prueba que
g(c

) = g(c

) g(0),
y por lo tanto la funcin g g verica las condiciones de la Proposicin
5.8. Por tanto ya slo falta probar que V ( g) < . Para esto, sea
0 = t
0
< t
1
< < t
n
= 1
una particin de [0, 1] y sea > 0. Sean s
i
(t
i
, t
i+1
) (0 i n) tales
que
[g(t
+
i
) g(s
i
)[ <

n
.
Entonces, haciendo s
n
= t
n
= 1 se tiene
n

i=1
[ g(t
i
) g(t
i1
)[ 2 +
n

i=1
[g(s
i
) g(s
i1
)[ 2 + V (g),
de donde
V ( g) V (g)
y se concluye la demostracin.

Finalmente podemos enunciar


Teorema 5.12 (Representacin de Riesz). C[0, 1]

= NBV [0, 1].


Demostracin. Sea
: NBV [0, 1] C[0, 1]

la aplicacin denida como


(g) = T
g
.
Ya hemos visto que es lineal, continua y que
|(g)| = |T
g
| |g|
v
.
114 5. ESPACIOS DUALES Y OPERADORES TRASPUESTOS
Por otro lado, si T C[0, 1]

hemos visto que existe g BV [0, 1] tal


que T
g
= T y que existe una nica g NBV [0, 1] tal que T
gg
= 0, es
decir T
g
= T
g
= T. Por lo tanto es sobreyectiva y una isometra ya
que
|T| = |( g)| | g|
v
|g|
v
= |T|.

Los duales de los L


p
[0, 1]
Utilizamos en esta seccin nuestros conocimientos de la Teora In-
tegral de Lebesgue para estudiar los duales de L
p
[0, 1].
Teorema 5.13. Sea 1 p, q con
1
p
+
1
q
= 1. Dado g L
q
[0, 1]
sea T
g
: L
p
[0, 1] K la aplicacin denida como
T
g
(f) =
_
1
0
fgd para todo f L
p
[0, 1].
Entonces T
g
(L
p
[0, 1]

y |T
g
| = |g|
q
. Por lo tanto la aplicacin
: L
q
[0, 1] (L
p
[0, 1])

dada por (g) = T


g
es una isometra (an no armamos que sea so-
breyectiva).
Demostracin. La demostracin es bastante similar a la de los
duales de los
p
. Los detalles se pueden ver por ejemplo en [31, Theorem
14.1].
Se puede probar adems que en el caso 1 p < la isometra del
teorema anterior es sobreyectiva y por lo tanto (L
p
[0, 1])

= L
q
[0, 1]. La
demostracin ms habitual de este hecho utiliza el Teorema de Radon-
Nikodym, un resultado profundo de Teora de la Medida (o de la Teora
de la Integral de Lebesgue) que preferimos no tener que exponer. Los
alumnos interesados pueden leer una demostracin laboriosa pero ele-
mental (sin Radon-Nikodym) de la sobreyectividad de en [31, Theo-
rem 14.3].
Traspuesto de un operador
Relacionado con la idea de espacio dual est la nocin de traspuesto
de un operador.
5. ESPACIOS DUALES Y OPERADORES TRASPUESTOS 115
Definicin 5.14. Sean X, Y espacios normados, y sea T : X
Y un operador. Denimos el traspuesto de T, y lo denotamos T

, como
el operador
T

: Y

X

denido por
T

(y

)(x) = y

(T(x)).
Para todo x B
X
, y

B
Y
se tiene que
[T

(y

)(x)[ = [y

(T(x)[ |T(x)| |T|


de donde se obtiene que T

est bien denido.Claramente es lineal y


de la misma desigualdad de arriba se sigue que T

es continuo y
|T

| |T|
Para obtener la igualdad de las normas podemos tomar supremos
en la expresin de arriba, o considerar T

= (T

: X

Y

. Es
fcil ver que
T

(J(x)) = T(x),
(esto se puede proponer como ejercicio) por lo que
|T| |T

| |T

| |T|
de donde se sigue la igualdad de todas las normas involucradas.
La notacin de traspuesto de un operador se justica con el siguiente
ejemplo que proponemos como ejercicio.
Ejercicio 5.1. Sea X = K
n
, Y = K
m
. Consideramos en ambos
espacios por ejemplo las bases cannicas. Sabemos que todo operador T :
X Y se representa de forma nica (respecto de las bases elegidas)
como una matriz A = (a
ij
) donde para todo 1 j n
T(e
j
) =
m

i=1
a
ij
e
i
Entonces si consideramos tambin las bases cannicas en los espacios
duales (K
n
)

= K
n
y (K
m
)

= K
n
demostrar que la matriz traspuesta
A
T
es la matriz que representa respecto de esas bases al operador T

:
Y

X

Ejercicio 5.2. (T + S)

= T

+ S

, (T S)

= S

Definicin 5.15. Sean X, Y espacios de Banach. Un operador


T : X Y se dice que es un isomorsmo inyectivo si existen dos
constantes c, C > 0 tales que para todo x X
c|x| |T(x)| C|T(x)|.
116 5. ESPACIOS DUALES Y OPERADORES TRASPUESTOS
Ejercicio 5.3. Sean X, Y espacios de Banach. Si T : X Y
isomorsmo inyectivo entonces T(X) Y es un subespacio cerrado y
T(X) y X son isomrfos.
Proposicin 5.16. Sean X, Y espacios de Banach, T : X Y
un operador. Entonces T es un isomorsmo inyectivo si y slo si T

es sobre. Anlogamente, T es sobre si y slo si T

es un isomorsmo
inyectivo.
Demostracin. Supongamos que T es un isomorsmo. Entonces
T(X) Y es un subespacio cerrado isomorfo a X. Por tanto T(X)

es isomorfo a X

. Para ver que T

: Y

X

es sobreyectiva, dado
x

podemos considerar el elemento de e

T(X)

dado por
e

(T(x)) = x

(x). Extendemos e

por Hahn-Banach a un elemento


y

Y

con |y

| = |e

|. Sea entonces T

: Y

X

. Se tiene que,
para todo x X
T

(y

)(x) = y

T(x) = e

T(x) = x

(x)
y por tanto T

(y

) = x

.
Recprocamente, si T

: Y

X

es sobreyectivo, para todo


x X, utilizando el Teorema de la Aplicacin Abierta se tiene que
|x| = sup
x

B
X

|x

(x)| sup
y

B
Y

|T

(y

)(x)| =
= sup
y

B
Y

|y

(T(x))| |||T||x|
y esta es la condicin que necesitamos para que T sea un isomorsmo
inyectivo.
La demostracin de la segunda mitad del teorema es anloga y se
deja como ejercicio.
Prcticas sugeridas
Ejercicio 5.4. Sean X, Y espacios de Banach, T : X Y un
operador. Entonces T(X) es denso si y slo si T

es inyectivo.
Ejercicio 5.5. Demostrar que la identidad T :
1

2
es inyec-
tivo, pero T

no es sobre (porque
2
es separable y

no lo es).
Ejercicio 5.6. Demostrar que si 1 p < y
1
p
+
1
q
= 1 entonces
L

p
[0, 1] = L
q
[0, 1].
5. ESPACIOS DUALES Y OPERADORES TRASPUESTOS 117
Ejercicio 5.7. Probar que | |
v
es una norma en BV [0, 1].
CAPTULO 6
Topologas dbil y dbil

Como ya dijimos al comentar el Teorema de Hahn-Banach, dado


un espacio normado (completo o no) X, dicho teorema nos garantiza
la existencia de un dual X

con una estructura sucientemente ri-


ca. Resulta entonces que el estudio de X

, y de X con la topologa
inducida por X

(la topologa dbil que denimos en este captulo)


tienen gran importancia en muchos resultados fundamentales del Anli-
sis Funcional. Con razonamientos similares se puede estudiar en X

la
topologa inducida por X (la topologa dbil

que denimos ms ade-


lante) y veremos que sta tambin tiene mltiples aplicaciones.
Adems de las deniciones y propiedades bsicas de las topologas
dbil y dbil

, estudiamos en este captulo los duales de X y X

con
las topologas dbil y dbil

respectivamente. Tambin presentamos


en Teorema de Alaoglu y el de Goldstine, y un breve estudio de la
reexividad en espacios de Banach.
Opcionalmente se podran estudiar tambin el Teorema de Eberlein
y el Teorema de Mazur acerca de la coincidencia de las clausuras en
las topologa dbil y de la norma para espacios convexos. Este ltimo
teorema resulta adems una bonita aplicacin del Teorema de Hahn-
Banch, al igual que el Teorema de Goldstine.
Este captulo tal y como lo hemos planteado es algo ms topolgi-
co que los anteriores, aunque los nicos conocimientos topolgicos real-
mente necesarios son la nocin de red, la misma nocin de topologa y
la de base de una topologa. Si los alumnos no estuvieran en posesin
de estas nociones topolgicas bsicas, se podra plantear el estudio de
este captulo como se hace en [31], deniendo tan slo la convergen-
cia dbil y dbil

de sucesiones. Con eso es suciente para probar, por


ejemplo, una versin dbil del Teorema de Alaouglu-Bourbaki, que dice
que la bola unidad del dual de un espacio separable es dbil

secuen-
cialmente compacta. Tambin se puede probar el Teorema de Eberlein,
la propiedad de Schur de
1
y varias aplicaciones interesantes de la
convergencia dbil y dbil

.
119
120 6. TOPOLOGAS DBIL Y DBIL

Nosotros hemos basado nuestra presentacin de este captulo sobre


todo en [15] y [24].
La topologa dbil
Definicin 6.1. Sea X un espacio normado con dual X

. Se dene
la topologa dbil de X como la topologa generada por la siguiente base
de entornos: dados x
0
X, n N, x

1
, . . . , x

n
X

, y > 0 un entorno
dbil de x
0
viene denido por
W(x
0
; x

1
, . . . , x

n
; ) = x X tales que [x

i
(x x
0
)[ <
para todo 1 i n
Veamos que la topologa dbil es separada (Hausdor). Dados x ,=
y X, |xy| , = 0 por lo que el Teorema de Hahn-Banach nos garantiza
que existe x

tal que
x

(x y) > 0,
es decir, x

(x) x

(y) > 0. Si = x

(x) x

(y) entonces
x W(x; x

;

3
), y W(y; x

;

3
) y W(x; x

;

3
) W(y; x

;

3
) =
La topologa dbil es una topologa vectorial, es decir, la suma y el
producto son continuos. Vemoslo para la suma, el producto es similar.
Sean x, y X. Sea W(x+y; x

1
, . . . , x

n
; ) un entorno de x+y. Entonces
tomando los entornos W(x; x

1
, . . . , x

n
;

2
) y W(y; x

1
, . . . , x

n
;

2
) se tiene
que W(x; x

1
, . . . , x

n
;

2
) +W(y; x

1
, . . . , x

n
;

2
) W(x +y; x

1
, . . . , x

n
; ),
lo que prueba que la suma es continua.
Ntese que si X tiene dimensin innita entonces los entornos d-
biles (por ejemplo del 0) son bastante grandes. Por ejemplo, un en-
torno W(0; x

1
, . . . , x

n
; ) contiene al subespacio

n
i=1
ker x

i
,
un subespacio de codimensin nita. Y eso es muy grande.
Como consecuencia de este comentario se tiene la siguiente proposi-
cin.
Teorema 6.2. Sea X un espacio de Banach. La topologa dbil
coincide con la topologa de la norma si y slo si dimX < .
Demostracin. Supongamos que la topologa dbil coincide con
la topologa de la norma. Entonces la bola unidad abierta U
X
= x
X tales que |x| < 1 es abierta para la topologa dbil. Si X tuviera
6. TOPOLOGAS DBIL Y DBIL

121
dimensin innita entonces por el comentario anterior existira un sube-
spacio no trivial dentro de U
X
, algo claramente imposible.
Recprocamente, supongamos que dimX = n < . Puesto que ya
hemos visto que en dimensin nita todas las normas son equivalentes,
X es isomorfo a
n

. Veamos ahora que la topologa dbil y la topologa


de la norma coinciden sobre X.
Si U X es un abierto dbil, ya hemos visto que U es abierto en
la topologa de la norma.
Por otro lado, si U X es un abierto en la topologa de la norma,
entonces tambin U
n

es abierto (en la topologa de la norma ).


Sea x
0
U. Existe r > 0 tal que B(x
0
, r) U. Pero
B(x
0
, r) = W(x
0
; e

1
, . . . , e

n
; r)
y por tanto U es un abierto dbil. (Ntese que estamos usando im-
plcitamente que puesto que X es isomorfo a
n

, se tiene que e

i
X

(1 i n).
De hecho podemos probar an ms, que la topologa dbil es metriz-
able si y slo si X tiene dimensin nita. Para ello necesitamos previ-
amente un lema algebraico que tendremos ocasin de utilizar en ms
de una ocasin.
Lema 6.3. Sea X un espacio vectorial y sean f, g
1
, . . . , g
n
formas
lineales sobre X tales que

n
i=1
ker g
i
ker f
Entonces f es una combinacin lineal de los g
i
s.
Demostracin. Razonamos por induccin sobre n. Para n = 1 el
lema es fcil y ya lo probamos en el Captulo 4. Supongamos que ya lo
hemos probado para k n 1. Entonces dados f, g
1
, . . . , g
n
como en
el enunciado del teorema aplicamos la hiptesis de induccin a
f
[
ker g
n
, g
1[
ker g
n
, . . . , g
n1[
ker g
n
y deducimos que, sobre el ker g
n
,
f =
n1

i=1
a
i
g
i
Por tanto f

n1
i=1
a
i
g
i
se anula en ker g
n
, es decir
ker g
n
ker
_
f
n1

i=1
a
i
g
i
_
122 6. TOPOLOGAS DBIL Y DBIL

y por tanto del caso n = 1 obtenemos que


f
n1

i=1
a
i
g
i
= a
n
g
n
de donde se obtiene lo pedido.
Ahora podemos probar que la topologa dbil nunca es metrizable
si X es de dimensin innita.
Proposicin 6.4. Sea X un espacio normado. En ese caso la
topologa dbil sobre X es metrizable si y slo si X es de dimensin
nita.
Demostracin. Ya vimos que si X es de dimensin nita entonces
la topologa dbil coincide con la topologa de la norma, y por tanto es
metrizable.
Recprocamente, supongamos que la topologa dbil es metrizable.
Puesto que las topologas metrizables satisfacen el I Axioma de Nu-
merabilidad, debe existir una sucesin (x

n
)
n
X

tal que para todo


entorno U de 0 existe un racional Q > 0 y un natural n(U) N
tales que U contiene a W(0; x

1
, . . . , x

n(U)
; ). Cada x

genera el
entorno dbil W(0; x

; 1) que a su vez contiene uno de los entornos


W(0; x

1
, . . . , x

n(W(0;x

;1))
; ). Pero por el Lema 6.3, esto obliga a que x

sea una combinacin lineal de x

1
, . . . , x

n(W(0;x

;1))
. Esto implicara que
X

tendra una base algebraica numerable, y puesto que X

es un es-
pacio de Banach, esto slo puede ocurrir si X

, y por tanto X tiene


dimensin nita.
De hecho se tiene que si dimX < entonces existe una nica
topologa vectorial separada sobre X (obviamente normable), aunque
no probaremos ese resultado en este curso.
Proposicin 6.5. Sea X un espacio normado. Una red (o una
sucesin) (x
i
)
iI
X tiende dbilmente a x
0
X si y slo si para todo
x

lm
i
x

(x
i
) = x

(x
0
)
Demostracin. En efecto, supongamos que lm
i
x

(x
i
) = x

(x
0
)
para todo x

. Si W es un abierto de la topologa dbil que


contiene a x
0
, entonces existe un W(x
0
; x

1
, . . . , x

n
; ) W. Puesto
que, por hiptesis lm
i
x

j
(x
i
) = x

j
(x
0
) para todo 1 j n, existe
i
0
I tal que para todo i i
0
(donde denota el orden en I)
[x

j
(x
i
) x

j
(x
0
)[ <
6. TOPOLOGAS DBIL Y DBIL

123
y por tanto para todo i i
0
se tiene que x
i
W(x
0
; x

1
, . . . , x

n
; ) W,
es decir (x
i
) x
0
en la topologa dbil.
Recprocamente, supongamos que (x
i
) x
0
en la topologa dbil,
y sea x

. Para todo > 0, consideramos el entorno W(x


0
; x

; ).
Entonces existe i
0
I tal que para todo i i
0
se tiene que x
i

W(x
0
; x

; ), lo que implica que


[x

(x
i
) x

(x
0
[ <
y de aqu se sigue que
lm
i
x

(x
i
) = x

(x
0
).

Proposicin 6.6. Sea X un espacio normado y x

: X K una
forma lineal. Entonces x

es continua para la topologa de la norma de


X si y slo si x

es continua para la topologa dbil de X.


Demostracin. Las bases de entornos utilizadas para denir la
topologa dbil son abiertos en la topologa de la norma (porque son

n
i=1
(x

i
)
1
(A) con A abierto y x

i
norma-continuos) y de ah se sigue
que los conjuntos abiertos en la topologa dbil son | |abiertos, es
decir, la topologa de la norma es ms fuerte (tiene ms abiertos) que
la topologa dbil (y de ah el nombre). Por tanto si x

: X K es
continuo para la topologa dbil, tambin lo es para la topologa de la
norma.
Recprocamente, si x

, para todo > 0 el conjunto (x

)
1
(, )
es un abierto dbil, y por tanto x

es dbilmente continua.
Una vez probado esto podemos probar el siguiente teorema
Teorema 6.7. Sea T : X Y una aplicacin entre dos espacios
normados X e Y . Entonces T es norma-norma continua si y slo si T
es dbil-dbil continua.
Demostracin. Supongamos que T es norma-norma continua.
Entonces para todo y

Y

se tiene que y

T : X K es nor-
ma continua; por tanto y

T X

y del resultado anterior se sigue


que y

T es dbil continua, y de aqu se sigue que T es dbil-dbil


continua.
Recprocamente, si T es dbil-dbil continua, entonces para todo
y

Y

se tiene que y

T es dbil continua. Por tanto y

T es
124 6. TOPOLOGAS DBIL Y DBIL

norma continua. Ahora el Teorema 4.9, consecuencia del Principio de


Acotacin Uniforme, nos dice que T es norma-norma continua.
Probamos a continuacin el Teorema de Mazur.
Teorema 6.8 (Mazur). Si K es un conjunto convexo del espacio
normado X, entonces la clausura dbil de K coincide con su clausura
en norma.
Demostracin. Puesto que la topologa de la norma es ms fuerte,
se tiene automticamente que
K
||
K
w
Para el otro contenido, supongamos que existe
x
0
K
w
K
||
Entonces, puesto que K
||
es convexo, por el teorema de Hahn-Banach
existen x

0
X

y < R tales que


(9) sup
xK

'(x

0
(x)) < '(x

0
(x
0
))
Pero puesto que x
0
K
w
, se tiene que existe una red (x
i
)
iI
K
tal que
x
0
= lm
i
x
i
ya vimos anteriormente que esto implicaba que
x

(x
0
) = lm
i
x

(x
i
)
pero esto es imposible ya que, segn (9),
lm
i
'(x

(x
i
)) < '(x

0
(x
0
))

El siguiente corolario, til en ocasiones, se puede proponer como


ejercicio
Corolario 6.9. Sea (x
n
) X una sucesin que tiende dbilmente
a 0. Entonces existe una sucesin (
n
) de combinaciones convexas de
los (x
n
) tal que (
n
) tiende en norma a 0.
Aunque la topologa dbil no se puede caracterizar en general por
sucesiones, muchas de las aplicaciones de la topologa dbil vendrn
6. TOPOLOGAS DBIL Y DBIL

125
precisamente a travs del estudio de la convergencia dbil de suce-
siones. Puesto que la topologa de la norma es mas fuerte, es ms fcil
para una sucesin converger en la topologa dbil, y la mayora de las
aplicaciones de la topologa dbil vendrn precisamente de conseguir
que una sucesin que no converge en norma s lo haga dbilmente, y
buscar las consecuencias que se puedan extraer de eso.
Veamos esto con un poco de detalle
Sea (x
n
) X una sucesin tal que x
n
||
x. Entonces x
n
w
x.
Demostracin: Para todo x

,
[x

(x
n
) x

(x)[ = [x

(x
n
x)[ |x
n
x| 0
Sea la sucesin (e
n
)
2
. Entonces |e
n
| = 1 para todo n N
pero e
n
w
0.
Demostracin: La primera armacin es evidente. Para la
segunda, sea a

2
. Ya vimos que entonces a = (a
n
)
2
y
a(e
n
) =

m
a
m

nm
= a
n
y puesto que (a
n
)
2
, se sigue que
lm
n
a(e
n
) = lm
n
a
n
= 0
Puede ocurrir en cambio que en un espacio normado X la conver-
gencia dbil de sucesiones coincida con la convergencia en norma de
sucesiones. Schur prob que este es el caso si X =
1
por lo que a los
espacios que verican esta propiedad se les llama espacios de Schur.
Teorema 6.10 (Schur). En
1
una sucesin (x
n
)
n
tiende dbil-
mente a x si y slo si (x
n
) tiende en norma a x.
Demostracin. Hacemos la demostracin en el caso real. Se adap-
ta al caso complejo de forma estandar y dejamos esto como ejercicio.
Veamos primero que si x
n
tiende dbilmente a 0 entonces tambin
converge a 0 en norma. Si no fuera as, tomando subsucesiones podemos
suponer la existencia de > 0 tal que para todo n N,
|x
n
| =

i=1
[x
i
n
[ > .
Sea N
1
tal que

i=N
1
+1
[x
i
1
[ <

5
.
126 6. TOPOLOGAS DBIL Y DBIL

Entonces
N
1

i=1
[x
i
1
[
4
5
,
y por tanto
N
1

i=1

i
1
x
i
1

4
5
,
donde
i
1
= sgn(x
i
1
). Obsrvese que para una eleccin arbitraria de
signos
i
= 1 tales que
i
=
i
1
si 1 i N
1
se tiene que

i=1

i
x
i
1

N
1

i=1

i
1
x
i
1
+

i=N
1
+1

i
x
i
1

N
1

i=1

i
1
x
i
1

i=N
1
+1
[x
i
1
[
4
5


5
=
3
5
.
A continuacin, puesto que (x
n
) tiende dbilmente a 0, considerando
todos los elementos
N
1
= (1,
(N
1
)
. . . , 1, 0, 0, . . .)

= (
1
)

elegimos
n
2
tal que
N
1

i=1
[x
i
n
2
[ <

5
.
Seguidamente consideramos N
2
> N
1
tal que

i=N
2
+1
[x
i
n
2
[ <

5
.
Entonces
N
2

i=1
[x
i
n
2
[
4
5
,
y por tanto, dados los signos
N
1
+1
= sgn(x
N
1
+1
n
2
), ,
N
2
= sgn(x
N
2
n
2
)
tenemos que
N
2

i=N
1
+1

i
x
i
n
2
=
N
2

i=N
1
+1
[x
i
n
2
[ =
N
2

i=1
[x
i
n
2
[
N
1

i=1
[x
i
n
2
[
4
5


5
=
3
5
.
De nuevo ntese que para una eleccin arbitraria de signos
i
= 1
tales que
i
=
i
1
si 1 i N
1
y
i
=
i
n
2
si N
1
i N
2
se tiene que

i=1

i
x
i
n
2

N
2

i=N
1
+1

i
x
i
n
2

N
1

i=1

i
x
i
n
2
+

i=N
2
+1

i
x
i
1

6. TOPOLOGAS DBIL Y DBIL

127

N
2

i=N
1
+1

i
x
i
n
2

N
1

i=1

i
x
i
n
2

i=N
2
+1

i
x
i
1

N
2

i=N
1
+1

i
x
i
n
2

N
1

i=1
[x
i
n
2
[

i=N
2
+1
[x
i
1
[

3
5


5


5
=

5
Continuando la construccin por induccin, obtenemos dos sucesiones
1 < n
2
< n
3
< y N
1
< N
2
< N
3
< y un vector = (
i
)

denido como

i
= sgn(x
i
n
k
) si N
k1
< i N
k
de manera que, para todo k N,
(x
n
k
)

5
en contradiccin con el hecho de que x
n
tienda dbilmente a 0.
Si ahora x
n
tiende dbilmente a x razonamos anlogamente con la
sucesin (x
n
x).
Hay varias demostraciones de este resultado fundamental. Hemos
reproducido la de [24, Thm 99] que es elemental y utiliza el mtodo de
la joroba deslizante que tiene aplicaciones en otros contextos. En [13,
Prop V.5.2] se puede encontrar una demostracin ms topolgica, que
utiliza el Teorema de Baire, el Teorema de Alaoglu y el hecho de que
(B

, w

) es metrizable (por ser


1
separable). En [15, p. 85] aparece
otra demostracin que utiliza el Lema de Philips y una descripcin del
dual de

.
La topologa dbil

Una vez estudiados algunos de los resultados bsicos ms impor-


tantes referidos a la topologa dbil, pasamos a estudiar la topologa
dbil

sobre un espacio dual X

.
Definicin 6.11. Sea X un espacio normado con dual X

. La
topologa dbil

en X

es la topologa generada por la siguiente base


de entornos: dados x

0
X

, n N, x
1
, . . . , x
n
X, y > 0 un
entorno dbil

de x

0
viene denido por
W(x

0
; x
1
, . . . , x
n
; ) = x

tales que
[J(x
i
)(x

0
)[ = [(x

0
)(x
i
)[ < para todo 1 i n
128 6. TOPOLOGAS DBIL Y DBIL

Dado un espacio dual X

, es obvio que la topologa dbil

sobre
X

es en general menos fuerte (tiene menos entornos) que la topologa


dbil sobre X

, puesto que la topologa dbil es la generada por todos


las formas x

, mientras que la dbil

es la generada tan slo


por las formas J(x) X

, con x X.
Veamos que a pesar de ello la topologa dbil

tiene sucientes en-


tornos para ser Hausdor.
Dados x

,= y

, existe x X tal que


(x

)(x) > 0,
y ahora podemos razonar como lo hacamos en el caso de la topologa
dbil.
De nuevo es fcil ver que la topologa dbil

es una topologa vec-


torial.
Siguiendo exactamente la misma demostracin que para el caso de la
topologa dbil, mutatis mutandi se demuestra que una red (o sucesin)
(x

i
)
iI
X

tiende dbil

a x

0
X

si y slo si para todo x X


lm
i
J(x)(x

i
) lm
i
x

i
(x) = x

0
(x)
Veamos a continuacin que al igual que el dual de X con la topologa
dbil es X

, tambin el dual de X

con la topologa dbil

es X.
Teorema 6.12. Sea X un espacio de Banach y sea x

una
forma acotada tal que x

: X

K es continuo para la topologa


dbil

. Entonces existe x X tal que J(x) = x

(o, dicho ms breve-


mente, x

X). Recprocamente, para todo x X se tiene que


x (X

, w

.
Demostracin. Si x

es continuo para la topologa dbil

, en-
tonces existe un entorno dbil

del origen W := W(0; x


1
, . . . , x
n
; ) tal
que [x

(x

)[ < 1 para todo x

W. Sea x

tal que J(x


i
)(x

) =
x

(x
i
) = 0 para todo 1 i n. Entonces para todo K, x

W
y por tanto [x

(x

)[ < 1 lo que implica que x

(x

) = 0. Por tanto

n
i=1
ker(J(x
i
)) ker x

Ahora el Lema 6.3 nos garantiza que


x

i
a
i
J(x
i
)
y puesto que J(X) es un subespacio vectorial, se sigue que x

J(X).
La otra implicacin es inmediata.
6. TOPOLOGAS DBIL Y DBIL

129
Enunciamos y probamos a continuacin los Teoremas de Goldstine
y Alaoglu.
Teorema 6.13 (Alaoglu). Para todo espacio normado X, B
X
es
dbil

compacto.
Demostracin. Por el Teorema de Tychono, el espacio [1, 1]
B
X
de las funciones f : B
X
[1, 1] con la topologa producto (la
topologa de la convergencia puntual) es un espacio compacto. Dado
un x

B
X
, podemos identicar de forma natural x

con un punto de
[1, 1]
B
X
, de forma que podemos identicar B
X
con un subconjunto
de [1, 1]
B
X
. Puesto que la topologa dbil

es la topologa de la conver-
gencia puntual, esta identicacin nos permite ver a (B
X
,
w
) como
un subconjunto de [1, 1]
B
X
con la topologa producto, de manera que
para probar el teorema simplemente tenemos que probar que B
X
es
un cerrado de [1, 1]
B
X
.
Sea (x

i
)
iI
B
X
una red convergente puntualmente (dbil

) a
f [1, 1]
B
X
. Veamos que f es lineal sobre B
X
: Para todos , K,
x, y X tales que x + y B
X
, se tiene
f(x + y) = lm
i
x

i
(x + y) =
= lm
i
x

i
(x) + lm
i
x

i
(y) = f(x) + f(y),
Adems f est acotada en B
X
y |f| 1 puesto que f(B
X
) [1, 1],
por lo que f B
X
.
Teorema 6.14 (Goldstine). Sea X un espacio normado. Entonces
B
X
es dbil

densa en B
X
y por lo tanto X es dbil

-denso en X

.
Demostracin. Sea x

y supongamos que x

, B
X
w

.
Aplicamos el Teorema de Hahn-Banach. Para ello observamos que
1. (X

, w

) es un espacio localmente convexo y (X

, w

= X

2. B
X
w

(X

, w

) es un conjunto dbil

cerrado y convexo.
3. x

es dbil

cerrado y convexo.
Por lo tanto podemos aplicar la versin geomtrica del Teorema de
Hahn-Banach para obtener un elemento x

tal que
sup
y

B
X
w

(y

) < x

(x

)
adems podemos suponer sin perdida de generalidad que |x

| = 1.
En ese caso
sup
y

B
X
w

(y

) |x

| = 1
130 6. TOPOLOGAS DBIL Y DBIL

por lo que
x

(x

) > 1,
luego |x

| > 1 y por lo tanto


B
X
B
X
w

.
La ltima armacin es ahora fcil.
Corolario 6.15. X es reexivo si y slo si B
X
es dbilmente
compacta.
Demostracin. Supongamos que X es reexivo. Entonces X =
X

y B
X
= B
X
, y esta es dbil

compacta. Puesto que la topologa


dbil y la dbil

coinciden en los reexivos, tenemos que B


X
es dbil
compacta.
Recprocamente, si B
X
es dbil compacta, entonces es dbil

cerrada
en X

. Puesto que la clausura dbil

de B
X
en B
X
es B
X
, tenemos
que B
X
= B
X
y por tanto X = X

.
Si bien la topologa dbil

sobre X

nunca es metrizable (si X es


innito-dimensional), s ocurre a menudo que el conjunto compacto
(B
X
, w

) es metrizable, y a menudo se puede sacar partido de ello.


Veamoslo
Teorema 6.16. Un espacio de Banach X es separable si y slo si
(B
X
, w

) es metrizable.
Demostracin. Sea (x
n
) S
X
una sucesin densa en S
X
, y de-
namos una funcin d : B
X
B
X
[0, ) mediante
d(x

, y

) =

n=1
[x

(x
n
) y

(x
n
)[
2
n
.
Claramente la serie es siempre convergente y por tanto d est bien
denida. Es fcil ver que d es una distancia en B
X
. Veamos entonces
que la aplicacin identidad
Id : (B
X
, w

) (B
X
, d)
es un homeomorsmo, lo que automticamente implicar que (B
X
, w

)
es metrizable.
Sea x

0
B
X
y sea
U = x

B
X
tales que d(x

, x

0
) <
un entorno abierto de x

0
(B
X
, d). Sea n
0
N un natural tal que
1
2
n
0
=

i=1
1
2
n
0
i
<

4
.
6. TOPOLOGAS DBIL Y DBIL

131
Sea entonces W(x
0
; x
1
, . . . , x
n
0
; ), un entorno de x

0
(B
X
, w

).
Para todo x

W(x
0
; x
1
, . . . , x
n
0
; ) se tiene que
d(x

, x

0
) =

n=1
[x

0
(x
n
) x

(x
n
)[
2
n
=
=
n
0

n=1
[x

0
(x
n
) x

(x
n
)[
2
n
+

n=n
0
+1
[x

0
(x
n
) x

(x
n
)[
2
n


2
+

n=1
1
2
n
0
n+1
< ,
y por tanto W(x
0
; x
1
, . . . , x
n
0
; ) U y se tiene que
Id : (B
X
, w

) (B
X
, d)
es continua. Al ser (B
X
, w

) compacto, (B
X
, d) Hausdor e Id biyec-
tiva se tiene automticamente que Id
1
: (B
X
, d) (B
X
, w

) es
continua, es decir Id es un homeomorsmo y por tanto (B
X
, w

) es
metrizable.
Recprocamente, si (B
X
, w

) es metrizable entonces verica el I


Axioma de Numerabilidad y por tanto sabemos que existe una sucesin
(U
n
)
n
de entornos del origen de (B
X
, w

) tales que

n=1
U
n
= 0. Por
la denicin de la topologa dbil

, para cada n N existe


n
y un
conjunto nito F
n
= x
n
1
, . . . , x
n
m
X tal que
W(0; x
n
1
, . . . , x
n
m
;
n
) U
n
Sea F =

n=1
F
n
. As denido F es contable. Adems (F

es la
envoltura lineal y cerrada de F y por tanto es separable. Pero si x F

quiere decir que para todo n 1 y para todo x F


n

|x

|
(x)

<
n
.
Por tanto
x

|x

|
U
n
para todo n 1, de donde x

= 0. Es decir
F

= 0 y por tanto (F

= X, por lo que X es separable.


Corolario 6.17. [Banach 1932] Sea X un espacio de Banach sep-
arable. Entonces toda sucesin acotada de X

tiene una subsucesin


dbil

convergente.
Demostracin. Si X es separable, (B
X
, w

) es metrizable, por
lo que la compacidad se caracteriza por sucesiones.
Veamos dos aplicaciones de este resultado. La primera es al anlisis
de Fourier.
132 6. TOPOLOGAS DBIL Y DBIL

Teorema 6.18. Sea (


n
)
nZ
una sucesin indexada por Z. Para
cada m N denamos la funcin s
m
: [, ] C como
s
m
(t) =
m

n=m

n
e
nt
y sea
a
m
=

m1
j=0
s
j
m
.
Sea 1 < q tal que la sucesin (a
m
) L
q
[, ] es acotada.
Entonces existe y L
q
[, ] tal que para todo n Z

n
=
1
2
_

y(t)e
nt
dt,
es decir

n=

n
e
nt
es la serie de Fourier de y.
Demostracin. Recordemos la notacin: Para todo x L
1
[, ]
y n Z, denimos el n-simo coeciente de Fourier de x como
x(n) =
1
2
_

x(t)e
nt
dt.
Resulta claro de las deniciones que para todo n Z y para todo
m > [n[ se tiene
a
m
(n) =
m[n[
m

n
.
Sea q como en la hiptesis y sea 1 p < de manera que
1
p
+
1
q
= 1.
Puesto que por hiptesis (a
m
)
m
L
q
= (L
p
)

es una sucesin aco-


tada, podemos aplicar el Corolario 6.17 y obtenemos una subsucesin
(a
m
j
) dbil

convergente a y L
q
. Para todo n Z sea x
n
(t) =
e
nt
2
.
Entonces para todos m, n
a
m
(x
n
) = a
m
(n), y y(x
n
) = y(n)
y por tanto
y(n) = lm
j
a
m
j
(n) = lm
j
m
j
[n[
m
j

n
=
n
.

6. TOPOLOGAS DBIL Y DBIL

133
Otra aplicacin que puede ser interesante estudiar es el llamado
Principio de seleccion de Helly. Se puede ver una exposicin sencilla
por ejemplo en ( [31, p. 273 y ss]).
Finalmente, de manera opcional se puede probar el Teorema de
Eberlein, que nos dice que, aunque la topologa dbil no se pueda car-
acterizar por sucesiones, la compacidad dbil s se puede caracterizar
por sucesiones.
Teorema 6.19 (Eberlein). Sea X un espacio de Banach y A X.
Entonces A es (relativamente) dbilmente compacto si y slo si A es
secuencialmente (relativamente) compacto.
Demostracin. En [15, p. 18] se puede ver una demostracin que
requiere una cierta dosis de topologa. Una demostracin totalmente
elemental usando tan slo la convergencia dbil de sucesiones de un
enunciado algo ms sencillo X es reexivo si y slo si toda sucesin
acotada tiene una subsucesin dbilmente convergente se puede ver en
[31, p. 288].
Prcticas sugeridas
Ejercicio 6.1. Probar que una sucesin (f
n
) C[0, 1] converge
dbilmente a f si y slo si (f
n
) est acotada y para todo t [0, 1] f
n
(t)
tiende a f(t).
Ejercicio 6.2. Dado un espacio de Banach separable X, encontrar
T L(
2
; X) tal que T(
2
) sea denso en X. Sugerencia: Sea (y
n
) X
una sucesin densa y sea T :
2
X dado por
T(a) =

n
a
n
y
n
2
n
Ejercicio 6.3. X es reexivo si y slo si X

es reexivo
Ejercicio 6.4. Si X es reexivo e Y es un subespacio vectorial
cerrado de X entonces Y es reexivo.
CAPTULO 7
Operadores compactos
En el estudio de las aplicaciones lineales entre espacios de Banach
(o simplemente normados) pronto se ve que muchos problemas que en
el caso de espacios nito-dimensionales tienen solucin resultan muy
difciles, o imposibles, de solucionar. Con frecuencia el problema es
esencialmente la no compacidad de la bola unidad. Buena parte de las
tcnicas del Anlisis Funcional van dirigidas a solucionar este problema.
Una de las formas de abordarlo es transformar la bola unidad mediante
un operador en un conjunto relativamente compacto. A los operadores
que cumplen esa condicin les llamamos operadores compactos.
Es claro que los operadores de rango nito son compactos. Adems
a menudo los operadores compactos son precisamente la clausura en el
espacio normado de los operadores de los operadores de rango nito.
Vemos tambin la propiedad de ideal de los operadores compactos,
as como el fundamental Teorema de Schauder, cuya demostracin con-
stituye una bonita aplicacin del Teorema de Ascoli-Arzela.
Para terminar el captulo, estudiamos la relacin de los operadores
compactos con los operadores completamente continuos.
Hemos basado la preparacin de este captulo en [31], [24] y [13].
Operadores compactos
Empezamos deniendo los operadores compactos.
Definicin 7.1. Sean X e Y espacios normados. Un operador
T : X Y se dice compacto si T(B
X
) es un conjunto compacto.
Denotaremos por /(X; Y ) al espacios de los operadores compactos de
X en Y con la norma heredada de L(X; Y ). Si X = Y escribiremos
/(X) en lugar de /(X; X).
Definicin 7.2. Sean X e Y espacios normados. Un operador T :
X Y se dice de rango nito si dim(Im(T)) < . Denotaremos
por T(X; Y ) al espacios de los operadores de rango nito de X en Y
con la norma heredada de L(X; Y ). Si X = Y escribiremos T(X) en
lugar de T(X; X).
135
136 7. OPERADORES COMPACTOS
Ntese primeramente que, como ya vimos, se sigue del Lema de
Riesz que la bola unidad de los espacios de dimensin innita no es
compacta, por lo que si dimX = , el operador identidad Id : X
X nunca es compacto. Tambin el Teorema de la Aplicacin Abierta nos
garantiza que los operadores sobreyectivos (sobre espacios de dimensin
innita) no son compactos.
Observacin 7.3. Si T S
X
es tal que T no alcanza su norma
en B
X
, entonces T(B
X
) = (1, 1), y tenemos que T L(X; R) pero
T(B
X
) no es compacto. Por tanto no se puede sustituir T(B
X
) por
T(B
X
) en la denicin de operador compacto.
Si se quiere un ejemplo explcito de esta situacin, considrese por
ejemplo la forma T : c
0
K denida por T(e
n
) =
1
2
n
. Entonces
|T| = 1
pero la norma no se alcanza en la bola de c
0
(se alcanza en la bola de

, por ejemplo en 11).


Proposicin 7.4. Sean X, Y espacios normados, T : X Y un
operador. Entonces
(i) T es compacto si y slo si dada cualquier sucesin acotada
(x
n
)
n
X, la sucesin T(x
n
)
n
tiene una subsucesin conver-
gente.
(ii) Si T es compacto, entonces T(B
X
) es un conjunto precom-
pacto. Recprocamente, si Y es Banach y T(B
X
) es precom-
pacto entonces T es compacto.
(iii) Si T es continuo y de rango nito entonces T es compacto y
Im(T) es cerrado en Y . Recprocamente, si X, Y son espacios
de Banach, T es compacto e Im(T) es cerrado automtica-
mente T es de rango nito.
Demostracin. (i) Sea T compacto y (x
n
) X una sucesin
tal que |x
n
| C para todo n N. Entonces T(
x
n
C
) T(B
X
) para
todo n N. Puesto que T(B
X
) es compacto, sabemos que existe una
subsucesin de T(
x
n
C
) que converge en T(B
X
) Y . Por tanto una
subsucesin de T(x
n
) converge en Y .
Recprocamente, para ver que T(B
X
) es compacto usamos la car-
acterizacin por sucesiones (puesto que estamos en un espacio metriz-
able). Sea (y
n
) T(B
X
). Entonces existe (x
n
) B
X
tal que
|y
n
T(x
n
)| <
1
n
7. OPERADORES COMPACTOS 137
Por hiptesis existe una subsucesin (x
n
j
)
j
(x
n
)
n
tal que T(x
n
j
)
j
converge a y Y . De ah se sigue con facilidad que la subsucesin
(y
n
j
)
j
(y
n
)
n
tambin converge a y, y por tanto y T(B
X
). Lo que
prueba que T(B
X
) es compacto.
(ii) Si T es compacto, entonces T(B
X
) es precompacto, y por tanto
T(B
X
) tambin lo es.
Recprocamente, si Y es Banach y T(B
X
) es precompacto, entonces
T(B
X
) es precompacto y completo, y por tanto compacto.
Probemos (iii): Si T es de rango nito entonces Im(T) es cerrado
en Y por ser un espacio de dimensin nita. Entonces T(B
X
) Im(T)
y T(B
X
) es un conjunto cerrado y acotado en un espacio de dimensin
nita, luego es compacto.
Recprocamente, sean X e Y espacios de Banach, T : X Y
compacto tal que T(X) es cerrado. Por ser T compacto, es continuo.
Adems T(X) es un espacio de Banach y T : X T(X) es un
operador sobre. Por el Teorema de la Aplicacin Abierta, T(U
X
) es un
abierto en T(X). Por tanto existe > 0 tal que
E := y T(X) tales que |y| < T(B
X
)
y puesto que T(X) es cerrado, tenemos que
E = y T(X) tales que |y| T(B
X
) T(X)
Puesto que T(B
X
) es compacto, tenemos que la bola cerrada de cen-
tro 0 y radio de T(X) es compacta. Y esto implica que dim(T(X)) <
.
Veamos a continuacin que los operadores compactos forman un
ideal de operadores cerrado en la norma usual de operadores
Teorema 7.5. Sean X, Y espacios normados. Entonces
1. Para todos T, S /(X; Y ), para todos , K, el operador
T + S : X Y
es compacto.
2. (Propiedad de ideal de operadores) Si E, F son espacios nor-
mados R : E X, S : Y F son operadores y T
/(X; Y ) entonces
S T R : E F
es compacto.
138 7. OPERADORES COMPACTOS
3. Si Y es un espacio de Banach, /(X; Y ) es un subespacio cer-
rado de L(X; Y )
Demostracin. 1. Es muy fcil ver que si T es compacto tambin
lo es T, por ejemplo usando la caracterizacin por sucesiones. De esta
misma forma demostramos que la suma de operadores compactos es
compacto: Sean T, S como en la hiptesis, y sea una sucesin (x
n
)
B
X
. Entonces existe una subsucesin (x
n
j
)
j
(x
n
) tal que T(x
n
j
)
converge. A su vez existe una subsubsucesin (x
n
j
k
)
k
(x
n
j
) tal que
S(x
n
j
k
) converge. Entonces
(T + S)(x
n
j
k
)
converge, lo que prueba que S + T es compacto.
2. Se puede demostrar tambin por sucesiones, pero lo haremos
directamente con la denicin. Notemos que
S T R(B
E
) = S T(R(B
E
)) S T(|R|B
X
) =
= S(T(|R|B
X
)) S(T(|R|B
X
))
Puesto que T(B
X
) es compacto, tambin lo es T(|R|B
X
) y puesto
que S es continua, transforma compactos en compactos. Por tanto
S T R(B
E
) es un cerrado contenido en un compacto, y por tanto
es compacto.
3. Sea (T
n
)
n
/(X; Y ) una sucesin que converge a T L(X; Y )
en la norma de operadores. Hemos de comprobar que T es compacto.
Por la Proposicin 7.4 basta comprobar que T(B
X
) es precompacto.
Sea > 0. Sea n N tal que
|T
n
T|

3
.
Puesto que T
n
es precompacto, existen x
1
, . . . , x
m
B
X
tales que
T
n
(B
X
)
m
i=1
B(T
n
(x
i
),

3
).
Entonces, dado x B
X
, sea x
j
tal que T
n
(x) B(T
n
(x
j
),

3
). As,
se tiene que
|T(x) T(x
j
)| |(T T
n
)(x)|+|T
n
(x) T
n
(x
j
)|+|(T
n
T)(x
j
)|
|T T
n
||x| +

3
+|T T
n
||x
j
| 3

3
=
y por tanto
T(B
X
)
m
i=1
B(T
n
(x
i
), ),
lo que implica que T es compacto.
7. OPERADORES COMPACTOS 139
Teorema 7.6 (Schauder). Sean X, Y espacios de Banach, T
L(X; Y ). Entonces T es compacto si y slo si T

es compacto.
Demostracin. Sea T : X Y un operador compacto. Sea
(y

n
)
n
B
Y
. Para todos y, z Y
[y

n
(y) y

n
(z)[ |y

n
||y z| |y z|
Sea K = T(B
X
). K es un espacio compacto metrizable y por lo anterior
el conjunto
y

n
; n N
es un conjunto de funciones equicontinuas uniformemente acotadas.
Entonces el Teorema de Ascoli-Arzela nos dice que existe una subsuce-
sin (y

n
j
)
jN
que converge uniformemente en K. Entonces, para todo
i, j N,
|T

(y

n
i
) T

(y

n
j
)| = sup
xB
X
[T

(y

n
i
y

n
j
)(x)[ =
= sup
xB
X
[(y

n
i
y

n
j
)T(x)[ sup
yK
[y

n
i
(y) y

n
j
(y)[
Puesto que la sucesin (y
n
j
)
jN
es uniformemente Cauchy en K,
se sigue que (T

(y

n
j
))
j
es una sucesin de Cauchy en X

. Por tanto,
puesto que X

es Banach, (T

(y

n
j
))
j
debe converger y de ah se sigue
que T

es compacto.
La otra implicacin es esencialmente la demostracin de que los
operadores compactos forman un ideal inyectivo de operadores, aunque
no presentaremos esa nocin a nuestros alumnos:
Supongamos que T

es compacto. Entonces T

= (T

tambin es
compacto por lo anterior. Puesto que T

J
X
= J
Y
T, la propiedad de
ideal de los operadores compactos nos garantiza que J
Y
T es compacto.
Esto quiere decir J
Y
(T(B
X
)) es precompacto. Puesto que J
Y
es una
isometra, se sigue que T(B
X
) es precompacto, y de aqu se sigue que
T es compacto.
Ejemplo 7.7. Ya hemos visto que todo operador de rango nito
es compacto, y puesto que los operadores compactos /(X; Y ) forman
un subespacio cerrado en el espacio de los operadores, se sigue que el
lmite (en la norma de operadores) de una sucesin de operadores de
rango nito es compacto. De hecho, en muchos casos (por ejemplo si
Y es un espacio con base de Schauder), todo operador compacto es el
lmite de una sucesin de operadores de rango nito. En 1932 Banach
pregunt si en un espacio separable todo operador compacto se podra
escribir como el lmite de una sucesin de operadores de rango nito, es
140 7. OPERADORES COMPACTOS
decir si /(X) = T(X). Ms tarde, en 1953 Grothendieck [23] deni
la Propiedad de Aproximacin de la siguiente forma
X tiene la Propiedad de Aproximacin si y slo si para todo espacio
de Banach Y , /(X; Y ) = T(X; Y )
Hubo que esperar a 1973 para que Eno [19] probara que no to-
do espacio de Banach tiene la Propiedad de Aproximacin, y diera
simultneamente una respuesta negativa a la pregunta de Banach.
Es fcil ver directamente que para los espacios de sucesiones sepa-
rables s se tiene que /(X) = T(X).
Proposicin 7.8. Sea X uno de los espacios c
0
,
p
(1 p < ).
Entonces /(X) = T(X).
Demostracin. Sea (e
n
)
nN
la base cannica habitual de estos
espacios. Dado n N, sea
n
: X X el operador que a cada
x = (x
i
)
iN
le asocia

n
(x) =
n

i=1
x
i
e
i
= (x
1
, . . . , x
n
, 0, 0, . . .).
Es inmediato ver que para cualquiera de los espacios mencionados
n
es continuo y que
lm
n

n
(x) = Id(x) = x.
Sea ahora T /(X). Claramente
n
T T(X) para todo n N.
Slo nos falta ver que
n
T converge a T en la norma de L(X). Esto
es lo mismo que ver que (
n
Id)(T(x)) converge a 0 uniformemente
en B
X
. Para probar eso es suciente probar que
n
Id tiende a 0
uniformemente en el compacto T(B
X
) y esto se sigue del Teorema de
Banach-Steinhaus.
Ejemplo 7.9. Sea T :
2

2
el operador dado por
T(x) =
_
x
n
2
n
_
n
.
Veamos que T es compacto, mientras que claramente no tiene rango
nito.
Para ver que es compacto, veamos que T(B

2
)
2
es precompacto.
Dado > 0 vamos a encontrar un 2-cubrimiento nito de T(B

2
). Sea
n
0
tal que 2
n
0
. Obsrvese que el conjunto
A = (
x
1
2
1
,
x
2
2
2
, . . . ,
x
n
0
2
n
0
, 0, 0, . . .); [x
n
[ 1
7. OPERADORES COMPACTOS 141
es compacto en
2
(es acotado y cerrado en R
n
0
que a su vez es cerrado
en
2
) y por tanto admite un -cubrimiento centrado en una -red F.
Veamos que F es una

2-red de T(B

2
). Sea x = (x
n
) B

2
. Entonces
x
n
1 para todo n N, y por tanto existe f F tal que
n
0

n=1
[2
n
x
n
f[
2
<
2
Entonces
|
_
x
n
2
n
_
n
f|
2
2
=
n
0

n=1

x
n
2
n
f
n

2
+

n=n
0
+1

x
n
2
n

2
<
2
+
2
= 2
2
Ejemplo 7.10. Sea A = (a
i,j
)
i,jN
una matriz innita y sea 1
p .
Si p = 1, supongamos que (j) =

i=1
[a
i,j
[ < para todo j N
y que (j) 0 cuando j .
Si p = supongamos que (i) =

j=1
[a
i,j
[ < para todo i N
y que (i) 0 cuando i .
Si 1 < p < y
1
p
+
1
q
= 1, supongamos que

p
=
_
_

i=1
_

j=1
[a
i,j
[
q
_
p
q
_
_
1
p
< .
Entonces el operador
T
A
:
p

p
dado por
T
A
(x) =
_

j=1
a
i,j
x
j
_
i
(est bien denido y) es compacto.
Comencemos viendo el caso p = 1. Veamos en primer lugar que T
A
est bien denido y es continuo. Notemos que si (j) 0 en particular
se tiene que sup
j
(j) = C < . Entonces
|T
A
(x)|
1
=
_
_
_
_
_
_
_

j=1
a
i,j
x
j
_
i
_
_
_
_
_
_
1
=

i=1

j=1
a
i,j
x
j

j
[a
i,j
x
j
[ =
=

i
[a
i,j
[[x
j
[

j
[x
j
[

i
[a
i,j
[ =

j
[x
j
[(j) M|x|
1
142 7. OPERADORES COMPACTOS
Veamos ahora que T es compacto. Sea A
n
la matriz doblemente
innita denida que se obtiene al mantener las primeras n columnas
de A sin cambios y colocar ceros en las restantes columnas. El operador
T
n
asociado a la matriz A
n
, dado por
T
n
(x) =
_
n

j=1
a
i,j
x
j
_
i
acta igual que T, salvo que previamente trunca x y se queda slo
con sus n primeras coordenadas. Es claro que T
n
= T
n
donde
n
vienen denidas como en el Ejemplo 7.8. Es claro entonces que para
cada n N el operador T
n
es continuo y tiene rango nito . Lo nico
que tenemos que probar es que T
n
tiende a T. Pero
|T
n
T| = sup
xB

1
|(T
n
T)(x)| = sup
xB

1
_
_
_
_
_
_
_

j=n+1
a
i,j
x
j
_
i
_
_
_
_
_
_
1
=
= sup
xB

i=1

j=n+1
a
i,j
x
j

sup
xB

i=1

j=n+1
[a
i,j
x
j
[ =
= sup
xB

j=n+1

i
[a
i,j
[[x
j
[ sup
xB

j=n+1
[x
j
[

i
[a
i,j
[ =
= sup
xB

j=n+1
[x
j
[(j) sup
xB

1
|x| sup
j>n
(j) = sup
j>n
(j) 0.
Veamos ahora el caso p = . Sea A
t
la matriz traspuesta de A.
Por el caso p = 1, A
t
dene un operador compacto T
A
t :
1

1
. Ex-
tendiendo los razonamientos hechos para espacios nito dimensionales
en el Ejercicio 5.1 es fcil ver que A = (A
t
)
t
es la matriz asociada al
operador traspuesto de T
A
t , es decir T = (T
A
t )

. Entonces el Teorema
de Schauder nos garantiza que T es compacto.
Veamos ahora el caso 1 < p < . Veamos en primer lugar que
T
A
:
p

p
(est bien denido y) es continuo. Ntese el uso de la
desigualdad de Hlder.
|T
A
(x)|
p
=
_
_
_
_
_
_
_

j=1
a
i,j
x
j
_
i
_
_
_
_
_
_
p

_
_
_
_
_
_
_

j=1
[a
i,j
x
j
[
_
i
_
_
_
_
_
_
p

7. OPERADORES COMPACTOS 143

_
_
_
_
_
_
_
_
_

j=1
[a
i,j
[
q
_1
q
_

j=1
[x
j
[
p
_1
p
_
_
i
_
_
_
_
_
_
p
=
= |x|
p
_
_
_
_
_
_
_
_
_

j=1
[a
i,j
[
q
_1
q
_
_
i
_
_
_
_
_
_
p
= |x|
p

p
.
Ya hemos visto que T
A
es continuo. Veamos que es incluso com-
pacto. Podemos de nuevo considerar las matrices A
n
y sus operadores
de rango nito asociados T
n
= T
n
. De nuevo slo necesitamos com-
probar que T
n
tiende a T en la norma de operadores.
|T
n
T
A
| = sup
xB

p
|(T
n
T
A
)(x)|
p
= sup
xB

p
_
_
_
_
_
_
_

j=n+1
a
i,j
x
j
_
i
_
_
_
_
_
_
p

sup
xB

p
_
_
_
_
_
_
_

j=n+1
[a
i,j
x
j
[
_
i
_
_
_
_
_
_
p

sup
xB

p
_
_
_
_
_
_
_
_
_

j=n+1
[a
i,j
[
q
_1
q
_

j=n+1
[x
j
[
p
_1
p
_
_
i
_
_
_
_
_
_
p
=
= sup
xB

p
|x|
p
_
_
_
_
_
_
_
_
_

j=n+1
[a
i,j
[
q
_1
q
_
_
i
_
_
_
_
_
_
p
0.
Tambin proponemos como ejercicio demostrar que si una de las
sucesiones (j) y (i) denidas ms arriba es acotada y la otra tiende
a 0 entonces T :
p

p
es compacto.
Veamos el anlogo continuo del ejemplo de arriba, lo que se conoce
como operadores integrales de Fredholm.
Ejemplo 7.11. Sea k(, ) : [0, 1] [0, 1] K una funcin a la
que de ahora en adelante nos referiremos como ncleo de Fredholm. El
ncleo k nos permite denir una aplicacin entre espacios funcionales
(de momento por precisar)
x T
k
(x)(s) =
_
1
0
k(s, t)x(t)dt
144 7. OPERADORES COMPACTOS
Estudiemos las caractersticas de este operador en funcin del ncleo k
y del espacio en que lo denamos.
Supongamos inicialmente que k : [0, 1] [0, 1] K es continua.
Sea x : [0, 1] K una funcin integrable. Si s
n
s [0, 1] entonces
el Teorema de la Convergencia Dominada nos garantiza que
T
k
(x)(s
n
) T
k
(x)(s),
es decir, T(x) : [0, 1] K es continua.
Supongamos ahora que (x
n
)
n
es una sucesin de funciones inte-
grables uniformemente acotadas en norma 1, es decir, existe > 0 tal
que |x
n
|
1
para todo n N. Veamos que la sucesin (T
k
(x
n
))
n
tiene una subsucesin que converge uniformemente (es decir, en | |

)
en [0, 1].
Por ser k continuo, existe > 0 tal que |k|

. Entonces es
fcil ver que
|T
k
(x
n
)|


para todo n N. Es decir, (T
k
(x
n
))
n
est acotada en norma innito.
Adems la sucesin es equicontinua en [0, 1]. Vemoslo. k es uniforme-
mente continua en [0, 1] [0, 1] (por la compacidad de [0, 1] [0, 1]).
Entonces para todo > 0 existe > 0 tal que si [su[ < y [t v[ <
entonces
[k(s, t) k(u, v)[ < .
Por lo tanto, para todo n N y para todos s, u [0, 1] con [su[ <
se tiene
[T
k
(x
n
)(s) T
k
(x
n
)(u)[ =
_
1
0
(k(s, t) k(u, t))x
n
(t)dt

_
1
0
[k(s, t) k(u, t)[[x
n
(t)[dt < .
Ahora, el Teorema de Ascoli-Arzela nos garantiza que la sucesin
(T
k
(x
n
))
n
tiene una subsucesin uniformemente convergente (es decir,
convergente en | |

.
Sean entonces X e Y dos cualesquiera de los espacios C[0, 1] o
L
p
[0, 1] (1 p ). Recordemos que para todo 1 p se tiene
que
| |
1
| |
p
| |

y
C[0, 1] L

[0, 1] L
p
[0, 1] L
1
[0, 1]
7. OPERADORES COMPACTOS 145
Por tanto, si x X entonces x es integrable y (es decir, x
L
1
[0, 1]) y si (x
n
) X es una sucesin acotada, entonces (x
n
) est
acotada en | |
1
.
Adems, si y C[0, 1] entonces y Y , y si (y
n
) es una sucesin
uniformemente convergente entonces (y
n
) converge en la norma de Y .
Por tanto, de los razonamientos anteriores se sigue que el operador
T
k
: X Y
(est bien denido y) es compacto.
Veamos que otras situaciones en las que podemos probar con facili-
dad la compacidad de T
k
: X Y . Sea 1 < p , sea X = L
p
[0, 1],
sea Y = L
q
[0, 1] con
1
p
+
1
q
= 1 y supongamos que k L
q
([0, 1] [0, 1]).
Entonces, para todo x X y s [0, 1] usando la desigualdad de Hlder
anlogamente a como lo hicimos en el Ejemplo 7.10 tenemos
[T
k
(x)(s)[
_
1
0
[k(s, t)[[x(t)[dt |x|
p
__
1
0
[k(s, t)[
q
dt
_
1
q
y por lo tanto
|T
k
(x)|
q
=
__
1
0
[T
k
(x)(s)[
q
ds
_
1
q

|x|
p
__
1
0
_
1
0
[k(s, t)[
q
dtds
_
1
q
= |x|
p
|k|
q
de donde se sigue la continuidad de T
k
.
Para ver la compacidad de T
k
, puesto que C([0, 1] [0, 1]) es denso
en L
q
([0, 1] [0, 1]) (esto se prueba anlogamente a la densidad de
C[0, 1] en L
q
[0, 1]), sea (k
n
) C([0, 1] [0, 1]) una sucesin tal que
|k
n
k|
q
0 y sea T
n
: L
p
[0, 1] L
q
[0, 1] el operador de Fredholm
de ncleo k
n
. Entonces
|T
k
T
n
|
/(L
p
;L
q
)
= sup
xB
L
p
[0,1]
|(T
k
T
n
)(x)|
q
=
= sup
xB
L
p
[0,1]
__
1
0
[(T
k
T
n
)(x)(s)[
q
ds
_
1
q

sup
xB
L
p
[0,1]
|x|
p
__
1
0
_
1
0
[k(s, t) k
n
(s, t)[
q
dtds
_
1
q
= |k k
n
|
q
0.
146 7. OPERADORES COMPACTOS
A menudo el inters de los operadores compactos radica en su ca-
pacidad de mejorar la convergencia. Para estudiar esto necesitamos
previamente una denicin.
Definicin 7.12. Sean X, Y espacios de Banach. Un operador T :
X Y se dice completamente continuo si transforma sucesiones
dbilmente convergentes en sucesiones convergentes en norma, es decir,
si para todo sucesin (x
n
)
n
X tal que x
n
tiende dbilmente a x X
se tiene que T(x
n
) converge en norma a T(x).
Teorema 7.13. Sean X, Y espacios de Banach. Entonces si T es
compacto T es completamente continuo. Recprocamente, si X es reex-
ivo y T : X Y es completamente continuo entonces T es compacto
(de hecho basta pedir X ,
1
, pero esto se sigue de un resultado muy
profundo de Rosenthal).
Demostracin. Supongamos que T es compacto y x
n
tiende d-
bilmente a x. (x
n
) X es dbilmente acotada, y por tanto acotada
por el Teorema 4.9. Si T(x
n
) , T(x) podemos suponer, pasando a una
subsucesin, que existe > 0 tal que
(10) |T(x
n
) T(x)|
para todo n N. Por ser T compacto y (x
n
)
n
acotada, existe una
subsucesin (x
n
j
) tal que T(x
n
j
) converge a y Y . Entonces, de (10)
se sigue que
|y T(x)|
de forma que y ,= T(x). Sin embargo, para todo y

Y

se tiene que
y

T X

y por tanto de la hiptesis sobre (x


n
) se sigue que
(y

T)(x) = lm
j
(y

T)(x
n
j
) = y

_
lm
j
T(x
n
j
)
_
= y

(y)
y por tanto x = y y hemos alcanzado una contradiccin.
Para probar la otra implicacin necesitamos el Teorema de Eberlein,
que no es seguro que incluyamos en el programa. Si lo hemos dado, la
demostracin es sencilla. Supongamos que X es reexivo y T : X Y
es completamente continuo. Si (x
n
) B
X
, por el Teorema de Eberlein
(x
n
) admite una subsucesin (x
n
k
) dbilmente convergente a un cierto
x B
X
(por el Teorema de Mazur, las clausuras en norma y dbil de
un convexo coinciden). Al ser T completamente continuo, la sucesin
(T(x
n
k
))
k
converge en norma, lo que termina la demostracin.
7. OPERADORES COMPACTOS 147
Prcticas sugeridas
Ejercicio 7.1. Sea X un espacio de Banach y sea T /(X)
F(X). Demostrar que 0 T(S
X
). Sugerencia: Utilizar el Teorema de
la Aplicacin Abierta.
Ejercicio 7.2. Sea C
(1)
[0, 1] el espacio de las funciones con deriva-
da primera continua con la norma |f| = |f|

+ |f
t
|

. Demostrar
que la inclusin formal
i : C
(1)
[0, 1] C[0, 1]
es un operador compacto. Sugerencia: Utilizar el Teorema de Ascoli-
Arzela.
Ejercicio 7.3. Sea 1 < p < y sea T : c
0

p
un operador
continuo. Demostrar que T es compacto.
CAPTULO 8
Teora espectral de operadores compactos
Comenzamos en este captulo el estudio de uno de los grandes
temas del Anlisis Funcional, la Teora Espectral. Un enfoque posible
es comenzar estudiando lgebras de Banach, demostrar en ese marco
general los resultados que se van a necesitar y posteriormente observar
que los operadores de un espacio en s mismo es un lgebra de Banach y
obtener los resultados de la Teora Espectral prcticamente como coro-
lario. Sin embargo, pensamos que ese enfoque es algo arduo, puesto
que le exige al alumno el estudio abstracto inicial de las lgebras de
Banach sin haber trabajado previamente en un modelo intuitivo en el
que apoyarse. Preferimos empezar enunciando y demostrando los re-
sultados para el caso de operadores en este captulo y dejar para ms
adelante (y no en esta asignatura) la generalizacin de estos resultados
al contexto de lgebras de Banach.
Empezamos estudiando operadores inversibles y perturbaciones in-
versibles de la identidad, para a continuacin denir el espectro de un
operador y sus autovalores. Probamos que el espectro es compacto y
probamos el Teorema de Gelfand, utilizando variable compleja. Tras
ello se puede probar la Frmula del Radio Espectral, aunque no la
necesitamos. A continuacin desarrollamos la Teora Espectral de Op-
eradores Compactos en Espacios de Banach, conocida como Teora de
Riesz-Schauder. Como corolario de esta teora se obtiene la Alternativa
de Fredholm, de gran utilidad en las aplicaciones.
Hemos seguido principalmente [31] y [24] en la preparacin de la
primera parte del captulo y [33] y [31] en la presentacin de la Teora
de Riesz-Schauder.
Teora espectral de operadores compactos
Empezamos deniendo operadores inversibles, y estudiando algunas
de sus propiedades bsicas.
Definicin 8.1. Dados dos espacios de Banach X, Y , un operador
T : X Y es inversible si es un isomorsmo biyectivo.
149
150 8. TEORA ESPECTRAL DE OPERADORES COMPACTOS
Observacin 8.2. T L(X; Y ) es inversible si y slo si existe
T
1
L(Y ; X) tal que T
1
T = I
X
y TT
1
= I
Y
Se sigue del Teorema de la Aplicacin Abierta que T es inversible
si y slo si T es biyectivo.
Es fcil ver que si T L(X; Y ) y S L(Y ; Z) son inversibles
entonces ST es inversible y
(ST)
1
= T
1
S
1
De forma anloga si T, S L(X), es fcil ver que S y T son in-
versibles si y slo si ST y TS lo son.
Proposicin 8.3. T : X Y es inversible si y slo si T

:
Y

X

es inversible, y en ese caso (T

)
1
= (T
1
)

.
Demostracin. Si T es inversible tenemos que
T

(T
1
)

= (T
1
T)

= (I
X
)

= I
X

y
(T
1
)

= (TT
1
)

= (I
Y
)

= I
Y
.
Recprocamente, si T

es inversible, tambin lo es T

. Si denimos
S = (T

)
1
[
Y
es fcil ver que S es un operador inyectivo de Y en X (para
ver que S toma valores en Y , notemos que para todo y Y existe x X
tal que T(x) = y. Por lo tanto T

(x) = y y S(y) = x X). Adems


claramente ST = I
X
y TS = I
Y
, de forma que T es inversible.
Lema 8.4. Sea X un espacio de Banach y T L(X) un operador
tal que
|T| < 1.
Entonces el operador (I T) L(X) es inversible y adems
(I T)
1
=

n=0
T
n
,
donde la convergencia es en la norma de operadores, y
|(I T)
1
|
1
1 |T|
.
Demostracin. Ntese en primer lugar que

n=0
|T
n
|

n=0
|T|
n
=
1
1 |T|
.
8. TEORA ESPECTRAL DE OPERADORES COMPACTOS 151
Es decir, la serie

n=0
T
n
es absolutamente convergente en el espacio
de Banach L(X) y por tanto la serie

n=0
T
n
converge en L(X). Por
tanto
(I T)

n=0
T
n
= (I T) + (T T
2
) + (T
2
T
3
) + = I
y
_

n=0
T
n
_
(I T) = (I T) + (T T
2
) + (T
2
T
3
) + = I

Lema 8.5. Sea X un espacio de Banach y T L(X) un operador


tal que existe p N de manera que
|T
p
| < 1.
Entonces el operador (I T) L(X) es inversible y adems
(I T)
1
=

n=0
T
n
,
donde la convergencia es en la norma de operadores, y
|(I T)
1
|
1 +|T| + +|T
p1
|
1 |T
p
|
.
Demostracin. Puesto que
|T
pn+j
| |T
p
|
n
|T
j
|
y que
|T
p
| < 1
se tiene que

n=0
|T
n
| =

n=0
|T
pn
| +

n=0
|T
pn+1
| + +

n=0
|T
pn+p1
|

n=0
|T
p
|
n
(1 +|T| + |T
p1
|) =
1 +|T| + +|T
p1
|
1 |T
p
|
Ahora la demostracin sigue muy similar a la del lema anterior.
152 8. TEORA ESPECTRAL DE OPERADORES COMPACTOS
Lema 8.6. Sea X un espacio de Banach y S, T L(X). Si T es
inversible y
|T
1
(S T)| < 1
entonces S es inversible y
S
1
= T
1

n=0
_
(S T)T
1
_
n
,
|S
1
|
|T
1
|
1 |(S T)T
1
|
y
|S
1
T
1
|
|T
1
|
2
|S T|
1 |(S T)T
1
|
Demostracin. Si |(T S)T
1
| < 1, el Lema 8.4 nos dice que
I (T S)T
1
= I I + ST
1
= ST
1
es inversible y
(ST
1
)
1
=

n=0
_
(T S)T
1
_
n
.
Como T es inversible, se sigue que
(ST
1
)T = S
es inversible y
S
1
= T
1
(ST
1
)
1
= T
1

n=0
(T S)T
1
)
n
.
Por hiptesis, |(T S)T
1
| < 1 y por tanto
|S
1
|T
1
|

n=0
|(T S)T
1
|
n
=
|T
1
|
1 |(T S)T
1
|
.
Adems, como
(S
1
T
1
) = S
1
TT
1
S
1
ST
1
= S
1
(T S)T
1
tenemos que
|S
1
T
1
| |S
1
||(T S)T
1
|
|T
1
|
2
|S T|
1 |(S T)T
1
|
.

Corolario 8.7. Sea X un espacio de Banach. Entonces el conjun-


to ( de operadores inversibles en X es un conjunto abierto de L(X) y
la aplicacin T T
1
es un homeomorsmo de ( en s mismo.
8. TEORA ESPECTRAL DE OPERADORES COMPACTOS 153
Demostracin. Si T es inversible, consideremos la bola de centro
T y radio
1
|T
1
|
. Entonces, para todo S en esa bola, se tiene que
|S T| <
1
|T
1
|
y por tanto
|T
1
(S T)| |T
1
||S T| < 1
y el lema anterior nos dice que S es inversible. Por tanto ( es abierto.
Sea ahora (T
n
)
n
( una sucesin de operadores inversibles tales
que T
n
T (. Entonces a partir de cierto n
0
se tiene que
|T
n
T| <
1
|T
1
|
y de nuevo por el lema anterior tenemos que
|T
1
n
T
1
|
|T
1
|
2
|T
n
T|
1 |(T
n
T)T
1
|
lo que implica que |T
1
n
T
1
| 0, es decir, la inversin es continua,
y esto es todo lo que hace falta probar.
La idea bsica de la teora espectral es estudiar los valores de k K
para los que T kI es, o no, inversible. Recordemos que esa es la
pregunta bsica a la que nos lleva la teora de diagonalizacin.
Tenemos entonces la siguiente denicin:
Definicin 8.8. Sea T : X X. El espectro de T es el conjunto
(T) = k K tales que T kI no es inversible .
El conjunto (T) := K (T) se denomina resolvente de T.
Y la pregunta ser cmo calcular los valores espectrales k (T).
Veremos que nos interesar destacar dos conjuntos dentro del es-
pectro.
Definicin 8.9. Sea T : X X. El conjunto de autovalores de
T es el conjunto

e
(T) = k K tales que T kI no es inyectivo .
El conjunto de autovalores aproximados de T es el conjunto

a
(T) = k K tales que T kI no est acotado inferiormente .
Notemos que k
e
(T) si y slo si existe 0 ,= x X tal que
T(x) = kx
154 8. TEORA ESPECTRAL DE OPERADORES COMPACTOS
En ese caso k es un autovalor y x es un autovector asociado a k. El
subespacio
ker(T kI)
es el autoespacio asociado a k.
Notemos tambin que k
a
(T) si y slo si existe una sucesin
(x
n
)
n
S
X
tal que
|T(x
n
) kx
n
| 0
En ese caso k es un autovalor aproximado de T. Si k
e
(T) es un
autovalor y x es uno de sus autovectores haciendo x
n
=
x
|x|
para todo
n se ve que k es un autovalor aproximado. Por tanto se tiene

e
(T)
a
(T) (T)
Veamos que de hecho para los operadores de rango nito los tres
conjuntos son el mismo.
Teorema 8.10. Sea X un espacio normado y T T(X). Entonces

e
(T) =
a
(T) = (T)
Demostracin. Puesto que
e
(T)
a
(T) (T), basta ver que
(T)
e
(T), equivalentemente que (
e
(T))
c
((T))
c
. Sea entonces
k ,
e
(T), de forma que T kI es inyectivo. Veamos que entonces
T kI es inversible, es decir k , (T).
Consideramos primero el caso en que dimX = n < . En ese caso
el resultado es elemental y se estudia en lgebra Lineal: Puesto que
dimker(T kI) + dimIm(T kI) = n,
si TkI es inyectivo entonces ker(TkI) = 0 y por tanto dimIm(T
kI) = n, es decir T kI es sobreyectiva y por tanto inversible.
Consideramos ahora el caso en que X tiene dimensin innita. En-
tonces k ,= 0, ya que si T tiene rango nito no puede ser inyectivo
(por ejemplo, si x
n
; n N es un sistema linealmente independiente,
entonces T(x
n
); n N sera un sistema innito linealmente indepen-
diente contenido en un espacio de dimensin nita).
Sea
S = (T kI)
[
Im(T)
: Im(T) Im(T)
(para ver que S toma efectivamente valores en Im(T), ntese que
(T kI)(T(x)) = T(T(x)) kI(T(x)) = T(T kI)(x) Im(T).)
8. TEORA ESPECTRAL DE OPERADORES COMPACTOS 155
Como T kI es inyectivo, tambin lo es S. Por el caso anterior, S es
sobreyectiva. Sea ahora y X. Entonces T(y) Im(T) y por tanto
existe u Im(T) tal que S(u) = T(y). Es decir,
(T kI)(u) = T(y), equivalentemente T(u y) = ku.
Sea x =
uy
k
. Entonces
T(x) = T
_
u y
k
_
=
ku
k
= u = kx + y,
es decir
(T kI)(x) = y
y por tanto T kI es sobreyectiva, y ahora terminamos con el Teorema
de la Aplicacin Abierta.
En cambio no es difcil dar ejemplos de operadores para los que

e
(T) ,=
a
(T) y
a
(T) ,= (T) ([31, 12.7]).
Empecemos a estudiar el espectro de un operador
Teorema 8.11. Sea X un espacio de Banach y T L(X). En-
tonces
1. Sea k K tal que [k[
m
> |T
m
| para algn m N. Entonces
k , (T) y
(T kI)
1
=

n=0
T
n
k
n+1
.
Por tanto, para todo k (T) se tiene que
[k[ nf
n
|T
n
|
1
n
|T|.
2. (T) K es un conjunto compacto
3. Si X tiene dimensin innita y T /(X) entonces 0 (T).
Demostracin. 1. Notemos que k ,= 0 y que
T kI = k
_
I
T
k
_
.
El Lema 8.5 nos dice que T kI es inversible y que
(T kI)
1
=
1
k
_
I
T
k
_
1
=
1
k

n=0
T
n
k
n
=

n=0
T
n
k
n+1
El resto es fcil
156 8. TEORA ESPECTRAL DE OPERADORES COMPACTOS
2. De lo anterior se sigue que (T) k K; [k[ |T|, y por
tanto est acotado. Slo tenemos que ver que es cerrado. Sea (k
n
)
n

(T) una sucesin tal que k
n
k. Entonces es trivial que
T k
n
I T kI
Puesto que los operadores inversibles formaban un conjunto abierto (,
se sigue que su complementario, los no inversibles, forman un conjunto
cerrado. Puesto que, para todo n N T k
n
I no es inversible, se sigue
que T kI tampoco lo es, es decir k (T).
3. Un operador inversible T dene una norma |[[| en X equivalente
a la norma original de X dada por
|[x[| = |T(x)|.
Por tanto si T fuera inversible denira en X una norma equivalente
cuya bola unidad T(B
X
) es un compacto, lo que implicara que X es
de dimensin nita.
Ejemplo 8.12. Sea k : [0, 1] [0, 1] K una funcin continua y
sea T : C[0, 1] C[0, 1] el operador denido como
T(f)(s) =
_
1
0
k(s, t)f(t)dt para todo s [0, 1].
En el Ejemplo 7.11 ya vimos que T est bien denido y es compacto.
Adems
|T(f)| = sup
s
[T(f)(s)[ sup
s,t
[k(s, t)[|f| = |k|

|f|

,
luego
|T| |k|

.
En ese caso decimos que T es un operador integral de Fredholm con
ncleo continuo k(, ).
Si S es otro operador integral de Fredholm con ncleo continuo
h(, ), es fcil ver que TS es tambin un operador integral de Fred-
holm con ncleo continuo k h, donde para todos 0 s, t 1,
k h(s, t) =
_
1
0
k(s, u)h(u, t)du
y
|TS| |k|

|h|

.
Haciendo T = S (es decir, h = k) y aplicando induccin tenemos que
para todo n N T
n
es un operador integral de Fredholm con ncleo
8. TEORA ESPECTRAL DE OPERADORES COMPACTOS 157
continuo
k
(n)
(s, t) =
_
1
0

_
1
0
k(s, u
1
)k(u
1
, u
2
) k(u
n1
, t)du
1
du
n1
.
Sea k un ncleo continuo tal que |k|

1. Entonces |T| < 1 y por


el Lema 8.4 se tiene que I T es inversible e
(11) (I T)
1
(f)(s) =

n=0
T
n
(f)(s) = f(s) +

n=1
_
1
0
k
(n)
(s, t)f(t)dt
Observemos que la serie

n=1
k
(n)
(s, t) converge uniforme y absolu-
tamente en (s, t) [0, 1][0, 1] (ya que para todo n N |k
(n)
| |k|
n
)
y por tanto
h(s, t) :=

n=0
k
(n)
(s, t)
es una funcin continua en [0, 1] [0, 1]. Adems, por el Teorema de la
Convergencia Acotada podemos intercambiar el sumatorio y la integral
en (11) y tenemos que
(I T)
1
(f)(s) = f(s) +
_
1
0
h(s, t)f(t)dt,
es decir
(I T)
1
= I + B,
donde B es otro operador integral de Fredholm con ncleo continuo.
Si x es un autovector del autovalor k de T entonces se tiene que
T(span[x]) span[x]. Decimos entonces que span[x] es invariante por
T. En general un subespacio V X se dice invariante si por T si
T(V ) V . Claramente 0 y X son subespacios invariantes para todo
T L(X). Durante un largo tiempo no se supo si exista algn espacio
de Banach X y algn operador denido en l T que no tuviera ningn
subespacio invariante no trivial (esto es, distinto de 0 y X); reciente-
mente P. Eno ([18]) encontr tales X y T. An no se sabe si dado un
espacio de Hilbert H existe T L(H) sin subespacios invariantes no
triviales. En cambio, recientemente Lomonosov ha probado que todo
operador compacto entre espacios de Hilbert admite subespacios invari-
antes no triviales. Una exposicin de estos problemas se puede ver en
[20] y la bibliografa all citada.
En relacin a esto, veamos a continuacin un ejemplo de un oper-
ador entre espacios de Hilbert sin autovalores.
158 8. TEORA ESPECTRAL DE OPERADORES COMPACTOS
Ejemplo 8.13. Sea T : L
2
[0, 1] L
2
[0, 1] (sobre los nmeros
complejos) dado por
T(f(t)) = tf(t)
para todo t [0, 1].
Para ver que no tiene autovalores, si k C fuera un autovalor,
existira f(t) L
2
[0, 1] tal que, para casi todo t [0, 1]
tf(t) = kf(t), es decir (k t)f(t) = 0,
de donde, tomando t ,= k, tenemos que f(t) = 0 para casi todo t [0, 1],
es decir f(t) = 0.
En cambio, veamos que todo k [0, 1] pertenece al espectro de T.
Sea [0, 1] y sea > 0 tal que [, + ] [0, 1] o [ , ] [0, 1].
Supongamos por ejemplo que [, + ] [0, 1]. Sea
f

=
_

_
1

, si t [, + ]
0 , si t , [, + ]
Claramente |f

|
2
= 1. Adems
(T I)(f

)(t) = T(f

)(t) f

(t) = tf

(t) f

(t) = f

(t )
y por tanto
|(T I)(f

)|
2
2
=
_
1
0
1

( t)
2
dt = =

2
3
y por tanto (T I)(f

) 0 cuando tiende a 0. En consecuencia


T I no est acotado inferiormente y por ello no puede ser inversible.
Observemos nalmente que T tiene una gran abundancia de sube-
spacios invariantes no triviales. En particular, es fcil ver que para
todo r (0, 1), L
2
[0, r] es invariante por T.
Uno de los resultados fundamentales de la Teora Espectral es el
Teorema de Gelfand-Mazur que enunciamos a continuacin. Su de-
mostracin es una bonita aplicacin de la Teora de Variable Compleja.
Teorema 8.14 (Gelfand). Si X es un espacio de Banach complejo
y T L(X) entonces (T) ,= .
Demostracin. Sea f L(X)

. Recordemos que (T) = (T)


c
es la resolvente de T. Denimos la funcin
w
f
: (T) C
8. TEORA ESPECTRAL DE OPERADORES COMPACTOS 159
como
w
f
(z) = f((T zI)
1
).
Sabemos que (T) C es un abierto (porque (T) es cerrado). Veamos
que w
f
es analtica en (T): sea z
0
(T). Entonces
(T z
0
I) (T zI) = (z z
0
)I
y por tanto, componiendo por la derecha con (T zI)
1
y por la
izquierda con (T z
0
I)
1
tenemos
(T zI)
1
(T z
0
I)
1
= (z z
0
)(T z
0
I)
1
(T zI)
1
y por lo tanto
lm
zz
0
w
f
(z) w
f
(z
0
)
z z
0
= lm
zz
0
f
_
(T zI)
1
(T z
0
I)
1
z z
0
_
=
= lm
zz
0
f
_
(T z
0
I)
1
(T zI)
1
_
= f((T z
0
I)
1
)
por la continuidad de f y por la continuidad de la inversin probada
en el Corolario 8.7.
Supongamos ahora que (T) = . Entonces (T) = C por lo que w
f
es analtica en C, es decir, es una funcin entera. Adems, si [z[ > |T|,
usando el Lema 8.4 se tiene que
(12) |(T zI)
1
| =
_
_
_
_
_
1
z
_
T
z
I
_
1
_
_
_
_
_

1
[z[ |T|
0 cuando [z[ .
Como f es lineal y continua,
w
f
(z) = f((T zI)
1
) 0 cuando [z[
y por tanto w
f
est acotada en C.
Por tanto el Teorema de Liouville nos dice que w
f
, siendo entera
y acotada debe de ser constante. De (12) se sigue que w
f
debe de ser
siempre 0. En particular
0 = w
f
(0) = f(T
1
).
Como esto ocurre para todo f L(X)

se sigue que T
1
= 0, lo cual
es imposible. Por tanto (T) ,= 0.
Si se desea, y en funcin del tiempo disponible, en relacin con esto
se puede denir el radio espectral de un operador T : X X como
r

(T) = sup
k(T)
[k[
y a continuacin demostrar la
160 8. TEORA ESPECTRAL DE OPERADORES COMPACTOS
Proposicin 8.15 (Frmula del Radio Espectral). Si X es un es-
pacio de Banach complejo y T L(X) entonces
r

(T) = nf
nN
|T
n
|
1
n
= lm
n
|T
n
|
1
n
.
Demostracin. Una demostracin adecuada para este curso se
puede ver en [31, 12.8 b].
Pasamos a estudiar la llamada Teora de Riesz-Schauder. Necesita-
mos unos cuantos resultados previos, interesantes en s mismos, antes
de llegar a los resultados principales de la teora. Hemos seguido para
esta seccin [33] principalmente.
Lema 8.16. Sea X un espacio de Banach, T /(X). Entonces
ker(I T) tiene dimensin nita.
Demostracin. Sea U la bola unidad cerrada de ker(I T). En-
tonces T(U) = U. Puesto que T es compacto y U B
X
, se tiene que
U = T(U) es relativamente compacto, y por tanto compacto (puesto
que es cerrado). Por tanto, si la bola unidad es compacta el espacio
debe de ser de dimensin nita.
Lema 8.17. Sea X un espacio de Banach, T /(X). Si (x
n
)
n
X
es una sucesin acotada tal que x
n
T(x
n
) y X entonces existen
x X y una subsucesin (x
n
j
)
j
(x
n
)
n
tales que x
n
j
x X y
x T(x) = y.
Demostracin. Por la compacidad de T, existe una subsucesin
(x
n
j
)
j
(x
n
)
n
tal que T(x
n
j
) converge a z X. Entonces
x
n
j
= x
n
j
T(x
n
j
) + T(x
n
j
) y + z
Llamamos x = y + z y, puesto que T es continuo, se tiene
(I T)(x) = lm
j
x
n
j
T(x
n
j
) = y + z z = y.

Lema 8.18. Sea X un espacio de Banach, T /(X). Entonces


Im(I T) es cerrado y tiene codimensin nita.
Demostracin. Veamos en primer lugar que Im(I T) es cer-
rado. Sea q : X X/ ker(I T) la aplicacin cociente. Entonces
sabemos que I T factoriza como
I T = (

I T) q
donde

I T : X/ ker(I T) X
8. TEORA ESPECTRAL DE OPERADORES COMPACTOS 161
es la aplicacin cannica dada por

I T([x]) = (I T)(x).
Es un ejercicio comprobar que

I T est bien denida, es lineal y
continua, con |

I T| = |I T|.
Veamos que existe c > 0 tal que, para todo x X
(13) |x T(x)| = |(

I T)([x])| c|[x]|
Si no fuera as, existira una sucesin (x
n
) X tal que
(14) lm
n
|x
n
T(x
n
)| = 0 y |[x
n
]| = 1 para todo n N.
Por la denicin de la norma cociente podemos suponer que, para
todo n N,
1 |x
n
| 2.
Puesto que T es compacto, existen y X y una subsucesin (x
n
j
)
j

(x
n
)
n
tal que T(x
n
j
) y X. Por (14) sabemos que
lm
j
x
n
j
= y
y por tanto
y T(y) = lm
j
x
n
j
T(x
n
j
) = y y = 0,
es decir y ker(IT). Pero ahora usando la otra mitad de (14) tenemos
una contradiccin puesto que
1 = |[x
n
j
]| |x
n
j
y| 0
Por tanto sabemos que (13) es cierto. De (13) se sigue que (

I T)
es un isomorsmo sobre su imagen y en particular Im(

I T) X es
cerrado. Puesto que
Im(I T) = Im(

I T),
se sigue que Im(I T) es cerrado.
Veamos ahora que Im(I T) tiene codimensin nita. Puesto que
Im(IT) X es un subespacio cerrado, podemos considerar el espacio
cociente X/Im(I T) y sabemos que
(X/Im(I T))

= (Im(I T))

162 8. TEORA ESPECTRAL DE OPERADORES COMPACTOS


y por otro lado es un ejercicio ver que (Im(I T))

= ker((I T)

) =
ker(I

) = ker(I
X
T

). Puesto que T es compacto, el Teorema


de Schauder nos dice que T

es compacto. Ahora el Lema 8.16 nos dice


que ker(I
X
T

) tiene dimensin nita, y de


ker(I
X
T

) = (X/Im(I T))

se sigue que X/Im(I T) tiene dimensin nita, es decir Im(I T)


tiene codimensin nita.
Observemos que si X es un espacio de Banach y T L(X), por la
frmula del binomio de Newton, para todo n N se tiene
(I T)
n
= I
n

j=1
_
n
j
_
(1)
j1
T
j
=: I T
n
Si adems T es compacto, puesto que los operadores compactos forman
un subespacio vectorial, se sigue que cada uno de los T
n
as denidos
es compacto. Por tanto, para todo n N:
ker(I T)
n
tiene dimensin nita, y claramente ker(I T)
n

ker(I T)
n+1
.
Im(IT)
n
tiene codimensin nita, es cerrado y Im(IT)
n

Im(I T)
n+1
.
Veamos que los ncleos ker(I T)
n
no pueden crecer indenida-
mente, sino que a partir de cierto n
0
se estabilizan.
Proposicin 8.19. Sea X un espacio de Banach y T /(X).
Entonces existe n
0
N tal que, para todo n n
0
,
ker(I T)
n
0
= ker(I T)
n
.
Demostracin. Si no fuera as, podemos suponer ker(I T)
n

ker(I T)
n+1
para todo n N (si no, razonamos anlogamente toman-
do subsucesiones); en ese caso existira una sucesin (x
n
)
n
X tal que,
para todo n N
x
n
ker(I T)
n+1
ker(I T)
n
y
(15) 1 = nf
yker(IT)
n
|x
n
y| |x
n
| 2
(donde hemos usado el Lema de Riesz para la segunda armacin).
Entonces, si m < n, se tiene
(I T)
n
((I T)(x
n
) + x
m
(I T)(x
m
)) = 0
8. TEORA ESPECTRAL DE OPERADORES COMPACTOS 163
por lo que
(I T)(x
n
) + x
m
(I T)(x
m
) ker(I T)
n
y por tanto
|T(x
n
) T(x
m
)| = |x
n
((I T)(x
n
) + x
m
(I T)(x
m
)| 1
por (15).
Por tanto la sucesin (T(x
n
))
n
no puede tener subsucesiones con-
vergentes, en contradiccin con el hecho de que T sea compacto.
Lema 8.20. Sea X un espacio de Banach y T /(X). Entonces
existe n N tal que los subespacios cerrados N := ker(I T)
n
y
R := Im(I T)
n
verican
1. codimR <
2. N R = X
3. (I T)(N) N
4. (I T)(R) R
5. (I T)
R
:= (I T)
[
R
L(R) es invertible
6. (I T)
N
:= (I T)
[
N
L(N) es nilpotente, en concreto
(I T)
n
N
= 0
Demostracin. Aplicando el Lema 8.19 a (I T), a (I T)

=
I T

y a (I T)

= I T

tenemos que existe n N tal que, para


todo m n,
ker(I T)
n
= ker(I T)
m
, ker(I T

)
n
= ker(I T

)
m
, y
ker(I T

)
n
= ker(I T

)
m
.
Usando que Im(I T)
m
es cerrado para todo m N, ya hemos
visto en la demostracin del Lema 8.18 que
Im(I T)
m
= (ker(I T

)
m
)

.
Por tanto, para todo m n se tiene
Im(I T)
n
= (ker(I T

)
n
)

= (ker(I T

)
m
)

= Im(I T)
m
.
Sean entonces N y R como en el enunciado. Ambos son cerrados y
dimN < y codimR < (esto ya est probado en el Lema 8.18).
Adems
(I T)(N) = (I T)(ker(I T)
n+1
) ker(I T)
n
= N
y
164 8. TEORA ESPECTRAL DE OPERADORES COMPACTOS
(I T)(R) = (I T)(Im(I T)
n
) = Im(I T)
n+1
= R.
Por la eleccin de n, (I T)
n
N
= 0. Acabamos de ver que (I T)
R
es sobre. Veamos que tambin es inyectivo: Si
(I T)
R
(x) = 0,
entonces se tiene que x ker(I T)
n+1
= ker(I T)
n
, es decir,
(I T)
n
(x) = 0.
Es decir, (I T)
R
es inyectiva y por tanto biyectiva. De aqu se
sigue que N R = 0 -en efecto, si x N entonces (I T)
n
(x) = 0;
adems, si x R, por ser (I T)
n
biyectiva sobre R se tiene que
(I T)
n
(x) = 0 si y slo si x = 0- y por tanto
dimN codimR.
Aplicando un razonamiento anlogo a T

en lugar de T, se sigue
que
dimker(I T

)
n
codim(I T

)
n
(X).
Como en la prueba del Lema 8.18 podemos ver que
ker(I T

)
n
= X/Im(I T)
n
y por tanto
codimR = dim(X/Im(I T)
n
) = dimker(I T

)
n
y puesto que Im(I T

)
n
es cerrado, de nuevo razonando como en el
Lema 8.18 tenemos que
(ker(I T)
n
)

= X

/(ker(I T)
n
)

=
= X

/((I T)
n
)

(X) = X

/(I T

)
n
(X)
y por tanto
dimN = dimker(I T)
n
= dim(ker(I T)
n
)

= codim(I T

)
n
(X)
y juntando todo se tiene que
dimN = codim(I T

)
n
(X) dimker(I T

)
n
= codimR
As pues se tiene que
dimN = codimR
de donde se sigue que X = N R
8. TEORA ESPECTRAL DE OPERADORES COMPACTOS 165
Proposicin 8.21. Sea X un espacio de Banach de dimensin in-
nita y sea T /(X). Entonces, para todo k (T) 0 existen
dos subespacios de X topolgicamente complementarios N
k
y R
k
in-
variantes por kI T y que verican
1. (kI T)
[
R
k
es un isomorsmo de R
k
en s mismo.
2. Existe n
k
N tal que
(kI T)
n
k
[
N
k
0,
es decir, (kI T)
n
k
[
N
k
es nilpotente.
3. 0 , = ker(kI T) N
k
y dimN
k
< .
Demostracin. Puesto que
1
k
T es compacto para todo k ,= 0,
podemos aplicar la Proposicin 8.20 a (I
1
k
T) =
1
k
(kI T) para
obtener n
k
, N
k
, R
k
, (I T)
N
k
e (I T)
R
k
con las propiedades all es-
tablecidas. Entonces R
k
y N
k
son topolgicamente complementarios y
(1) y (2) quedan probados.
Puesto que (kI T)
[
R
k
es un isomorsmo (esto es (1)) y dado que
X = N
k
R
k
, si N
k
fuera 0, tendramos que (kI T) sera inversible,
en contradiccin con que k (T). Por tanto N
k
,= 0. Ahora, dado
que (kI T)
[
N
k
es nilpotente, no puede ser inyectivo, por lo que existe
0 ,= x
0
N
k
tal que (kI T)(x
0
) = 0, es decir
x
0
ker(kI T)
lo que prueba que ker(kI T) ,= 0.
Puesto que N
k
= ker(kI T)
n
k
(ver Lema 8.20), est claro que
ker(kI T) ker(kI T)
n
k
, y que
dimN
k
<
se sigue del Lema 8.20.
Finalmente podemos probar el resultado principal de la Teora de
Riesz-Schauder del espectro de operadores compactos.
Teorema 8.22. Sea X un espacio de Banach de dimensin innita,
y T /(X). Entonces
(i) 0 (T)
(ii) Todo k (T) 0 es un autovalor, y el autoespacio corre-
spondiente E
k
tiene dimensin nita.
(iii) Existe una sucesin convergente a cero (k
n
)
n
N (quizs even-
tualmente constante) tal que (T) = 0 k
n
; n N.
(iv) Para todo k K 0 se tiene
dimker(kI T) = codim(kI T)(X).
166 8. TEORA ESPECTRAL DE OPERADORES COMPACTOS
Demostracin. (i) y (ii) ya han sido probados en la Proposicin
8.21.
Para probar (iii), probemos en primer lugar el siguiente
Aserto: Para todo k K0 existe un entorno abierto U
k
de k tal
que kI T es un isomorsmo de X para todo z U
k
k.
Demostracin del Aserto: Puesto que (T) es cerrado, su comple-
mentario es abierto, por lo que dado k K (T) existe un entorno
U
k
de k totalmente contenido en K (T).
Ahora, si k (T) 0 y R
k
y N
k
son como en la Proposicin
8.21, entonces
(kI T)
[
R
k
= kI
R
k
T
[
R
k
es inversible, es decir k , (T
[
R
k
). Puesto que T
[
R
k
es compacto, de
nuevo (T
[
R
k
) es compacto y K(T
[
R
k
) es un abierto. Por tanto, existe
U
k
K un entorno de k tal que (zI T) es inversible para todo z U
k
.
Como (kI T)
[
N
k
es nilpotente, se tiene que ((kI T)
[
N
k
) = 0 (esto
es un ejercicio), es decir, para todo z ,= 0 se verica que (zI T)
[
N
k
es
inversible (como elemento de L(N
k
)).
Juntando ambas cosas tenemos que para todo z U
k
(zI T)
[
N
k
y
(zI T)
[
R
k
son inversibles. Como X = N
k
R
k
tenemos que (zI T)
es inversible (esto es un ejercicio) lo que prueba el aserto.
Ahora, puesto que (T) es compacto, para todo > 0 el conjunto
M

= k (T); [k[ = (T) k K; [k[ |T|


es compacto. Del Aserto se sigue que ningn punto de M

es de acu-
mulacin, por lo que M

ha de ser nito. De aqu se sigue que (T) es


(a lo sumo) numerable.
Finalmente probemos (iv). Si k , (T) entonces kIT es inversible,
ker(kI T) = 0 y Im(kI T) = X y (iv) es trivial.
Si k (T) 0 entonces (kI T)
[
R
k
es inversible y dimN
k
< .
Entonces
dimker(kI T) = dimker(kI T)
[
N
k
=
= codimIm(kI T)
[
N
k
= codimIm(kI T).

8. TEORA ESPECTRAL DE OPERADORES COMPACTOS 167


Como corolario resaltamos de forma explcita la llamada Alterna-
tiva de Fredholm, que enunciamos ms abajo. Antes de enunciarla,
intentaremos motivarla brevemente.
Pensemos en el sistema de n ecuaciones lineales con n incgnitas
Ax = y
y sea A
t
la matriz traspuesta de A. Ya hemos visto que si pensamos en A
como un operador T
A
: K
n
K
n
entonces A
t
es la matriz asociada al
operador (T
A
)

. Con esta notacin, los siguientes resultados del lgebra


Lineal son bien conocidos
1. El sistema Ax = y tiene una nica solucin para todo y K
n
si y slo si el sistema homogneo asociado Ax = 0 tiene x =
(0, . . . , 0) como nica solucin.
2. El sistema homogneo Ax = 0 tiene una solucin distinta de
(0, . . . , 0) si y slo si el sistema traspuesto A
t
x = 0 tiene una
solucin distinta de (0, . . . , 0). Adems, en ese caso el mxi-
mo nmero de soluciones linealmente independientes de ambos
sistemas es el mismo.
Es fcil ver que este resultado no se puede extender al caso de una
coleccin numerable de ecuaciones lineales con una cantidad numerable
de incgnitas. Por ejemplo, considrese el sistema innito con matriz
asociada dada por
A = (a
i,j
)
i,jN
con a
j+1,j
=
1
j
para todo j y a
i,j
= 0 si i ,= j + 1.
No es difcil ver que el sistema homogneo asociado Ax = 0 slo admite
la solucin x
1
= x
2
= = 0. En cambio, si y = (1, 0, 0, . . .) el
sistema Ax = y no tiene solucin. Adems, el sistema traspuesto A
t
x =
0 admite la solucin (1, 0, 0, . . .). El resultado probado por Fredholm
nos muestra precisamente que el resultado del lgebra Lineal arriba
mencionado s se mantiene si el operador A es de la forma I T, con
T un operador compacto. En concreto tenemos
Teorema 8.23 (Alternativa de Fredholm). Sea X un espacio de
Banach y sea T /(X). Entonces
1. Se verica una y slo una de las siguientes posibilidades
a) Para todo y X existe un nico x X tal que (I
T)(x) = y.
b) Existe 0 ,= x X tal que (I T)(x) = 0. Adems, en este
caso el mximo nmero de soluciones linealmente inde-
pendientes de la ecuacin homognea (I T)(x) = 0 es
nito.
168 8. TEORA ESPECTRAL DE OPERADORES COMPACTOS
2. La ecuacin homognea (I T)(x) = 0 tiene una solucin
distinta de 0 en X si y slo si la ecuacin traspuesta (I
T

)(x

) = 0 tiene una solucin distinta de 0 en X

. Adems
el mximo nmero de soluciones linealmente independientes
de ambas ecuaciones es el mismo.
Observacin 8.24. Sean X y T como en el Teorema y para todo
k K consideremos el operador kT. Este operador es compacto, por
serlo T. Por lo tanto, para la ecuacin
(I kT)(x) = y
se tienen las dos posibilidades enunciadas en el Apartado 1 del Teore-
ma. Ntese que la segunda posibilidad se tiene precisamente cuando
1
k
es un autovalor de T, y que sabemos que el conjunto de autovalores es
a lo sumo numerable.
Veamos cmo se puede aplicar la Alternativa de Fredholm a la res-
olucin de sistemas numerables de ecuaciones lineales y a la resolucin
de ecuaciones integrales, el problema al que se enfrentaba Fredholm y
que motiv el desarrollo de la teora.
Teorema 8.25. Sea A = (a
i,j
)
i,jN
una matriz innita y sea 1
p < . Si p = 1, supongamos que (j) =

i=1
[a
i,j
[ < para todo
j N y que (j) 0 cuando j . Si 1 < p < y
1
p
+
1
q
= 1,
supongamos que

i=1
_

j=1
[a
i,j
[
q
_
p
q
< .
Entonces
1. Se tiene una de las dos alternativas siguientes
a) O bien para todo y
p
existe un nico x
p
tal que
(1 a
1,1
)x
1
a
1,2
x
2
a
1,3
x
3
= y
1
a
2,1
x
1
+ (1 a
2,2
)x
2
a
2,3
x
3
= y
2
a
3,1
x
1
a
3,2
x
2
+ (1 a
3,3
)x
3
= y
3
.
.
.
.
.
.
.
.
.
b) O bien existe 0 ,= x
p
tal que
(1 a
1,1
)x
1
= a
1,2
x
2
+ a
1,3
x
3
+ (16)
(1 a
2,2
)x
2
= a
2,1
x
1
+ a
2,3
x
3
+ (17)
(1 a
3,3
)x
3
= a
3,1
x
1
+ a
3,2
x
2
+ (18)
.
.
.
.
.
.
.
.
. (19)
8. TEORA ESPECTRAL DE OPERADORES COMPACTOS 169
En este caso no pueden existir un nmero innito de vec-
tores x
p
linealmente independientes que veriquen
(16)
2. El mximo nmero de vectores x
p
linealmente independi-
entes que veriquen (16) es igual al mximo nmero de vec-
tores x
q
linealmente independientes que veriquen el sis-
tema traspuesto
(1 a
1,1
)x
1
= a
2,1
x
2
+ a
3,1
x
3
+
(1 a
2,2
)x
2
= a
1,2
x
1
+ a
3,2
x
3
+
(1 a
3,3
)x
3
= a
1,3
x
1
+ a
2,3
x
2
+
.
.
.
.
.
.
.
.
.
Demostracin. Ya hemos visto en el Ejemplo 7.10 que la matriz
A dene un operador compacto T :
p

p
. Es un ejercicio comprobar
que A
t
dene precisamente el operador traspuesto T

:
q

q
. Por
lo tanto no hay ms que aplicar la alternativa de Fredholm.
Veamos el anlogo continuo del ejemplo de arriba, lo que se conoce
como ecuacin integral de Fredholm de segunda clase.
Teorema 8.26. Sea k(, ) L
2
([0, 1] [0, 1]). Entonces
1. Se tiene una y slo una de las dos siguientes alternativas
a) O bien para todo y L
2
[0, 1] existe un nico x L
2
[0, 1]
tal que para casi todo s [0, 1]
x(s)
_
1
0
k(s, t)x(t)dt = y(s)
b) O bien existe 0 ,= y L
2
[0, 1] tal que para casi todo s
[0, 1]
(20) x(s) =
_
1
0
k(s, t)x(t)dy
Adems, en este caso el mximo nmero de x L
2
[0, 1]
linealmente independientes que verican (20) es nito.
2. La ecuacin (20) tiene una solucin distinta de 0 si y slo si
la ecuacin traspuesta
z(s) =
_
1
0
k(t, s)z(t)dt
tiene una solucin distinta de 0 en L
2
[0, 1]
170 8. TEORA ESPECTRAL DE OPERADORES COMPACTOS
Demostracin. Ya hemos visto en el Ejemplo 7.11 que el oper-
ador T : L
2
[0, 1] [0, 1] dado por
T(x)(s) =
_
1
0
k(s, t)x(t)dt
es compacto. Una vez que identiquemos adecuadamente su operador
traspuesto, bastar con aplicar de nuevo la alternativa de Fredholm.

Prcticas sugeridas
Ejercicio 8.1. Sea X un espacio de Banach y T L(X). De-
mostrar que el operador exp(T) :=

n=0
T
n
n!
(est bien denido y) es
inversible. Sugerencia: (exp(T))
1
= exp(T).
Demostrar adems que
(exp(T)) = exp((T))
Ejercicio 8.2 (Lomonosov). Sea X un espacio de Banach de di-
mensin innita y sea T /(X). Demostrar que X posee un subespa-
cio invariante no trivial. [24, Ej. 18 p. 143].
Ejercicio 8.3. Sea X un espacio de Banach y sea T L(X).
Sean
1
, . . . ,
n
autovalores distintos de T y sea e
i
un autovector de
i
(1 i n). Demostrar que los vectores e
1
, . . . , e
n
son linealmente
independientes.
CAPTULO 9
Espacios de Hilbert
Los espacios hoy conocidos como espacios de Hilbert estuvieron
histricamente entre los primeros espacios de Banach estudiados y
pronto se comprendi que ocupan un lugar destacado entre todos el-
los. La diferencia radical entre un espacio de Hilbert y un espacio de
Banach cualquiera es que en un espacio de Hilbert tenemos un gran
conocimiento de la geometra del espacio, y adems esta geometra
coincide con nuestra intuicin geomtrica nito dimensional. Esto se
traduce en una serie de teoremas como el Teorema de Representacin
de Riesz, el Teorema de la Proyeccin Ortogonal y otros que permiten
que trabajar en un espacio de Hilbert sea en general mucho ms cmo-
do que trabajar en otro espacio de Banach. Una de las consecuencias
de esta facilidad en el estudio de los espacios de Hilbert es que muchos
de los grandes teoremas del Anlisis Funcional se pueden probar de
manera mucho ms sencilla en el caso de los espacios de Hilbert. Hemos
incluido algn ejemplo de esta situacin en el desarrollo del captulo y
planteamos otros ejemplos de esto como ejercicios.
Comenzamos estudiando el producto escalar, a continuacin la De-
sigualdad de Schwartz y la denicin de norma asociada a un producto
escalar, lo que nos lleva a la denicin de espacio de Hilbert. Seguida-
mente vemos la Identidad de Polarizacin, que nos muestra como la
norma caracteriza al producto escalar, la crucial nocin de ortogonali-
dad y el Teorema de Pitgoras y la Ley del Paralelogramo, todas ellas
herramientas sencillas de probar pero imprescindibles en la Teora de
espacios de Hilbert.
A continuacin probamos el primero de los resultados grandes del
captulo, probando que la distancia de un punto a un convexo cerrado
en un espacio de Hilbert se alcanza en un nico punto. De ah deducimos
el Teorema de la Proyeccin Ortogonal, lo que nos lleva a estudiar la
complementacin en espacios de Hilbert.
Seguidamente comenzamos el estudio de los conjuntos ortogonales,
lo que nos lleva a probar la Desigualdad de Bessel y el Teorema de
Riesz-Fischer, a denir Bases Hilbertianas y a presentar seguidamente
171
172 9. ESPACIOS DE HILBERT
el desarrollo en serie de Fourier en espacios de Hilbert y la Frmula
de Parseval.
No hemos visto necesario denir explcitamente familias sumables
en todo este desarrollo, sino que comprobamos que ciertos sumatorios
formales que aparecen indexados por conjuntos quizs no numerables
tienen a lo sumo una cantidad numerable de trminos no nulos, por
lo que los podemos reducir a series. Creemos que este enfoque es algo
ms sencillo, pero no descartamos denir familias sumables si con la
experiencia nos pareciera mejor alternativa. Otra opcin desde luego
es limitarse a espacios de Hilbert separables.
A continuacin se puede, aunque no es fundamental para la asig-
natura, denir dimensin hilbertiana y probar que todo espacio de Hil-
bert es (isomtrico a) un espacio
2
(I) para algn conjunto de ndices
I.
Terminamos el captulo con el fundamental Teorema de Riesz y
algunas aplicaciones de la Teora de espacios de Hilbert.
Como ya hemos dicho, los espacios de Hilbert son centrales en la
teora de espacios de Banach, y muchos conceptos denidos y estudia-
dos en esta memoria en el contexto general de los espacios normados re-
sultan mucho ms sencillos si nos restringimos a los espacios de Hilbert.
Resulta por ello una opcin didctica a tener en cuenta el comenzar el
estudio de la asignatura con este captulo (adecuadamente modicado,
claro est, para no presuponer resultados no conocidos) y proceder a
continuacin con el estudio de los espacios de Banach generales. Este
es el esquema seguido en, por ejemplo, [13], donde el autor dice que
el nmero de demostraciones que deben aparecer casi duplicadas por
seguir este esquema es mucho menor de lo que l esperaba.
No nos hemos decidido a seguir ese esquema, pero tampoco descar-
taramos experimentarlo en alguna ocasin nalmente impartiramos
la asignatura desarrollada en esta memoria.
Hemos seguido en la presentacin de este Captulo los libros [13] y
[31].
Espacios de Hilbert
Definicin 9.1. Sea H un espacio vectorial sobre K. Un producto
escalar en H es una funcin
, ) : H H K
que verica
9. ESPACIOS DE HILBERT 173
1. Es denida positiva, es decir, para todo x H
x, x) 0
y
x, x) = 0 si y slo si x = 0
2. Linealidad en la primera variable, es decir, para todos x, y, z
H, , K,
x + y, z) = x, z) + y, z)
3. Es antisimtrica, es decir, para todos x, y H
x, y) = y, x)
4. Como consecuencia de 2 y 3 se tiene que un producto escalar
siempre es conjugado-lineal en la segunda variable, es decir,
para todos x, y, z H, , K,
x, y + z) = x, y) + x, z)
Ntese que si estamos trabajando sobre el cuerpo de los reales, un
producto escalar es una forma bilineal, simtrica y denida positiva.
Ejemplo 9.2. Enumeramos a continuacin algunos ejemplos sen-
cillos de espacios con un producto escalar.
El ejemplo ms elemental es precisamente el producto escalar
en R
n
que nuestros alumnos ya conocen. La versin compleja
de este ejemplo es el producto escalar en C
n
que viene dado
por
(z
i
), (w
i
)) =
n

i=1
z
i
w
i
.
Usando la desigualdad de Hlder se comprueba que la funcin
, ) :
2

2
K
dada por
(z
i
), (w
i
)) =

i=1
z
i
w
i
est bien denida y es un producto escalar en
2
.
Si nuestros alumnos ya conocen la medida de Lebesgue, usando
la desigualdad de Hlder para integrales se comprueba que la
aplicacin
, ) : L
2
[0, 1] L
2
[0, 1] K
174 9. ESPACIOS DE HILBERT
dada por
f, g) =
_
1
0
f(t)g(t)dt
est bien denida y es un producto escalar en L
2
[0, 1].
Sea H el conjunto de las funciones f : [0, 1] K absoluta-
mente continuas en [0, 1] tales que f(0) = 0 y f
t
L
2
[0, 1]. Si
denimos
f, g) =
_
1
0
f
t
(t)g
t
(t)dt
para todo f, g H entonces H es un espacio de Hilbert.
Si bien nuestros alumnos probablemente ya conocern la desigual-
dad de Schwarz, probablemente no est de ms recordarles brevemente
su enunciado y demostracin.
Proposicin 9.3 (Desigualdad de Schwarz). Si , ) es un producto
escalar en H entonces para todo x, y H se tiene que
[x, y)[
2
x, x)y, y)
y se tiene la igualdad si y slo si x e y son linealmente dependientes.
Demostracin. Sean x, y H y sea z = y, y)x x, y)y. En-
tonces
0 z, z) =
= y, y)
2
x, x)y, y)x, y)x, y)x, y)y, y)y, x)+x, y)x, y)y, y) =
= y, y)
_
x, x)y, y) [x, y)[
2
_
Entonces si y = 0 el resultado se sigue trivialmente. Si y ,= 0, entonces,
puesto que y, y) > 0, se tiene que
x, x)y, y) [x, y)[
2
0
y de aqu se sigue la desigualdad.
Si x e y son linealmente dependientes, si uno de ellos es 0, la igualdad
se sigue trivialmente. Si no, existe K tal que x = y, y se ve
fcilmente que se da la igualdad. Recprocamente, si se da la igualdad,
tenemos que, con las notaciones anteriores,
z, z) = 0
de forma que z = 0 y por tanto y, y)x = x, y)y, de donde se sigue que
x e y son linealmente dependientes.
A partir de la desigualdad de Schwarz se ve fcilmente que el pro-
ducto escalar permite denir una norma asociada a l
9. ESPACIOS DE HILBERT 175
Corolario 9.4. Sea H un espacio vectorial en el que est denido
un producto escalar , ). Entonces la funcin
| | : H [0, )
denida como
|x| = (x, x))
1
2
es una norma en H.
Demostracin. La nica dicultad reside en la desigualdad tri-
angular. Observemos que para todos x, y H,
|x + y|
2
= x + y, x + y) = |x|
2
+|y|
2
+ 2'x, y).
Por otro lado, de la desigualdad de Schwarz se sigue que
'x, y) [x, y)[ |x||y|
y por tanto
|x + y|
2
|x|
2
+|y|
2
+ 2|x||y| = (|x| +|y|)
2
de donde se sigue la desigualdad triangular.
Probamos a continuacin la Identidad de Polarizacin para el pro-
ducto escalar. Dicha identidad es cierta en el contexto ms general de
las formas sesquilineales si los espacios son complejos y en de las for-
mas bilineales simtricas si los espacios son reales, y as lo podemos
mencionar a los alumnos, pero puesto que no la usaremos ms que en
el producto escalar, slo la probamos en ese caso.
Lema 9.5 (Identidad de Polarizacin). Sea H un espacio vectorial
complejo con un producto escalar denido en l. Entonces
x, y) =
1
4
_
|x + y|
2
|x y|
2
+ |x + y|
2
|x y|
2
_
y
'x, y) =
1
2
_
|x + y|
2
|x|
2
|y|
2
_
por lo que en el caso real
x, y) =
1
2
_
|x + y|
2
|x|
2
|y|
2
_
Demostracin. Inmediata.
Obsrvese que x, y) = 0 para todo y H si y slo si x = 0; clara-
mente, si x = 0 entonces x, y) = 0 para todo y H. Recprocamente,
si x, y) = 0 para todo y H en particular x, x) = 0 lo que implica
que x = 0.
176 9. ESPACIOS DE HILBERT
Al igual que ocurra en el caso de dimensin nita, la gran ventaja de
la geometra de los espacios de Hilbert es que la existencia de producto
escalar nos permite denir el concepto de ortogonalidad.
Definicin 9.6. Sea H un espacio de Hilbert, x, y H. Decimos
que x e y son ortogonales, y lo escribimos
xy,
si
x, y) = 0.
Decimos que dos subconjuntos A, B H son ortogonales, AB si para
todo a A, b B se tiene que ab.
Los alumnos ya saben del caso nito-dimensional que esta nocin se
corresponde precisamente con la nocin geomtrica de ortogonalidad.
Proposicin 9.7 (Pitgoras). Sea H un espacio de Hilbert. Si
x
1
, . . . , x
n
H son dos a dos ortogonales (es decir x
i
x
j
para todo
i ,= j) entonces
_
_
_
_
_
n

i=1
x
i
_
_
_
_
_
2
=
n

i=1
|x
i
|
2
Demostracin. Aplicando induccin, es fcil ver que slo es nece-
sario probar el caso n = 2. Supongamos que x
1
x
2
. Entonces
|x
1
+x
2
|
2
= x
1
+x
2
, x
1
+x
2
) = x
1
, x
1
)+x
2
, x
2
)+x
1
, x
2
)+x
2
, x
1
) =
x
1
, x
1
) +x
2
, x
2
) = |x
1
|
2
+|x
2
|
2
.

Proposicin 9.8 (Ley del Paralelogramo). Sea H un espacio de


Hilbert, x, y H. Entonces
|x + y|
2
+|x y|
2
= 2
_
|x|
2
+|y|
2
_
Demostracin. Dados x, y se sigue de la identidad de polar-
izacin que
|x + y|
2
= |x|
2
+|y|
2
+ 2'x, y)
y
|x y|
2
= |x|
2
+|y|
2
2'x, y).
Ahora se suman las dos ecuaciones.
9. ESPACIOS DE HILBERT 177
La Ley del Paralelogramo caracteriza de hecho a los espacios de
Hilbert, y proponemos esto como ejercicio.
Los dos siguientes resultados son fundamentales en la Teora de
espacios de Hilbert, y son los que permiten que trabajar en un Hilbert
sea cmodo, en el sentido de que nuestra intuicin geomtrica nito
dimensional se mantiene razonablemente intacta.
Teorema 9.9. Sea H un espacio de Hilbert, K H un conjunto
cerrado convexo no vaco y x H. Entonces existe un nico k
0
K
tal que
|x k
0
| = dist(x, K) := nf
k

K
|x k|.
Demostracin. Supongamos inicialmente x = 0. Buscamos pro-
bar la existencia y unicidad de un k
0
K tal que
|k
0
| = dist(0, K) = nf
k

K
|k|.
Sea d = dist(0, K). Por denicin existe una sucesin (k
n
) K tal
que |k
n
| d. De la Ley del Paralelogramo se sigue que
(21)
_
_
_
_
k
n
k
m
2
_
_
_
_
2
=
1
2
(|k
n
|
2
+|k
m
|
2
)
_
_
_
_
k
n
+ k
m
2
_
_
_
_
2
.
Puesto que K es convexo,
k
n
+k
m
2
K y por tanto
_
_
_
_
k
n
+ k
m
2
_
_
_
_
2
d
2
.
Sea > 0. Entonces existe N N tal que para todo n N |k
n
|
2
<
d
2
+
1
4

2
. Por tanto, para todos m, n N se sigue de (21) que
_
_
_
_
k
n
k
m
2
_
_
_
_
2
<
1
2
(2d
2
+
1
2

2
) d
2
=
1
4

2
.
Por tanto (k
n
)
n
es una sucesin de Cauchy y puesto que H es com-
pleto y K es cerrado sabemos que existe k
0
K tal que k
n
k
0
.
Adems
d |k
0
| = |k
0
k
n
+ k
n
| |k
0
k
n
| +|k
n
| d
y por tanto
|k
0
| = d.
178 9. ESPACIOS DE HILBERT
Ya hemos probado la existencia de k
0
, veamos la unicidad. Supong-
amos que existe k
1
K tal que |k
1
| = d. Por convexidad
k
0
+k
1
2
K y
por tanto
d
_
_
_
_
k
0
+ k
1
2
_
_
_
_

1
2
(|k
0
| +|k
1
|) = d
de forma que
d =
_
_
_
_
k
0
+ k
1
2
_
_
_
_
.
Pero la Ley del Paralelogramo implica que
d
2
=
_
_
_
_
k
0
+ k
1
2
_
_
_
_
2
= d
2

_
_
_
_
k
0
k
1
2
_
_
_
_
2
de donde k
0
= k
1
.
Si x ,= 0, basta aplicar lo anterior a 0 = x x y el cerrado convexo
no vaco xK para obtener un nico xk
0
que cumple las condiciones
pedidas. Entonces es un ejercicio ver que k
0
es el nico elemento de K
en el que se alcanza la distancia.
Los subespacios son siempre trivialmente convexos, por lo que pode-
mos aplicar el teorema anterior al caso de un subespacio cerrado. Pero
en ese caso podemos decir an ms.
Teorema 9.10 (Proyeccin ortogonal). Sea H un espacio de Hil-
bert y M H un subespacio cerrado. Sea x H y sea y
0
M el nico
elemento de M tal que |x y
0
| = dist(x, M). Entonces
x y
0
M.
Recprocamente, si y
0
M es tal que x y
0
M entonces
|x y
0
| = dist(x, M).
Demostracin. Hacemos la demostracin en el caso complejo.
El caso real es esencialmente igual aunque algo ms sencillo y queda
como ejercicio para los alumnos. Supongamos que y
0
M verica
|x y
0
| = dist(x, M). Si y M, entonces y + y
0
M y por tanto,
usando la identidad de polarizacin,
|x y
0
|
2
|x (y
0
+ y)|
2
= |(x y
0
) y|
2
=
= |x y
0
|
2
2'x y
0
, y) +|y|
2
.
Por lo tanto
(22) 2'x y
0
, y) |y|
2
9. ESPACIOS DE HILBERT 179
para todo y M.
Queremos probar que x y
0
, y) = 0. Fijemos y M y llamemos
x y
0
, y) = re

, con r 0, [0, 2). Consideremos para cada


t R
+
el vector te

y M. La ecuacin (22) aplicada a este vector nos


dice que
2'(te

re

) t
2
|y|
2
es decir
2tr t
2
|y|
2
.
Puesto que esto ocurre para todo t R
+
se sigue que r = 0, de
donde
x y
0
, y) = 0
como queramos probar.
Recprocamente, sea y
0
M tal que x y
0
M. Entonces, para
todo y M, tambin y y
0
M y por tanto
x y
0
y y
0
y de ah se sigue, por el Teorema de Pitgoras, que
|x y|
2
= |(x y
0
) + (y
0
y)|
2
=
= |x y
0
|
2
+|y
0
y|
2
|x y
0
|
2
y por tanto
|x y
0
| = dist(y, M).

El Teorema anterior no es el n de la historia. Vemos que dado un


subespacio vectorial M de H existe una aplicacin bien denida
P : H M
que lleva x a y
0
, donde y
0
es el nico elemento de M tal que |xy
0
| =
dist(x, M) y simultneamente es el nico elemento de M tal que
x y
0
M.
Por supuesto, dado que estamos trabajando con espacios norma-
dos, nos gustara que esta aplicacin fuera lineal y continua. Este es el
contenido principal del siguiente teorema, que en realidad es una con-
tinuacin del anterior. Necesitamos previamente un concepto nuevo.
180 9. ESPACIOS DE HILBERT
Definicin 9.11. Sea A H un subconjunto. Se dene el ortogo-
nal de A, y lo llamamos A

, como
A

:= x H tales que xa para todo a A.


Ms adelante, cuando veamos el Teorema de Representacin de
Riesz veremos que esta notacin es coherente con el A

denido
anteriormente.
Teorema 9.12 (Proyeccin ortogonal). Sea H un espacio de Hil-
bert y M H un subespacio vectorial cerrado. Sea P : H M H
la aplicacin denida en el prrafo anterior. Entonces
1. P es lineal y continua, y |P| = 1 si M ,= 0.
2. P
2
= P (pensando P como aplicacin de H en H, para que
P
2
tenga sentido).
3. ker P = M

e Im(P) = M
Demostracin. 1. Sean x
1
, x
2
H, , K. Entonces, para
todo y M se tiene que
x
1
+ x
2
(P(x
1
) + P(x
2
)), y) =
= x
1
P(x
1
), y) + x
2
P(x
2
), y) = 0 + 0 = 0.
Puesto que P(x
1
+ x
2
) es el nico elemento de M que verica la
condicin de ortogonalidad, se tiene que
P(x
1
+ x
2
) = P(x
1
) + P(x
2
).
Para ver que P es continua y de norma 1, sea x H. Entonces
x = (x P(x)) + P(x)
donde P(x) M y x P(x) M

. Por tanto P(x)(x P(x)) y del


Teorema de Pitgoras se sigue que
|x|
2
= |x P(x)|
2
+|P(x)|
2
|P(x)|
2
,
es decir
|P(x)| |x|
y por tanto |P| 1. Por otro lado, considerando 0 ,= x M , P(x) = x
(esto es trivial) y por tanto |P| 1.
2. Como P(y) = y para todo y M, tomando x H tenemos que
P(x) M y por tanto
P
2
(x) = P(P(x)) = P(x)
lo que prueba lo pedido.
9. ESPACIOS DE HILBERT 181
3. Si P(x) = 0 entonces x = x P(x) M

. Recprocamente, si
x M

, sea P(x) = y M el nico vector de M tal que x y M

.
Esto implica en particular que
0 = x y, y) = x, y) y, y) = 0 |y|
2
de donde y = 0.
Para ver que Im(P) = M, puesto que Im(P) M, slo es necesario
observar que P(y) = y para todo y M.
Definicin 9.13. Si M H es un subespacio vectorial cerrado,
a la proyeccin P anteriormente se le denomina Proyeccin Ortogonal
sobre M.
Corolario 9.14. Sea H un espacio de Hilbert, M H un sube-
spacio vectorial. Entonces M est complementado en H. De hecho un
complementario de M en H es precisamente M

.
El Teorema de la Proyeccin Ortogonal es uno de los resultados ms
utilizados en espacios de Hilbert, y uno de los motivos de que dichos
espacios sean mucho ms cmodos que otros espacios de Banach a
la hora de trabajar con ellos. De hecho, el ltimo corolario enunciado
caracteriza a los Hilbert, en el sentido de que cualquier espacio de
Banach X en el que todo subespacio cerrado est complementado es
isomorfo a un espacio de Hilbert [32]
Podemos obtener como fcil corolario una versin dbil en espacios
de Hilbert del Teorema Bipolar
Corolario 9.15. Si M H es un subespacio vectorial cerrado,
entonces (M

= M.
Demostracin. Si P es la proyeccin ortogonal sobre M, en-
tonces es fcil ver que I P es la proyeccin ortogonal sobre M

.
Por tanto, del apartado 3 del Teorema de la Proyeccin ortogonal se
sigue que
(M

= ker I P.
Pero x ker I P si y slo si P(x) = x, es decir, x M.
Aplicacin: Como aplicacin de la proyeccin ortogonal podemos
mostrar una construccin elemental de la esperanza condicional.
Sea (, T, P) un espacio de probabilidad (es decir un espacio con
una -lgebra T denida en l y una probabilidad P : T [0, 1]).
Supongamos para jar ideas que = [0, 1], T = B, la -lgebra de
Borel y P = es la medida de Lebesgue.
182 9. ESPACIOS DE HILBERT
Para todo 1 p sea L
p
([0, 1], B, ) el espacio de Lebesgue
habitual.
Consideremos una sub--lgebra ( B. Tiene entonces sentido
hablar de L
p
([0, 1], (, ), como el espacio de las funciones (-medibles
con | |
p
nita. Es fcil ver que L
p
([0, 1], (, ) es un subespacio cerrado
de L
p
([0, 1], B, ). Se puede ver que de hecho L
p
([0, 1], (, ) est comple-
mentado en L
p
([0, 1], B, ), y que podemos una denir una proyeccin
E([() : L
p
([0, 1], B, ) L
p
([0, 1], (, )
que denominaremos esperanza condicional con propiedades especiales
que lo hacen muy importante en la Teora de Probabilidad y Estadsti-
ca. Las propiedades interesantes de la esperanza condicional son conse-
cuencia de ser una proyeccin que verica que para todo G ( y para
todo f L
p
([0, 1], B, )
(23)
_
G
E(f[() =
_
G
fd
La interpretacin probabilstica que se suele hacer de esto es la sigu-
iente. La -lgebra ( (o ms bien las funciones (-medibles) representa
lo que sabemos, o ms correctamente los observables en unas circun-
stancias dadas, o un instante dado. Entonces E(f[() representa lo
mejor que podemos decir acerca de una funcin f con los conocimien-
tos representados en (.
La construccin de la Esperanza Condicional se hace habitualmente
utilizando el Teorema de Radon-Nikodym. Vamos a ver una construc-
cin elemental.
Consideramos primeramente el caso p = 2. En ese caso, es un ejer-
cicio ver que la proyeccin ortogonal es una proyeccin que verica
(23).
Recordemos que si p q, L
p
[0, 1] L
q
[0, 1]. Por tanto para todo
p > 2 podemos denir el operador esperanza condicional simplemente
restringiendo la esperanza condicional en L
2
. De nuevo es fcil ver que
el operador as denido termina en L
p
([0, 1], (, ) y verica (23).
El problema viene cuando tratamos de extender la esperanza condi-
cional a todo L
1
. No incluimos todos los detalles, que requieren de un
uso ms o menos cuidadoso de tcnicas estandar de Teora de la Medi-
da. El lector interesado puede consultar por ejemplo [35].
Quizs sea interesante mencionar que siguiendo en esta direccin se
puede, a partir de esta denicin de esperanza condicional y utilizando
un teorema de convergencia de martingalas, demostrar el Teorema de
9. ESPACIOS DE HILBERT 183
Radon-Nikodym, cerrando as el crculo de ideas. Los detalles se pueden
ver por ejemplo en [28].
Conjuntos ortogonales
Buscamos ahora ver la versin innito-dimensional de la nocin, ya
conocida por los alumnos en dimensin nita, de base ortonormal de un
espacio de Hilbert. Puesto que previsiblemente no trataremos bases de
Schauder en el programa de la asignatura, damos aqu una presentacin
autocontenida. Si hubiramos visto bases de Schauder en espacios de
Banach, entonces relacionaramos esta seccin con aquellas.
Definicin 9.16. Un subconjunto E H se dice ortogonal si para
todo x, y E, x ,= y, se tiene que
xy.
Si adems |x| = 1 para todo x X decimos que X es ortonormal.
Proposicin 9.17. Sea E H un subconjunto ortogonal. Si 0 , E
entonces E es linealmente independiente. Si adems E es ortonormal
entonces
|x y| =

2
para todo x, y E con x ,= y.
Demostracin. Sean x
1
, . . . , x
n
E y sean
1
, . . . ,
n
K tales
que
n

i=1

i
x
i
= 0.
Entonces, para todo 1 j n se tiene
0 = 0, x
j
) =
n

i=1

i
x
i
, x
j
) =
n

i=1

i
x
i
, x
j
) =
j
x
j
, x
j
).
Por tanto, puesto que x
j
,= 0 se tiene que
j
= 0 para todo 1 j
n y E es linealmente independiente. Si E es ortonormal entonces para
todo x ,= y E
|x y|
2
= x y, x y) = x, x) +y, y) = 2.

184 9. ESPACIOS DE HILBERT


Puesto que los sistemas ortonormales son linealmente independi-
entes, dado un sistema ortonormal podemos, utilizando el lgebra Lin-
eal, extender dicho sistema a una base de Hamel de H. Sin embargo
habitualmente las bases de Hamel no son un objeto de trabajo intere-
sante en la Teora de Espacios Normados, y en particular en la Teora
de Espacios de Hilbert, puesto que no sacan partido de la estructura
topolgica y esto las hace ser demasiado grandes: ya vimos en el Ejer-
cicio 4.5 que los espacios de Banach innito dimensionales no pueden
tener bases de Hamel numerables.
En la Teora de Espacios Normados de dimensin Innita el obje-
to interesante son las bases de Schauder, y en la Teora de Espacios
de Hilbert son las bases ortonormales que iremos construyendo en los
resultados siguientes.
Vamos a empezar viendo el proceso de ortogonalizacin de Gram-
Schmidt que nos dice que, dado un conjunto contable linealmente inde-
pendiente de H podemos construir inductivamente un conjunto ortonor-
mal que genera el mismo subespacio vectorial en cada paso.
Teorema 9.18 (Proceso de Ortonormalizacin de Gram-Schmidt).
Sea H un espacio de Hilbert y x
n
; n N H un subconjunto lineal-
mente independiente. Entonces existe un conjunto ortonormal e
n
; n
N tal que, para todo n N,
span[e
1
, . . . , e
n
] = span[x
1
, . . . , x
n
]
Demostracin. Denimos inductivamente la sucesin (e
n
)
n
. En
realidad vamos deniendo una sucesin ortogonal (y
n
) y a continuacin
la normalizamos. En primer lugar,
y
1
= x
1
y
e
1
=
y
1
|y
1
|
.
Obviamente span[e
1
] = span[x
1
]
Supongamos que ya hemos denido los n 1 primeros trminos de
la sucesin. Denimos entonces
y
n
= x
n
x
n
, e
1
)e
1
x
n
, e
n1
)e
n1
y
e
n
=
y
n
|y
n
|
.
Todo es como en el caso nito dimensional que ya conocen as que
dejamos como ejercicio la conclusin de la demostracin.
9. ESPACIOS DE HILBERT 185
Ejemplo 9.19. 1. En
2
, la base cannica (e
n
) es ortonor-
mal.
2. Si en
2
ortonormalizamos la base sumante (x
n
)
n
con x
n
=
(1,
(n)
, 1, 0, 0, . . .) obtenemos la base cannica.
3. Si ortonormalizamos la sucesin 1, x, x
2
, . . . , en L
2
[0, 1] obten-
emos la sucesin
e
n
=
_
2n + 1
2
P
n
(x)
donde los polinomios
P
n
(x) =
1
2
n
n!
d
n
dx
n
(x
2
1)
son los Polinomios de Legendre.
Enunciamos y probamos a continuacin la desigualdad de Bessel en
su forma habitual, es decir para conjuntos ortonormales numerables.
Ms adelante vemos como corolario que la desigualdad tambin es cierta
para conjuntos ortonormales arbitrarios.
Teorema 9.20 (Desigualdad de Bessel). Sea H un espacio de Hil-
bert, e
n
; n N H un conjunto ortonormal y x H. Entonces

n=1
[x, e
n
)[
2
|x|
2
.
Adems se tiene la igualdad si y slo si
x =

n=1
x, e
n
)e
n
.
Demostracin. Fijemos n N y sea x
n
= x

n
k=1
x, e
k
)e
k
. De
la ortogonalidad de los (e
n
) se sigue que para todo n N y para todo
1 k n,
x
n
e
k
.
Puesto que x = x
n
+

n
k=1
x, e
k
)e
k
, del teorema de Pitgoras se
tiene que
|x|
2
= |x
n
|
2
+
n

k=1
|x, e
k
)e
k
|
2
= |x
n
|
2
+
n

k=1
[x, e
k
)[
2
de lo que se sigue que
n

k=1
[x, e
k
)[
2
|x|
2
.
186 9. ESPACIOS DE HILBERT
Puesto que esto es cierto para un n N arbitrario, se tiene la
desigualdad buscada.
Si se tiene la igualdad, esto implica que
|x
n
|
2
0
y de aqu se sigue que
x = lm
n
n

k=1
x, e
k
)e
k
=

k=1
x, e
k
)e
k
Recprocamente, si x =

k=1
x, e
k
)e
k
entonces |x
n
| 0 y se sigue
la igualdad buscada.
Corolario 9.21. Si E H es un conjunto ortonormal y x H
entonces x, e) = 0 excepto en a lo sumo una cantidad numerable de
elementos e E.
Demostracin. Para todo n N, sea
E
n
:= e E tales que x, e)
1
n

Por la desigualdad de Bessel E


n
es nito para cada n, y

nN
E
n
= e E tales que x, e) , = 0.

Corolario 9.22 (Desigualdad de Bessel). Sea H un espacio de


Hilbert, E H un conjunto ortonormal y x H. Entonces

eE
[x, e)[
2
|x|
2
.
Estudiamos ahora la convergencia de las series

n
k
n
e
n
donde los
k
n
son escalares y e
n
; n N forman un sistema ortonormal.
Teorema 9.23 (Riesz-Fischer). Sea H un espacio de Hilbert, sea
e
n
; n N H un sistema ortonormal y sea (k
n
)
nN
K. Si

n
k
n
e
n
converge a x H entonces, para todo n N,
x, e
n
) = k
n
y

n
[k
n
[
2
< .
Recprocamente, si

n
[k
n
[
2
< entonces

n
k
n
e
n
9. ESPACIOS DE HILBERT 187
converge en H.
Demostracin. Si x =

n
k
n
e
n
entonces de la ortogonalidad de
los (e
n
) se sigue que x, e
n
) = k
n
para todo n N y la desigualdad de
Bessel implica que

n
[k
n
[
2
< .
Recprocamente, para todo m N sea
x
m
=
m

n=1
k
n
e
n
.
Entonces para todo 1 j m se tiene que
x
m
x
j
=
m

n=j+1
k
n
e
n
y de nuevo la ortonormalidad de los (e
n
) implica que
|x
m
x
j
|
2
= x
m
x
j
, x
m
x
j
) =
m

n=j+1
[k
n
[
2
.
Por tanto, si

n=1
[k
n
[
2
< se sigue que la sucesin (x
m
)
m
es de
Cauchy y por tanto convergente.
Con la desigualdad de Bessel y el Teorema de Riesz-Fischer estamos
ya bastante cerca de la nocin de base Hilbertiana o base ortonormal.
Definicin 9.24. Un sistema ortonormal E H es una base
ortonormal, o base Hilbertiana, si es un sistema ortonormal maxi-
mal para el orden dado por la inclusin. Es decir, si existe un sistema
ortonormal E
t
tal que E E
t
entonces E = E
t
.
Podemos garantizar la existencia de bases ortonormales mediante
el Lema de Zorn (3.3).
Proposicin 9.25. Sea H ,= 0 un espacio de Hilbert. Entonces
existe una base ortonormal en H. De hecho, dado un sistema ortonor-
mal E
t
H, E
t
se puede ampliar a una base ortonormal E.
Demostracin. Notemos en primer lugar que si 0 ,= x H,
el sistema
x
|x|
es trivialmente ortonormal. Veamos que un sistema
ortonormal E
t
se puede extender a una base ortonormal. Sea c el con-
junto de todos los sistemas ortonormales de H que contienen a E
t
. Este
es un conjunto dirigido por la inclusin conjuntista. Si consideramos un
subconjunto totalmente ordenado (una cadena) es fcil ver que la unin
de todos los elementos de la cadena es una cota superior de esta. Por
188 9. ESPACIOS DE HILBERT
tanto el Lema de Zorn nos dice que existe un elemento maximal de c
al que llamamos E. Claramente E
t
E y E es base ortonormal.
Necesitaremos ahora un lema que nos permite reducir las sumas
que aparecen en los sistemas ortonormales a series.
Lema 9.26. Sea E = e

; A H un sistema ortonormal y
x H. Sea
E
x
:= e

E tales que x, e

) , = 0.
Entonces E
x
es un conjunto numerable y llamando E
x
= e
n
; n N
se tiene
lm
n
x, e
n
) = 0.
Adems

n
x, e
n
)e
n
converge a un elemento y H tal que
x ye

para todo A.
Demostracin. Si x = 0 no hay nada que demostrar. Sea en-
tonces x ,= 0. Que E
x
es contable es el Corolario 9.21. Llamando
E
x
= e
n
; n N de la desigualdad de Bessel se sigue que

n
[x, e
n
)[
2
|x|
2
<
y por tanto [x, e
n
)[
2
0.
Puesto que ya hemos visto que

n
[x, e
n
)[
2
< , del Teorema de
Riesz-Fischer (9.23) se sigue que

n
x, e
n
)e
n
converge a un elemento y H. Adems, para todo A,
y, e

) =
_

n
x, e
n
)e
n
, e

_
=

n
x, e
n
)e
n
, e

) = x, e

)
y de ah se sigue fcilmente que
x ye

para todo A.

Teorema 9.27. Sea E = e

; A H un sistema ortonormal.
Entonces son equivalentes
(1) E es una base ortonormal de H
9. ESPACIOS DE HILBERT 189
(2) (Desarrollo en serie de Fourier) Para todo x H
x =

A
x, e

)e

entendiendo este sumatorio como una serie, puesto que a lo


sumo una cantidad numerable de trminos son no nulos, es
decir
x =

n
x, e
n
)e
n
donde e
n
; n N = e

E tales que x, e

) , = 0.
(3) (Frmula de Parseval) Para todo x H
|x|
2
=

A
[x, e

)[
2
entendiendo este sumatorio igual que en el apartado anterior.
(4) span[E] es denso en H
(5) Si x H es tal que x, e

) = 0 para todo A entonces


x = 0.
Demostracin. Supongamos que (1) es cierto, es decir E es una
base ortonormal, es decir un sistema ortonormal maximal. Sea x H.
Por el Lema 9.26 existe y H tal que

x, e

)e

= y
y adems
x yE.
Si y ,= x, sea
e =
x y
|x y|
.
Entonces |e| = 1 y eE por lo que E e es un sistema ortonormal,
en contradiccin con que E sea maximal. Por tanto
x = y =

x, e

)e

y (2) es cierto.
La equivalencia de (2) y (3) se sigue de la condicin descrita para
la igualdad en la desigualdad de Bessel (Teorema 9.20).
Fcilmente (2) implica (4), ya que

m
n=1
x, e
n
)e
n
span[E].
190 9. ESPACIOS DE HILBERT
Para ver que (4) implica (5), sea x H tal que x, e

) = 0 para
todo A, y sea (x
n
)
nN
span[E] una sucesin tal que x
n
x.
Entonces x
n
, x) = 0 para todo n y por tanto
0 = x
n
, x) x, x)
de donde se sigue que x = 0
Finalmente, para ver que (5) implica (1), sea E
t
H un sistema
ortonormal tal que E E
t
. Sea e E
t
E. De la ortogonalidad de E
t
se sigue que e, e

) = 0 para todo A. De (5) se sigue que e = 0, una


contradiccin puesto que si E
t
es ortonormal y por tanto |e| = 1.
Podemos ahora denir un concepto de dimensin Hilbertiana, de
forma anloga a la denicin de la dimensin algebraica.
Proposicin 9.28. Si H es un espacio de Hilbert, todas sus bases
ortonormales tienen el mismo cardinal.
Demostracin. Sean E y F dos bases ortonormales de H, sean
= card(E), = card(F). Si o son nitos, entonces = , usando
razonamientos totalmente anlogos a los nito-dimensionales. Supong-
amos entonces que ambos son innitos. Dado e E, sea
F
e
:= f F tales que e, f) , = 0.
Ya hemos visto que F
e
es contable. Adems todo f F debe de
pertenecer al menos a uno de los F
e
, puesto que si no e, f) = 0 para
todo e E y de la condicin (5) del Teorema 9.27 se seguira que f = 0.
Por tanto
F =
eE
F
e
y por tanto

0
=
Anlogamente se puede ver que

0
=
de donde se sigue que = .
Ya podemos denir la dimensin hilbertiana.
Definicin 9.29. Sea H un espacio de Hilbert. Se dene su di-
mensin hilbertiana como el cardinal de una cualquiera de sus bases
ortonormales.
Proposicin 9.30. Sea H un espacio de Hilbert. Entonces H es
separable si y slo si dimH =
0
.
9. ESPACIOS DE HILBERT 191
Demostracin. Si dimH =
0
, y E = e
n
; n N, es un ejercicio
ver que el conjunto numerable
_
m

n=1
q
n
e
n
; q
n
Q+ Q, m N
_
es denso en H.
Recprocamente, sea E una base ortonormal de H. Ya vimos que
para todo e
1
, e
2
E
|e
1
e
2
|
2
= |e
1
|
2
+|e
2
|
2
= 2.
Por tanto, el conjunto
B
e,
1

2
; e E
es un conjunto de bolas abiertas de H disjuntas dos a dos. De aqu se
sigue que este conjunto, y por tanto E, deben de ser numerables. El
razonamiento es el siguiente:
Si (X, d) es un espacio mtrico separable y B
i
= B
x
i
,r
i
; i I es
un conjunto de bolas abiertas disjuntas dos a dos, entonces I tiene que
ser contable. En efecto, sea D X un subconjunto contable denso.
Entonces para todo i I,
B
i
D ,=
y por tanto existe y
i
B
i
D. Por tanto y
i
; i I es un subconjunto
de D con el cardinal de I. De aqu se sigue que
cardI cardD =
0
.

Ejemplo 9.31. Sea H =


2
y sea E = e
n
; n N
2
la base
cannica. X es un sistema ortonormal, y dado x
2
, si
x, e
n
) = 0
para todo n N se sigue que x = 0. Por tanto E es una base ortonor-
mal, y queda justicado el nombre de base cannica, y tenemos que
2
es separable.
En la categora de espacios de Hilbert, el concepto natural de iso-
morsmo es aquel que preserva el producto escalar junto con la es-
tructura lineal. Es decir, una aplicacin lineal que preserve productos
escalares. Veamos que los isomorsmos en este sentido son exactamente
las isometras.
192 9. ESPACIOS DE HILBERT
Proposicin 9.32. Sean H, K espacios de Hilbert y T : H K
una aplicacin lineal. Entonces T es una isometra si y slo si para
todo x, y H
T(x), T(y)) = x, y).
Demostracin. Si T(x), T(y)) = x, y) entonces
|T(x)|
2
= T(x), T(x)) = x, x) = |x|
2
y T es una isometra.
Recprocamente, si T es una isometra, se sigue de la identidad de
polarizacin que se conserva el producto escalar.
Decimos que dos espacios de Hilbert H, K son isomorfos como es-
pacios de Hilbert si son isomtricos como espacios de Banach, es decir,
si existe una isometra T : H K sobreyectiva.
Teorema 9.33. Dos espacios de Hilbert H, K son isomorfos como
espacios de Hilbert si y slo si tienen la misma dimensin.
Demostracin. Si T : H K es una isometra sobre y E es una
base de H, entonces T(E) = T(e); e E es un sistema ortonormal
de K. Adems, dado y K, sea x H el nico elemento de H tal
que T(x) = y. Si T(e), y) = 0 para todo T(e) T(E) tenemos que
T(e), T(x)) = e, x) = 0 para todo e E. Puesto que E es una base
ortonormal, esto implica que x = 0 y por tanto y = 0 y se tiene que
T(E) es una base ortonormal por el Teorema 9.27. De ah se sigue que
dimH = dimK.
Recprocamente, sea H es un espacio de Hilbert y E H una base
ortonormal. Consideramos el espacio

2
(E) = (a
e
)
eE
tales que

e
[a
e
[
2
<
De manera totalmente anloga a como vimos que
2
es un espacio
de Hilbert, se comprueba que
2
(E) es un espacio de Hilbert con el
producto escalar obvio. (En algn momento de la demostracin hay que
observar, como ya hemos hecho anteriormente en ms de una ocasin,
que si

e
[a
e
[
2
< entonces a lo sumo una cantidad numerable de
coordenadas a
e
son distintas de 0.)
Denimos entonces la aplicacin
T : H
2
(E)
como
T(x) = (x, e))
eE
9. ESPACIOS DE HILBERT 193
La linealidad de T est clara. De la desigualdad de Bessel, o del Teore-
ma de Parseval, se sigue que T est bien denido y que es una isometra.
Para ver que es sobre, si (a
e
)
eE

2
(E), entonces del Teorema de de
Riez-Fischer (9.23) se sigue que la serie

eE
a
e
e
(serie porque slo una cantidad numerable de trminos son no nulos)
converge en H a un elemento x. Es claro entonces que T(x) = (a
e
).
Luego hemos probado que todo espacio de Hilbert es isomorfo a

2
(I) donde I es un conjunto tal que card(I) = dimH, y de ah se
sigue lo pedido.
Observacin 9.34. Ntese que hemos probado algo ms que lo
anunciado; hemos probado que de hecho todo espacio de Hilbert es (iso-
morfo a) un espacio
2
(I), y que tiene sentido hablar de las coordenadas
de un vector de forma al menos similar a las coordenadas de los espa-
cios vectoriales nito dimensionales.
Corolario 9.35. Dos espacios de Hilbert separables son isomorfos
(e isomorfos a
2
)
Ahora podemos estudiar al Anlisis de Fourier dentro del marco
terico que hemos preparado.
Tenemos el siguiente teorema, que se puede probar directamente
desde la Variable Compleja, aunque nosotros lo probamos aqu como
consecuencia del Teorema de Stone-Weierstrass.
Teorema 9.36. Sea T = z C tales que [z[ = 1 y sea
f : T C
una funcin continua. Entonces existe una sucesin (P
n
(z, z))
nN
de
polinomios en z y z tales que P
n
(z, z) tiende a f uniformemente en T.
Demostracin. T es un espacio compacto. Consideramos el es-
pacio de Banach C(T) que adems tiene estructura de lgebra. Sea
A C(T) el lgebra de los polinomios en z y z. Es un ejercicio ver-
icar que A cumple las hiptesis del Teorema de Stone-Weierstrass y
por tanto A = C(T).
Observemos que para todo z T, z = z
1
y z = e

y por tanto
todo polinomio P(z, z) se puede escribir como una funcin de z = e

dada por
P(z) =
n

k=n

k
e
k
194 9. ESPACIOS DE HILBERT
Por este motivo a estos polinomios se les llama polinomios trigonomtri-
cos. Veamos ahora que el conjunto
_
e
n

2
; n Z
_
L
2
[0, 2]
es una base ortonormal de L
2
[0, 2].
Es un ejercicio elemental de variable compleja vericar que dicho
sistema es ortonormal.
Para vericar que es una base hay al menos dos posibilidades, ambas
no triviales. Una es seguir por ejemplo [13, Theorem 5.7] y probarlo
utilizando que C[0, 2] es denso en L
2
[0, 2] y a continuacin utilizar
la caracterizacin, no probada pero fcilmente probable como ejercicio
que dice que un sistema ortonormal E H es una base ortonormal
si y slo si la clausura topolgica de span[E] = H. La otra opcin es
seguir [31, p. 392], pero para esto es necesario utilizar el Teorema de
Fejer, lo cual quiz nos lleve demasiado lejos.
Una vez que sabemos que
_
e
n

2
; n Z
_
es una base ortonormal
podemos asignar a cada funcin f L
2
[0, 2] sus coordenadas respecto
de dicha base.
Definicin 9.37. Sea f L
2
[0, 2]. Denimos su n-simo coe-
ciente de Fourier como

f(n) = f,
e
n

2
) =
1

2
_
2
0
f(t)e
nt
dt.
Por los resultados anteriores, lo que hemos probado es que
f =
1

2
+

n=

f(n)e
n
,
es decir que toda funcin f L
2
[0, 2] admite un desarrollo en serie de
Fourier que converge a f en norma 2. Queda probado como corolario
tambin la Identidad de Parseval clsica. La cuestin de la convergencia
en casi todo punto de la serie de Fourier de f a f es ms na y se
mete ms en el Anlisis Armnico. Es un resultado profundo que si
1 < p y f L
p
[0, 2] entonces el desarrollo en serie de Fourier
de f (que existe) converge a f en casi todo punto. Ya hemos visto que
existen funciones f L
1
[0, 2] tal que su desarrollo en serie de Fourier
en casi todo punto no converge a f.
Queda tambin probada como corolario de los resultados anteri-
ores la versin en L
2
del lema de Riemann-Lebesgue: Si f L
2
[0, 2]
entonces sus coecientes de Fourier convergen a 0.
9. ESPACIOS DE HILBERT 195
Tambin como corolario queda probado no slo que L
2
[0, 2] es
separable, sino que la transformada de Fourier dene una isometra
sobreyectiva entre L
2
[0, 2] y
2
.
Teorema de Riesz
El Teorema de Representacin de Riesz es otro de los resultados
fundamentales de la Teora de Espacios de Hilbert. Nos dice que los
espacios de Hilbert no slo son reexivos, sino que de hecho pode-
mos identicar de manera natural H y H

. Como forma de jar la


intuicin aqu, podemos recordar al alumno que ya hemos visto que

2
=
2
, y acabamos de ver que todo espacio de Hilbert es un
2
(I)
para algn conjunto de ndices I. Recordemos que hay otro Teorema
de Representacin de Riesz, con el que no nos debemos confundir, el
que representa el dual de C[0, 1].
Teorema 9.38 (de Representacin de Riesz). Sea H un espacio de
Hilbert y f H

. Entonces existe un nico y H tal que


f(x) = x, y)
para todo x H. Adems |f| = |y|. De hecho, si 0 ,= z H es un
elemento ortogonal a ker f entonces
y =
f(z)z
z, z)
.
Demostracin. Lo demostraremos utilizando el Teorema de la
Proyeccin Ortogonal, siguiendo [31, 24.3] o [13, 3.4]. Existe otra de-
mostracin posible utilizando bases ortonormales
Si f = 0 entonces y = 0 cumple lo pedido. Sea entonces f ,= 0.
Puesto que f es continuo, M := ker f es un subespacio cerrado de
H. Al ser f ,= 0, M ,= H y por tanto M

,= 0. Por tanto, existe


0 ,= z M

. Sea x H. Por ser M un hiperplano, x se puede escribir


como
x = w + kz,
donde w M y k K.
Entonces
x, z) = w, z) + kz, z) = kz, z)
de forma que
k =
x, z)
z, z)
196 9. ESPACIOS DE HILBERT
y por tanto
f(x) = f(w) + kf(z) = kf(z) =
x, z)
z, z)
f(z) =
_
x,
f(z)
z, z)
z
_
y por tanto
y =
f(z)
z, z)
z
cumple lo pedido.
Para ver la unicidad de y, sea y
t
H otro vector de H tal que
f(x) = x, y
t
).
Entonces considerando x = y y
t
tenemos que
y y
t
, y) = f(y y
t
) = y y
t
, y
t
)
y por tanto
y y
t
, y y
t
) = 0,
es decir, y = y
t
.
El Teorema de Representacin de Riesz nos proporciona de hecho
una isometra sobre y conjugada-lineal de H en H

y nos permite dotar


a H

de estructura de espacio de Hilbert.


Teorema 9.39. Sea H un espacio de Hilbert y sea : H

H
la aplicacin que a todo f H

le asigna el nico y
f
H que verica
que
f(x) = x, y
f
)
para todo x H. Entonces es una isometra sobreyectiva y conjugada-
lineal.
Demostracin. Dados f, g H

, para todo x H se tiene que


(f + g)(x) = f(x) + g(x) = x, y
f
) +x, y
g
) = x, y
f
+ y
g
)
y por tanto
(f + g) = y
f
+ y
g
= (f) + (g).
Anlogamente, dados f H

y k K para todo x H se tiene


que
kf(x) = kf(x) = kx, y
f
) = x, ky
f
)
y por tanto
(kf) = ky
f
= k(f).
Por tanto es conjugado-lineal. Para ver que es sobre, sea y H y
sea f : H K la funcin denida como
f(x) = x, y)
9. ESPACIOS DE HILBERT 197
para todo x H. Entonces f H

y (f) = y. Para ver que es una


isometra,
|f| = sup
xB
H
[f(x)[ = sup
xB
H
[y
f
, x)[ =
_
y
f
,
y
f
|y
f
|
_
=
=
y
f
, y
f
)
|y
f
|
=
|y
f
|
2
|y
f
|
= |y
f
|.

Por tanto podemos denir en H

un producto escalar simplemente


transportando el producto escalar de H de manera que H

tambin es
un espacio de Hilbert.
Teorema 9.40. Con la notacin del teorema anterior, sean f, g
H

. Denimos
f, g) = (f), (g)).
Entonces as denida, , ) es un producto escalar en H

tal que |f|


2
=
f, f) para todo f H

y por tanto H es un espacio de Hilbert.


Demostracin. Trivial.
Veamos nalmente que todo espacio de Hilbert es reexivo.
Corolario 9.41. Todo espacio de Hilbert es reexivo.
Demostracin. Sea J : H H

la aplicacin cannica dada


por
J(x)(f) = f(x).
Sea : H

H la isometra sobre descrita ms arriba y


t
: H

la isometra respectiva construida sobre los duales. Tenemos simple-


mente que ver que J es sobre. Sea x

H. Consideramos el elemento
x = (
t
(x

) H. Veamos que J(x) = x

. Para todo f H

(f) = f,
t
(x

)) = (
t
(x

), (f)) = x, (f)) = f(x) = J(x)(f)


de donde se sigue lo pedido.
Para terminar este captulo, podemos ver cmo en el caso particular
de espacios de Hilbert tanto el Teorema de Hahn-Banach como la con-
secuencia del Principio de Acotacin Uniforme que dice que dbilmente
acotado implica acotado se pueden probar de una manera ms sencilla,
incluso diciendo ms que en al caso general. En el camino estudiaremos
la convergencia dbil de sucesiones en espacios de Hilbert, uno de los
problemas que justicaron la introduccin de estos espacios por Hil-
bert. Hemos incluido el resultado de extensin en el cuerpo del texto
y los otros resultados en los ejercicios, pero todo ello es susceptible de
modicacin.
198 9. ESPACIOS DE HILBERT
Teorema 9.42 (Extensin de Hahn-Banach con unicidad). Sea H
un espacio de Hilbert, G H un subespacio (no necesariamente cer-
rado) y g : G K una forma lineal y continua. Entonces existe una
nica extensin f : H K de g tal que
f
[
G
= g y |f| = |g|.
Demostracin. Sea M = G. Entonces g se extiende por con-
tinuidad de forma nica a una funcin que seguimos llamando g :
M K que sigue siendo lineal y continua. Esto es un ejercicio.
Puesto que M es un espacio de Hilbert, sabemos que existe y
g
M tal
que, para todo x M
g(x) = x, y
g
)
Sea entonces f : H K el elemento de H

dado por
f(x) = x, y g) para todo x H
Entonces f extiende a g y |f| = |y
g
| = |g|.
Para probar la unicidad, sea h H

otra extensin de g con la


misma norma. Sea y
h
el elemento de H representante de h. Entonces
|y
h
| = |h| = |g| = |y
g
|
y
y
g
, y
h
) = h(y
g
) = g(y
g
) = y
g
, y
g
) = |y
g
|
2
Por lo tanto, la Identidad de Polarizacin nos dice que
|y
g
y
h
|
2
= |y
g
|
2
2'y
g
, y
h
) +|y
h
|
2
= 2|y
g
|
2
2'y
g
, y
h
) = 0
y por tanto y
g
= y
h
y de ah f = h.
Aplicaciones.
Teorema Isoperimtrico: Como consecuencia de la Frmula de
Parseval para el sistema trigonomtrico real se tiene
Teorema 9.43. De todas las curvas simples, cerradas y diferen-
ciables a trozos de longitud L en el plano, el crculo es la que encierra
mayor rea.
Una demostracin de este resultado se puede ver en [22, p. 178].
Teorema de Mntz: En [22, p. 181] se puede ver una demostracin
del siguiente hecho
9. ESPACIOS DE HILBERT 199
Teorema 9.44. Sea 1 n
1
< n
2
< una sucesin de nmeros
naturales y sea A = t
n
1
, t
n
2
, . . . L
2
[0, 1]. Entonces span[A] es denso
en L
2
si y slo si

i=1
1
n
i
= .
Prcticas sugeridas.
Ejercicio 9.1. Si queremos enfatizar la relacin entre el Anlisis
Funcional y la Variable compleja podemos proponer el siguiente ejerci-
cio [13, Prop. 1.13, p. 6].
Sea G C un abierto y sea L
2
a
(G) el espacio de las funciones
analticas f : G C tales que
_ _
G
[f(x + y)[
2
dxdy <
Vericar que es un espacio de Hilbert con el producto escalar hereda-
do. La nica dicultad reside en vericar que L
2
a
(G) es un subespacio
cerrado de L
2
(G, ) donde es la medida de Lebesgue en C.
Ejercicio 9.2. Demostrar que la Ley del Paralelogramo caracteriza
a los espacios de Hilbert, es decir, probar que si | | es una norma en
un espacio de Banach X que satisface la identidad del paralelogramo,
esto es, para todos x, y X
|x + y|
2
+|x y|
2
= 2
_
|x|
2
+|y|
2
_
entonces dicha norma proviene de un producto escalar, y por tanto X
es un espacio de Hilbert. Sugerencia. Denir
x, y) =
1
4
(|x + y|
2
|x y|
2
+ |x + y|
2
|x y|
2
).
An es necesario dar alguna sugerencia ms para acabar el ejercicio.
Ejercicio 9.3. Utilizando la Ley del paralelogramo, probar que
p
o L
p
es un espacio de Hilbert si y slo si p = 2.
Ejercicio 9.4. Sea e
1
, . . . , e
n
H un sistema ortonormal y sea
M = span[e
1
, . . . , e
n
]. Si P : H M es la proyeccin ortogonal
entonces para todo h H
P(h) =
n

k=1
h, e
k
)e
k
.
Ejercicio 9.5. Sea f L
2
[0, 1] y sea n N. Demostrar que ex-
iste un nico polinomio de grado menor o igual que n que es la mejor
aproximacin polinmica de orden n en el sentido de L
2
.
200 9. ESPACIOS DE HILBERT
Ejercicio 9.6. Sea H un espacio de Hilbert, sea x
1
, . . . , x
n
H
un conjunto ortonormal y sea x H. Demostrar que de todas las sumas
de la forma

i
a
i
x
i
la distancia
|x

i
a
i
x
i
|
2
es mnima precisamente cuando los a
i
son los coecientes de Fourier
de x con respecto a x
1
, . . . , x
n
.
Pueden ser ejercicios interesantes (con sucientes sugerencias) com-
probar que ciertos resultados profundos y necesitados de herramienta
ms o menos sosticada en el contexto de espacios de Banach se pueden
probar de manera elemental, aunque laboriosa, en el contexto de los
espacios de Hilbert. En este sentido se pueden proponer.
Ejercicio 9.7. Sea (x
n
) H una sucesin en un espacio de Hil-
bert. Entonces x
n
x en norma si y slo si x
n
x en la topologa
dbil y
lmsup
n
|x
n
| |x|
Ejercicio 9.8. Se puede probar el Teorema de Eberlein para espa-
cios de Hilbert: Sea (x
n
) H una sucesin en un espacio de Hilbert.
Entonces (x
n
) tiene una subsucesin dbilmente convergente.
Ejercicio 9.9. Tambin se puede probar directamente una de las
consecuencias del PAU: Sea H un espacio de Hilbert y B H. En-
tonces B es acotado si y slo si B es dbilmente acotado, es decir, si
para todo x

, x

(B) est acotado.


CAPTULO 10
Teora espectral en espacios de Hilbert: Operadores
compactos normales
La Teora Espectral alcanza su mxima perfeccin en los operadores
denidos en espacios de Hilbert. En particular los operadores normales,
aquellos que conmutan con su traspuesto, son diagonalizables en un
sentido amplio gracias al Teorema Espectral. Nosotros no llegamos a
desarrollar en este programa el Teorema Espectral para operadores
normales, ya que eso requiere ms tiempo y conocimientos de los que
disponemos en esta asignatura. En cambio, s desarrollamos en este
captulo el Teorema Espectral para operadores normales y compactos.
En presencia de la compacidad el espectro del operador se reduce a una
cantidad a lo sumo numerable de puntos, todos ellos excepto quizs el
0 autovalores, y eso permite que lo que en el Teorema Espectral sea
una integral respecto de una medida vectorial, la medida espectral, se
convierta en una serie mucho ms fcil de construir con tcnicas ele-
mentales. Es frecuente en muchos libros de Anlisis Funcional comen-
zar demostrando el Teorema Espectral para operadores compactos au-
toadjuntos de una manera razonablemente sencilla y a continuacin
demostrar el Teorema Espectral para operadores compactos normales
usando razonamientos sensiblemente ms delicados.
Nosotros hemos preferido seguir [39], con algunas modicaciones,
donde el autor demuestra de manera bastante sencilla directamente el
resultado para operadores compactos normales, y de ah es fcil luego
obtener la versin para operadores autoadjuntos como corolario.
Hemos incluido tambin para terminar el captulo el Teorema es-
pectral para operadores compactos no necesariamente normales, que
no es otra cosa que la versin para operadores compactos en espacios
innito dimensionales del resultado de lgebra lineal que nos dice que
toda forma bilineal (entre espacios nito dimensionales) admite una
representacin diagonal como forma bilineal (es decir, permitiendo ele-
gir dos bases diferentes, una en cada espacio). La demostracin de este
resultado la hemos obtenido de [16].
Antes de obtener los Teoremas Espectrales que forman el punto
principal del captulo necesitamos denir el adjunto de un operador
201
202 10. TEORA ESPECTRAL EN ESPACIOS DE HILBERT
(y lo relacionamos con el traspuesto que hemos venido utilizando has-
ta ahora. Tambin hacemos un estudio de los operadores normales y
autoadjuntos, y sus espectros.
Al nal del captulo se puede estudiar como aplicacin uno de los
problemas que motivaron el comienzo del desarrollo de la Teora Es-
pectral, los llamados problemas de Sturm-Liouville, aunque su estudio
requiere un cierto tiempo.
Operadores normales y autoadjuntos
Definicin 10.1. Sean H, K dos espacios de Hilbert y sea T :
H K un operador. Se dene su adjunto en el sentido de espacio de
Hilbert, al que llamaremos T

, como el nico operador T

: K H
que verica que para todos x H, y K,
x, T

(y)) = T(x), y).


Es necesario ver que T

est bien denido y es lineal y continuo.


Para ver que est bien denido ntese que, dado y K, la aplicacin
x

: X K dada por
x

= T(x), y)
est bien denida, es lineal y continua. Por tanto x

. Por el
Teorema de Representacin de Riesz sabemos que existe z H tal que
x

(x) = x, z) para todo x H. Denimos entonces T

(y) = z. Ahora
es un ejercicio ver que T

es lineal y continuo y que es nico. Tambin


es un ejercicio comprobar que para todos x, y H se tiene
y, T(x)) = T

(y), x).
Se puede sugerir como ejercicio claricar la relacin entre este tras-
puesto en el sentido de operador hilbertiano y el traspuesto habitual
que habamos denido anteriormente. No hay ms que utilizar el teore-
ma de Representacin de Riesz y dibujar un diagrama para comprobar
que ambos traspuestos son (esencialmente) el mismo.
Ejercicio 10.1. Sean K, H espacios de Hilbert, T : H K un
operador. Sea T
B
: K

su traspuesto en el sentido de espacios


de Banach y sea T

: K H su adjunto en el sentido de espacios de


Hilbert. Sean adems R
H
: H H

y R
K
: K K

las isometras
dadas por el Teorema de Representacin de Riesz. Comprobar que T
B

R
K
= R
H
T

.
En todo este captulo entenderemos por T

el adjunto en el sentido
de espacios de Hilbert.
10. TEORA ESPECTRAL EN ESPACIOS DE HILBERT 203
Adems, de ahora en adelante consideraremos exclusivamente op-
eradores T : H H.
Ejercicio 10.2. Sea H un espacio de Hilbert, sean S, T L(H),
K. Entonces
1. (S + T)

= S

+ T

2. (T)

= T

3. (ST)

= T

4. (T

= T
5. T es inversible si y slo si T

es inversible, y en ese caso


(T

)
1
= (T
1
)

Todo es fcil de probar, y adems son esencialmente las mismas cuen-


tas que ya hemos hecho para los adjuntos en el sentido anterior. Otra
posibilidad es considerar los resultados conocidos para los adjuntos an-
teriores y hacer el diagrama conmutativo. Ntese que la aparicin del
conjugado de en el punto 2 est relacionada con el hecho de que la
isometra que aparece en el Teorema de Representacin de Riesz no es
lineal sino conjugada-lineal.
Proposicin 10.2. Si T L(H) entonces
|T

| = |T| y |T

T| = |TT

| = (|T|)
2
.
Demostracin. Se deja como ejercicio probar que |T

| = |T|.
Para todos x, y H se tiene
|T(x)|
2
= T(x), T(x)) = x, T

T(x)) |T

T||x|
2
.
Tomando supremos en x B
H
se tiene
|T|
2
|T

T|.
Como adems
|T

T| |T

||T| = |T|
2
se tiene la igualdad |T|
2
= |T

T|. La otra igualdad se prueba anloga-


mente.
Veamos algunos ejemplos de operadores adjuntos.
Ejemplo 10.3. Si T
k
: L
2
[0, 1] L
2
[0, 1] un operador integral de
Fredholm con ncleo k(, ) : [0, 1] [0, 1] K como en el Ejemplo
7.11. Entonces (T
k
)

es un operador integral de Fredholm con ncleo


k

(x, y) = k(y, x). De nuevo ntese la necesidad de tomar el conjugado


complejo.
204 10. TEORA ESPECTRAL EN ESPACIOS DE HILBERT
Ejemplo 10.4. Si S :
2

2
es el desplazamiento a la derecha
dado por
S(x
1
, x
2
, . . .) = (0, x
1
, x
2
, . . .)
entonces S

es el desplazamiento a la izquierda dado por


S

(x
1
, x
2
, . . .) = (x
2
, x
3
, . . .)
Ejercicio 10.3. Si T L(H) entonces
ker(T) = (Im(T

))

y ker(T

) = (Im(T))

donde M

es el ortogonal de M en el sentido hilbertiano.


En general la composicin de operadores no es conmutativa. Veamos
que los operadores con cierto grado de conmutatividad van a ser pre-
cisamente aquellos para los que podemos dar una descomposicin es-
pectral.
Definicin 10.5. Sea T L(H). Entonces
1. T se dice normal si T

T = TT

.
2. T se dice unitario si T

T = TT

= I, es decir, si T

= T
1
3. T se dice autoadjunto si T

= T.
Claramente los operadores autoadjuntos o unitarios son normales.
En cambio un operador normal no tiene por qu ser unitario ni autoad-
junto. Por ejemplo, sea H = C
2
y sea
T(x) = (x
1
x
2
, x
1
+ x
2
)
Entonces
T

(x) = (x
1
+ x
2
, x
1
+ x
2
)
y es un ejercicio ver que T no es unitario ni autoadjunto.
Observacin 10.6. Si T es normal y S es tal que S

S = I (para
esto S no tiene por qu ser necesariamente unitario, pinsese en el
operador desplazamiento a la derecha del Ejemplo 10.4) entonces U =
STS

es normal. En efecto, en ese caso U

= ST

y por tanto
UU

= STS

ST

= STT

= ST

TS

= ST

STS

= U

U.
Proposicin 10.7. Sea T L(H). Entonces
1. T es normal si y slo si para todos x, y H
T(x), T(y)) = T

(x), T

(y))
2. T es unitario si y slo si para todos x, y H
T(x), T(y)) = x, y) = T

(x), T

(y))
10. TEORA ESPECTRAL EN ESPACIOS DE HILBERT 205
3. T es autoadjunto si y slo si para todos x, y H
T(x), y) = x, T(y))
Demostracin. Si T es normal entonces para todos x, y H
T(x), T(y)) = x, T

T(y)) = x, TT

(y)) = T

(x), T

(y))
Recprocamente, si para todos x, y H
T(x), T(y)) = T

(x), T

(y))
entonces
x, T

T(y)) = T(x), T(y)) = T

(x), T

(y)) = x, TT

(y))
de donde se sigue que TT

(y) = T

T(y) para todo y H, es decir


T es normal. Los otros dos apartados se prueban anlogamente y la
demostracin se deja como ejercicio.
Ejemplo 10.8. Sea H = L
2
[0, 1], sea f L

[0, 1] y sea
T : L
2
[0, 1] L
2
[0, 1]
el operador denido como
T(g) = fg.
T est bien denido y es lineal y continuo, ya que para todo g H
|T(g)|
2
=
__
1
0
[fg[
2
_
1
2
|f|

|g|
2
.
Adems para todos g, h H
g, T

(h)) = T(g), h) =
_
1
0
fgh =
_
1
0
g(fh) = g, fh),
y por lo tanto
T

(g) = fg para todo g H.


Entonces
T

T(g) = [f[
2
g = TT

(g)
y por tanto T es normal. En cierto sentido, todo operador normal es
as (factorizndolo va una isometra adecuada), aunque esto no es
nada obvio y es una consecuencia (o ms bien una reformulacin) del
Teorema Espectral. El lector interesado en este resultado y en general
en el Teorema Espectral debera consultar [25].
206 10. TEORA ESPECTRAL EN ESPACIOS DE HILBERT
Sea H un espacio de Hilbert separable. Ya hemos visto que en ese
caso H es (isomtrico a )
2
. Como consecuencia de los resultados que
probamos en el captulo anterior referidos a bases hilbertianas, queda
claro que todo operador T :
2

2
queda completamente determina-
do si conocemos la matriz doblemente innita A = (a
i,j
)
i,jN
denida
como
a
i,j
= T(e
j
), e
i
)
ya que en ese caso sabemos que para todo x
2
T(x) =

i=1
_

j=1
a
i,j
x
j
_
e
i
.
Anlogamente a lo visto en el Ejercicio 5.1 se puede probar con
facilidad que el operador T

lleva asociado la matriz A


t
, la matriz
conjugada y traspuesta de A de manera que para todo x
2
T

(x) =

i=1
_

j=1
a
j,i
x
j
_
e
i
.
(De nuevo la razn de que aparezca el conjugado complejo es que esta-
mos considerando el adjunto hilbertiano de T).
Entonces resulta claro que T es autoadjunto si y slo si A = A
t
, es
decir, si A es conjugada-simtrica.
Adems T

T viene representado por


A
t
A =

n
k
n,i
k
n,j
y TT

viene representado por


AA
t
=

n
k
i,n
k
j,n
.
As pues T es unitario si y slo si A
t
A = AA
t
= I, es decir, si
(24)

n
k
n,i
k
n,j
=

n
k
i,n
k
j,n
=
i,j
.
Ntese que, en ese caso, si consideramos el vector k
i

2
denido
como k
i
(n) = k
n,i
entonces (24) nos dice que
k
i
, k
j
) =
i,j
,
es decir, T es unitario si y slo si las columnas (y anlogamente las
las) de la matriz A forman una base ortonormal de
2
.
10. TEORA ESPECTRAL EN ESPACIOS DE HILBERT 207
Veremos ms adelante que si T es compacto, entonces T es normal si
y slo si A es diagonalizable. De momento, observando que T es normal
si y slo si A
t
A = AA
t
, queda claro que si A es diagonal entonces T es
normal.
Veamos ahora algunas caracterizaciones de los operadores normales,
unitarios y autoadjuntos que nos sern tiles ms adelante.
Proposicin 10.9. Sea H un espacio de Hilbert complejo y sea
T L(H). Entonces T es autoadjunto si y slo si T(x), x) R para
todo x H.
Demostracin. Si T = T

entonces, para todo x H,


T(x), x) = x, T

(x))) = x, T(x)) = T(x), x)


y por tanto T(x), x) R.
Recprocamente, supongamos que T(x), x) R para todo x H.
Entonces para todo C y para todos x, y H se tiene
T(x + y), x + y) = T(x), x) + T(x), y)+
+T(y), x) +[[
2
T(y), y) R
Por lo tanto, como T(x), x) R y [[
2
T(y), y) R, se sigue que
T(x), y) + T(y), x) R,
es decir
T(x), y) + T(y), x) = T(x), y) + T(y), x) =
= y, T(x)) + x, T(y)) = T

(x), y) + T

(y), x).
Puesto que esto es cierto para todo C, se sigue con facilidad (por
ejemplo tomando = 1 y = y operando) que para todo x, y H
T(x), y) = T

(x), y),
de donde T = T

.
Observacin 10.10. Obviamente el resultado anterior es falso si
H es un espacio de Hilbert real, ya que en ese caso T(x), y) R para
todos x, y H y obviamente hay operadores no autoadjuntos (consid-
rese cualquier matriz cuadrada no simtrica de por ejemplo orden 2).
Proposicin 10.11. Sea T L(H). Si T es autoadjunto entonces
|T| = sup
xB
H
[T(x), x)[.
208 10. TEORA ESPECTRAL EN ESPACIOS DE HILBERT
Demostracin. Sea M = sup
xB
H
[T(x), x)[. Claramente M
|T|. Para la otra desigualdad observemos que para todos x, y H se
tiene
T(x + y), x + y) T(x y), x y) = 2T(x), y) + 2T(y), x) =
= 2T(x), y) + 2y, T

(x)) = 2T(x), y) + 2y, T(x)) =


= 2T(x), y) + 2T(x), y) = 4'T(x), y).
Por lo tanto
4'T(x), y) = T(x + y), x + y) T(x y), x y)
M(|x + y|
2
|x y|
2
) = 2M(|x|
2
+|y|
2
),
donde en la ltima igualdad hemos usado la Ley del Paralelogramo.
Por tanto, si |x| = |y| = 1 tenemos
4'T(x), y) 4M.
Finalmente, si T(x), y) = [T(x), y)[e

, sustituyendo x por xe

obtenemos que para todo x, y H


[T(x), y)[ M
de donde |T| M.
Ntese que, en particular, si T es autoadjunto y T(x), x) = 0 para
todo x H entonces T = 0.
Proposicin 10.12. Sea T L(H). Entonces T es normal si y
slo si para todo x H
|T(x)| = |T

(x)|.
Adems en ese caso
|T
2
| = |T

T| = |T|
2
Demostracin. Si x H, se tiene
|T(x)|
2
|T

(x)|
2
= T(x), T(x)) T

(x), T

(x)) =
= T

T(x), x) TT

(x), x) = (T

T TT

)(x), x).
Como S = T

T TT

es autoadjunto (esto se ve muy fcil), de la


Proposicin 10.11 nos dice que S = 0 si y slo si
S(x), x) = 0 para todo x H,
es decir, si y slo si
(T

T TT

)(x), x) = |T(x)|
2
|T

(x)|
2
= 0 para todo x H,
es decir si y slo si
|T(x)| = |T

(x)| para todo x H.


10. TEORA ESPECTRAL EN ESPACIOS DE HILBERT 209
Adems en ese caso
|T
2
(x)| = |T(T(x))| = |T

(T(x))| para todo x H


y por tanto
|T
2
| = |T

T| = |T|
2
,
donde la segunda igualdad se sigue de 10.2
Proposicin 10.13. Sea T L(H). Entonces T es unitario si y
slo si T es sobre y |T(x)| = |x| para todo x H. Adems en ese
caso |T
1
(x)| = |x| para todo x H y
|T| = |T
1
| = 1.
Demostracin. Para todo x H se tiene
|T(x)|
2
|x|
2
= T(x), T(x)) x, x) =
= T

T(x), x) x, x) = (T

T I)(x), x).
Como S = T

T I es autoadjunto se tiene que S = 0 si y slo si


|T(x)| = |x| para todo x H.
Por tanto, si |T(x)| = |x| para todo x H y T es sobreyectiva,
entonces T

T = I y T es biyectivo, de forma que


TT

= (TT

)(TT
1
) = T(T

T)T
1
= TT
1
= I
y por tanto T es unitario.
Recprocamente, si T es unitario entonces T

T = I y T
1
= T

de
forma que |T(x)| = |x| para todo x H y T es sobreyectiva.
En ese caso adems
|T
1
(x)| = |T

(x)| = |x|
y tomando supremos en x B
H
se tiene
|T| = |T
1
| = 1.

Como quizs ya se haya podido apreciar en este momento puede


haber una cierta analoga entre los operadores en L(H) y los nmeros
complejos, donde el conjugado de un complejo se asocia al adjunto de
un operador. En esta analoga los operadores autoadjuntos se corre-
sponderan con los nmeros reales (coinciden con su conjugado). Eso
motiva el siguiente
210 10. TEORA ESPECTRAL EN ESPACIOS DE HILBERT
Teorema 10.14. Sea H un espacio de Hilbert complejo y T
L(H). Entonces existen dos nicos operadores autoadjuntos R, S
L(H) tales que
T = R + S.
Adems T es normal si y slo si RS = SR, T es unitario si y slo si
RS = SR y R
2
+ S
2
= I y T es autoadjunto si y slo si S = 0.
Tambin tendremos ocasin de usar ms adelante que T es com-
pacto si y slo si R y S son compactos.
Demostracin. Sean
R =
T + T

2
y S =
T T

2
.
Es fcil ver que R y S son autoadjuntos y T = R + S.
Si T = R
t
+ S
t
con R
t
y S
t
autoadjuntos, entonces
T

= (R
t
+ S
t
)

= R
t
S
t
= R
t
S
t
y por tanto
R
t
=
T + T

2
= R y S
t
=
T T

2
= S.
El resto de la demostracin se deja como ejercicio.
Con estos conocimientos previos acerca de operadores autoadjun-
tos y normales a nuestra disposicin, podemos desarrollar la Teora
Espectral de operadores normales compactos.
Lema 10.15. Sea H un espacio de Hilbert complejo y sea T L(H).
Entonces (T) si y slo si (T

).
Si adems T es normal, entonces para todo C se tiene que
ker(T I) = ker(T I)

= ker(T

I)
Demostracin. La Proposicin 8.3 nos dice que un operador es
inversible si y slo si su adjunto lo es. Por lo tanto T I es inversible
si y slo si (T I)

= T

I es inversible. A partir de ah el primer


enunciado se sigue inmediatamente.
Supongamos ahora que T es normal. Entonces es fcil ver que TI
tambin es normal. Por lo tanto, la Proposicin 10.12 nos dice que para
todo x H
|(T I)(x)| = |(T I)

(x)|,
y de ah se sigue que ker(T I) = ker(T I)


10. TEORA ESPECTRAL EN ESPACIOS DE HILBERT 211
Teorema 10.16. Sea H un espacio de Hilbert y sea 0 ,= T L(H)
un operador compacto y autoadjunto. Entonces al menos uno de los
nmeros |T| o |T| es un autovalor de T.
Demostracin. Sea T como en la hiptesis. De la Proposicin
10.11 se sigue que existe una sucesin (x
n
) B
H
tal que
[T(x
n
), x
n
)[ |T|.
Puesto que T(x), x) R para todo x H, tomando una subsucesin si
fuera necesario podemos suponer que T(x
n
), x
n
) , donde = |T|
o = |T|. Como [[ = |T|, se tiene
0 |(T I)(x
n
)|
2
= |T(x
n
)|
2
2T(x
n
), x
n
) +
2
|x
n
|
2

|T|
2
|x
n
|
2
+
2
|x
n
|
2
2T(x
n
), x
n
)
2
2
2T(x
n
), x
n
) 0
de donde se obtiene que |(T I)(x
n
)| 0.
Como adems T es compacto, existe una subsucesin (x
n
k
)
k
(x
n
)
y un elemento y H tales que T(x
n
k
) y, y por tanto, como T(x
n
k
)
x
n
k
0, se sigue que x
n
k

y

y
T(y) = y
lo que termina la demostracin.
Vamos a ver ahora que, usando el resultado anterior, podemos decir
algo similar de los operadores compactos normales.
Teorema 10.17. Sea H un espacio de Hilbert complejo y sea 0 ,=
T L(H) un operador normal y compacto. Entonces T admite al
menos un autovalor distinto de 0.
Demostracin. Sea T = R+S con R, S compactos y autoadjun-
tos y SR = RS (vase el Teorema 10.14). Supongamos inicialmente que
R ,= 0. Entonces, el Teorema 10.16 nos dice que R tiene al menos un
autovalor ,= 0. Sea M = ker(RI). El subespacio M es invariante
por S ya que para todo x M, puesto que SR = RS, se tiene
RS(x) = SR(x) = S(x) = S(x)
luego S(x) ker(R I) = M.
Entonces S
[
M
: M M es un operador compacto y autoadjunto
del espacio de Hilbert M en s mismo. Aplicando de nuevo el Teorema
10.16 tenemos que existe
e
(S
[
M
). Sea ahora 0 ,= x M tal que
S(x) = x. Entonces
T(x) = (R + S)(x) = ( + )(x)
212 10. TEORA ESPECTRAL EN ESPACIOS DE HILBERT
luego +
e
(T)
Finalmente, si R = 0, entonces S ,= 0 y por tanto el Teorema 10.16
nos dice que S tiene un autovalor ,= 0. Entonces es autovalor de
T = S.
Ya vimos en la Proposicin 8.19 que si T es compacto entonces para
todo K existe n
0
N tal que para todo n n
0
ker(T I)
n
= ker(T I)
n
0
.
Dado un autovalor denimos entonces su ndice () como el
primer n
0
N para el que ocurre eso, es decir () es el nico natural
tal que
ker(T I)
n
ker(T I)
n+1
para todo 0 n < ()
y
ker(T I)
n
= ker(T I)
n+1
para todo n ()
Proposicin 10.18. Si T L(H) es normal entonces para todo

e
(T) se tiene que () = 1.
Demostracin. Sea
e
(T) y sea x ker(T I)
2
. Slo hay
que ver que x ker(T I).
Puesto que T I es normal, aplicando la Proposicin 10.12 se
tiene que
|(T

I)(T I)(x)| = |(T I)(T I)(x)| = 0.


Por lo tanto
|(T I)(x)|
2
= (T I)(x), (T I)(x)) =
= (T

I)(T I)(x), x) = 0
lo que implica que x ker(T I) y termina la demostracin.
Definicin 10.19. Sea T L(H) un operador normal. Dado

e
(T) denimos su multiplicidad como la dimensin de ker(T I).
Observacin 10.20. En realidad se dene la multiplicidad alge-
braica como dimker(T I)
()
y la multiplicidad geomtrica como
dimker(T I). En el caso de que T sea normal (el nico que es-
tudiaremos en esta memoria), la Proposicin 10.18 nos garantiza que
ambos nmeros coinciden, y los denominamos multiplicidad simple-
mente.
Proposicin 10.21. Si T L(H) es normal y ,
e
(T) en-
tonces
ker(T I)ker(T I)
10. TEORA ESPECTRAL EN ESPACIOS DE HILBERT 213
Demostracin. En efecto, para todos x ker(T I), y
ker(T I), usando que
e
(T

) (Lema 10.15), se tiene


y, x) = y, x) = y, T

(x)) = T(y), x) = y, x) = y, x).


Puesto que ,= , se sigue que y, x) = 0.
Necesitamos un lema ms antes de enunciar y demostrar el Teorema
Espectral para operadores compactos normales.
Lema 10.22. Sea M H un subespacio cerrado y sea P : H M
la proyeccin ortogonal. Entonces P

= P
Demostracin. Para todo x H, P(x) y x P(x) son ortogo-
nales, es decir
P(x), x P(x)) = 0
y por tanto
P(x), x) = P(x), P(x)) = |P(x)|
2
.
Ahora la Proposicin 10.9 nos garantiza que P es autoadjunto.
Teoremas Espectrales para Operadores Compactos
Teorema 10.23. (Teorema espectral para operadores compactos
normales) Sea H un espacio de Hilbert complejo y T L(H) un oper-
ador compacto normal. Sea (
n
)
n
C la sucesin (nita o innita) de
autovalores, ordenados por mdulos decrecientes y contado cada uno de
ellos tantas veces como su multiplicidad. Entonces existe una sucesin
(e
n
)
n
H con tantos elementos como la sucesin (
n
)
n
, de manera
que para cada n se tiene que T(e
n
) =
n
e
n
, es decir, e
n
es un autovalor
de
n
. Adems la sucesin (e
n
)
n
es un sistema ortonormal y se verica
que para todo x H
(25) T(x) =

n
x, e
n
)e
n
Demostracin. Sea H y T como en la hiptesis. Por el Teorema
10.17 sabemos que
e
(T) ,= , y por el Teorema 8.22 sabemos que

e
(T) es a lo sumo numerable. Sea (
n
)
n
la sucesin (nita o innita)
de autovalores. De nuevo por el Teorema 8.22 sabemos que si (
n
) es
innita entonces
n
0. Por tanto, siempre tiene podemos elegir la
sucesin (
n
) ordenada de tal forma que
[
1
[ [
2
[ [
3
[ ,
214 10. TEORA ESPECTRAL EN ESPACIOS DE HILBERT
bien entendido que esta ordenacin no tiene por qu ser nica ya que
pueden existir autovalores distintos con el mismo mdulo. Para todo n,
sea m(n) la multiplicidad (algebraica o geomtrica) de
n
. Sea entonces
(
n
) la sucesin (nita o innita) formada por m(1) copias de
1
, a
continuacin m(2) copias de
2
, etc. Est claro que (
n
) es innita si y
slo si (
n
) es innita, y que en ese caso tambin
n
0. Para todo n
sea N(n) = ker(T
n
I). Por denicin de multiplicidad dimN(n) =
m(n). Para cada n, sea
e
n,1
, e
n,2
, . . . , e
n,m(n)
H
una base ortonormal de N(n). Sea entonces (e
n
) H la sucesin (nita
o innita) formada por
(e
1,1
, e
1,2
, . . . , e
1,m(1)
, e
2,1
, e
2,2
, . . . , e
2,m(2)
,
Por la construccin de (e
n
) y (
n
) queda claro que para todo n
T(e
n
) =
n
e
n
.
Adems, puesto que para todo j ,= i se tiene que N(j)N(i) (Proposi-
cin 10.21) se sigue que la familia (e
n
) es ortonormal.
Consideramos ahora el operador T
0
: H H dado por
T
0
(x) =

n
x, e
n
)e
n
para todo x H.
Aserto: T
0
est bien denido y es continuo.
Demostracin del Aserto: Para todo x H, utilizando la desigual-
dad de Bessel se tiene

n
[
n
x, e
n
)[
2
[
1
[
2

n
[x, e
n
)[
2
[
1
[|x|
2
,
y por el Teorema de Riesz-Fischer se sigue el Aserto.
De hecho se sigue de la implicacin fcil del Teorema 10.24, que
podramos haber probado previamente, que T
0
es compacto y normal,
aunque no lo necesitamos.
Veamos que T = T
0
.
Sea M = span[(e
n
)], el subespacio vectorial cerrado generado por
los (e
n
). Por el Teorema de la Proyeccin Ortogonal, M est comple-
mentado en H. Sea P : H M la proyeccin ortogonal de H sobre
M. Es claro que para todo n
T
0
(e
n
) =
n
e
n
= T(e
n
).
10. TEORA ESPECTRAL EN ESPACIOS DE HILBERT 215
Por tanto, por linealidad y continuidad se sigue que T
[
M
= T
0[
M
, es
decir
T
0
P = TP
Adems, usando que P

= P (Lema 10.22), se tiene que para todo


x H
T
0
(x) =

n
x, e
n
)e
n
=

n
x, P(e
n
))e
n
=
=

n
P(x), e
n
)e
n
= T
0
P(x).
Por lo tanto T
0
= T
0
P y de ah TP = T
0
.
Adems, como PT(x) M para todo x H y como (e
n
) es una
base ortonormal de M (entendindola como base hilbertiana si es una
coleccin innita), se tiene
PT(x) =

n
PT(x), e
n
)e
n
=

n
x, T

P(e
n
))e
n
.
Adems, por el Lema 10.15 se tiene que
T

P(e
n
) = T

(e
n
) =
n
e
n
y por tanto
PT(x) =

n
x, e
n
)e
n
= T
0
(x)
As pues TP = PT = T
0
y, tomando adjuntos,
PT

= T

P.
Por lo tanto, dos cualesquiera de T, T

y P conmutan entre si y de
aqu se deduce fcilmente que T T
0
= T TP es normal.
Finalmente, supongamos que T ,= T
0
. Como T T
0
es normal,
compacto y distinto de 0, el Teorema 10.17 nos dice que T T
0
tiene
al menos un autovalor ,= 0. Sea 0 ,= e H un autovector de .
Como (T T
0
)(e
n
) = 0 para todo n y (T T
0
)e = e, se tiene que
ee
n
para todo n. En efecto,
e
n
, e) = e
n
, e) = e
n
, (T T
0
)

e) = (T T
0
)e
n
, e) = 0, e) = 0
Por lo tanto,
T
0
(e) =

n
e, e
n
)e
n
= 0
y de ah
T(e) = (T T
0
)e = e.
216 10. TEORA ESPECTRAL EN ESPACIOS DE HILBERT
De forma que es un autovalor de T. Puesto que la sucesin
n
estaba
formada por todos los los autovalores
n
(contados con su multiplici-
dad), se tiene que =
j
para algn j, y por tanto e N(j). Pero
entonces es imposible que e sea distinto de 0 y simultneamente sea
ortogonal a todos los elementos de una base ortogonal de N(j).
De esta contradiccin se sigue que T = T
0

En el caso especial de operadores compactos autoadjuntos, el Teo-
rema anterior queda igual, con la salvedad de que ahora todos los au-
tovalores (
n
) (y por tanto tambin los (
n
)) son reales.
Es fcil ver, y se puede proponer como ejercicio, que el Teorema
10.23 (y su correspondiente versin para operadores compactos au-
toadjuntos) son caracterizaciones, de manera que dado un operador
T : H H, T es compacto y normal si y slo si T se puede escribir
en la forma 25, y T es compacto y autoadjunto si y slo si T se puede
escribir en la forma 25, con (
n
) R.
Veamos para acabar este captulo qu se puede decir si T es un
operador compacto, no necesariamente normal. Sugerimos para ello la
demostracin de [16]. Se puede dar otra demostracin, como en [39],
utilizando el Teorema 10.23 y la descomposicin polar de un operador.
Teorema 10.24. Sea H un espacio de Hilbert. Entonces un op-
erador T L(H) es compacto si y slo si existen sucesiones (nitas
o innitas) (
n
) C, (e
n
) H y (f
n
) H con el mismo nmero
de ndices tales que
n
0 si es innita y (e
n
) y (f
n
) son familias
ortonormales de forma que T se puede escribir en la forma
(26) T(x) =

n
x, e
n
)f
n
Aplicaciones
Una aplicacin clsica de la Teora espectral de operadores com-
pactos en espacios de Hilbert es la resolucin de los llamados Problemas
de Sturm-Liouville. En los libros de nuestra bibliografa, se pueden ver
presentaciones de esta aplicacin en [31, Apendix C] o en [13, Section
II.6].
Prcticas sugeridas.
10. TEORA ESPECTRAL EN ESPACIOS DE HILBERT 217
Ejercicio 10.4. 1. Si S, T son autoadjuntos entonces S +T
es autoadjunto. Adems ST es autoadjunto si y slo si S y T
conmutan.
2. Si S, T son unitarios entonces ST es unitario. Buscar alguna
condicin necesaria y suciente sobre S y T para que S + T
sea unitario.
3. Si S y T son normales y adems S conmuta con T

y T con-
muta con S

entonces S + T y ST son normales.


Ejercicio 10.5. Si (T
n
) L(H) es una sucesin de operadores
autoadjuntos (resp. unitarios, normales) tal que T
n
T entonces T es
autoadjunto (resp. unitario, normal). Sugerencia: Si T
n
T entonces
T

n
T

.
Ejercicio 10.6 (Operador de Volterra). Sea k : [0, 1] [0, 1] R
la funcin caracterstica del conjunto (x, y) [0, 1][0, 1] tales que y <
x. Sea V : L
2
L
2
el operador integral dado por
V (f)(s) =
_
1
0
f(t)k(s, t)dt.
Ntese que V (f)(s) =
_
s
0
f(t)dt. Se pide
1. Calcular V

2. Demostrar que V no tiene autovalores.
3. Demostrar que V es compacto.
Ejercicio 10.7. Si S :
2

2
calcular S

, SS

y S

S. Adems
demostrar
1.
e
(S

) = z C; [z[ < 1
2.
e
(S) =
3. (S) = (S

) = z C; [z[ 1
Ejercicio 10.8. Sea D :
2

2
el operador diagonal dado por
D(a) = (d
n
a
n
)
n
con d
n
= 2
n
, y sea S :
2

2
el desplazamiento a la derecha. Sea
T = SD. Demostrar que T es compacto,
e
(T) = y (T) = 0.
Ejercicio 10.9 (Teorema de Hellinger-Tplitz). Sea H un espa-
cio de Hilbert y sea T : H H un operador lineal y autoadjunto.
Entonces T es continuo.
Ejercicio 10.10. Sea H un espacio de Hilbert complejo y sea T
L(H) un operador compacto normal no nulo. Entonces existe
e
(T)
tal que [[ = |T|.
CAPTULO 11
Requisitos
219
CAPTULO 12
Funciones holomorfas
Definicin 12.1. Sea A C un conjunto abierto. Una funcin
f : A C
se dice derivable en un punto z
0
A si el siguiente lmite existe
lm
zz
0
f(z) f(z
0
)
z z
0
.
En ese caso, a dicho lmite se le denomina la derivada de f en z
0
y se denota por f
t
(z
0
).
Si f es derivable en todo punto z A, se dice que f es holomorfa en
A. Si existe un entorno de z
0
tal que f es holomorfa en dicho entorno,
se dice que f es holomorfa en z
0
.
Usaremos la notacin H(A) para denotar al conjuntro de las fun-
cions holomorfas en A.
Definicin 12.2. Dada una funcin f : A C, podemos denir
su parte real como
u(z) = Re(f(z))
y su parte imaginaria como
v(z) = Im(f(z))
. Se tiene entonces trivialmente
f(z) = u(z) + v(z)
Teorema 12.3 (Cauchy-Riemann). Sea A C un abierto, sea
f : A C una funcin y sea z
0
= (x
0
, y
0
) A. Entonces f
t
(z
0
) existe
si y slo si f es diferenciable en (x
0
, y
0
)(en el sentido real como funcin
vectorial de dos valores reales) y adems se cumplen las condiciones de
Cauchy-Riemann:
u
x
=
v
y
,
u
y
=
v
x
.
Por tanto, si
u
x
,
v
y
,
u
y
,
v
x
existen y son continuas en z
0
y verican
las condiciones de Cauchy-Riemann, entonces f es derivable en Z
0
.
221
222 12. FUNCIONES HOLOMORFAS
Adems, en ese caso se tiene
f
t
(z
0
) =
u
x
+
v
x
=
f
x
=
v
y

u
y
=
f
y
Proposicin 12.4. PROPIEDADES BSICAS DE LAS FUN-
CIONES HOLOMORFAS (MAR)
CAPTULO 13
Funciones meromorfas
223
CAPTULO 14
Teorema de la Aplicacin abierta
225
CAPTULO 15
Topologa compacto-abierta
227
CAPTULO 16
Funciones Armnicas
229
Bibliografa
[1] C. D. Aliprantis and K. C. Border, Innite Dimensional Analysis. A Hitch-
hiker guide, Springer 1999.
[2] T. M. Apostol, Anlisis Matemtico Revert 1991.
[3] K. Arrow and G. Debreu, Existence of an equilibrium for a competitive econ-
omy, Econometrica 22 (1954), 265290.
[4] P. Artzner, F. Delbaen, J.M. Eber, D. Heath, Coherent measures of risk,
Math. Finance 9 (1999), 203228.
[5] G. Bachman y L. Narichi, Functional Analysis, Academic Press, 1966.
[6] S. Banach, Sur les oprations dans les ensembles abstraits et leurs applications
aux quations intgrales, Fundamenta Math., 3 (1922) 131-181.
[7] S. Banach, Theorie des oprations linaires, Warszawa, 1932.
[8] B. Beauzamy, Introduction to Banach spaces and their geometry, North-
Holland, 1985.
[9] F. Black and M. Scholes, The valuation of option contracts and a test of
market eciency, J. Finance 27 (1972), 399417.
[10] F. Black and M. Scholes, The pricing of options and corporate liability, J.
Political Economy, 81 (1973), 637659.
[11] F. Bombal, Anlisis Funcional: una perspectiva histrica, Proceedings of the
Seminar of Mathematical Analysis 2002-2003, Secretariado de Publicaciones,
Universidad de Sevilla (2993) 81117.
[12] G. Choquet, Cours dAnalyse. Tome II. Topologie., Masson et C
ie
, Paris,
1964.
[13] J. B. Conway, A Course in Functional Analysis Springer 1990.
[14] F. Delbaen y W. Schachermayer, The fundamental theorem of asset pricing
for unbounded stochastic processes, Math. Ann. 312 (1998) 215250.
[15] J. Diestel, Sequences and series in Banach Spaces, Springer-Verlag 1984.
[16] J. Diestel, H. Jarchow and A. Tonge, Absolutely Summing Operators, Cam-
bridge Stud. Adv. Math. 43, Cambridge Univ. Press, Cambridge 1995.
[17] D. Due, Security Markets. Stochastic models Academic Press 1988.
[18] P. Eno, On the invariant subspace problem for Banach spaces, Acta Math
158 (1987) 213313.
[19] P. Eno, A counterexample to the Approximation problem for Banach spaces,
Acta Math 130 (1973) 309317.
[20] P. Eno y V. Lomonosov, Some aspects of the invariant subspace problem.
Handbook of the geometry of Banach spaces, Vol. I, 533559, North-Holland,
Amsterdam, 2001.
[21] M. Fabian, P. Habala, P. Hajek, V. Montesinos Santalucia, J. Pelant, V. Zizler,
Functional Analysis and innite dimensional geometry, Springer, 2001.
231
232 BIBLIOGRAFA
[22] C. Goman and G. Pedrick, First Course in Functional Analysis, Prentice-
Hall 1965.
[23] A. Grothendieck, Rsum de la thorie mtrique des produits tensoriels
topologiques, Bol. Soc. Mat. So Paulo 8 (1953) 179.
[24] P. Habala, P. Hajek, V. Zizler, Introduction to Banach Spaces I, Matfyzpress
Univerzity Karlovy, Prague, 1996.
[25] P. R. Halmos, What does the spectral theorem say?, Amer. Math. Monthly
70 (1963) 241247.
[26] J. Horvath, Topological Vector Spaces and Distributions, Addison-Wesley
Publishing Company 1966.
[27] R. C. James, A non-reexive Banach space isometric with its second conjugate
space, Proc. Nat. Acad. Sci. USA, 37 (1951) 174177.
[28] P. E. Kopp, Martingales and stochastic integrals, Cambridge University Press
1984.
[29] S. Lang, Real and Functional Analysis, Springer-Verlag 1993.
[30] R. Larsen, Functional Analysis: An Introduction, Marcel Dekker, New York
1973.
[31] B. V. Limaye, Functional Analysis, Wiley Eastern Ltd., 1981.
[32] J. Lindenstrauss y L. Tzafriri, On the complemented subspace problem, Israel
J. Math 9 (1971) 263269.
[33] R. Meise y D. Vogt, Introduction to Functional Analysis, Oxford Science Pub-
lications, 1997.
[34] R. Merton, The theory of rational option pricing, Bell. J. Econ. Manage. Sci.,
4 (1973), 141183.
[35] J. Neveu, Discrete parameter martingales, North-Holland 1975.
[36] R. I. Ovsepian y A. Peczyski, On the existence of a fundamental total and
bounded biorthogonal sequence in every separable Banach space, and related
constructions of uniformly bounded orthonormal systems in L
2
, Studia Math.
54 (1975), no. 2, 149159.
[37] G. K. Pedersen, Analysis Now, Springer-Verlag 1989.
[38] A. M. Plichko y D. Yost, Complemented and uncomplemented subspaces of
Banach spaces. III Congress on Banach Spaces (Jarandilla de la Vera, 1998).
Extracta Math. 15 (2000), 335371.
[39] J. R. Ringrose, Compact Non-Self-Adjoint Operators Van Nostrand Reinhold
Company, London, 1971.
[40] W. Rudin, Principios de Anlisis Matemtico, McGraw-Hill 1978.
[41] W. Rudin, Functional Analysis, McGraw-Hill 1973.
[42] W. Rudin, Real and Complex Analysis, McGraw-Hill 1966.
[43] W. Shachermayer, A Hilbert space proof of the fundamental theorem of asset
pricing in nite discrete time, Insurance Math. Econ. 11 (1992) 249257.
[44] H. H. Schaefer, Topological Vector Spaces, Springer-Verlag 1971.

Você também pode gostar